Contracts (Forbes) - 2016

You might also like

Download as pdf or txt
Download as pdf or txt
You are on page 1of 94

Table of Contents

Formation of the Contract .......................................................................................... 4


INTENTION TO CREATE LEGAL RELATIONSHIPS: .....................................................................................4
Balfour v Balfour [1919] EN .............................................................................................................................................6
Merrit v Merrit [1970] CA USA........................................................................................................................................6
FAMILY LAW ACT, RSO 1990, c F. 3 ..............................................................................................................................6
Jones v Padvattan [1969] CA EN.....................................................................................................................................6
Carlill v Carbolic Smoke Ball Company [1893] ........................................................................................................7
Rose and Frank Company ..................................................................................................................................................8
Offers ................................................................................................................................................................................8
Offer Mechanics ....................................................................................................................................................... 8
Pharmaceutical Society v Boots [1953] UK ............................................................................................................ 10
Boyer and Co. v Duke [1905]......................................................................................................................................... 11
Harvey v Facey [1893] CA .............................................................................................................................................. 11
Johnston Bros v Rogers Bros [1899] ON CA .......................................................................................................... 11
Carlill v Carbolic Smoke Ball Company [1893] ..................................................................................................... 12
The Santanita [1895] EN ................................................................................................................................................. 12
MJB Enterprises v Defense Construction [1999] ................................................................................................. 12
Duration of the Offer .......................................................................................................................................... 13
Dickinson v Dodds [1876] .............................................................................................................................................. 13
Petterson v Pattberg Bad Law [1928]....................................................................................................................... 13
Errington v Errington [1952] UK CA ......................................................................................................................... 14
Shuey v USA [1875] US SC .............................................................................................................................................. 14
Livingstone v Evans [1925] ........................................................................................................................................... 14
Barrick v Clark [1951] (Lapse)..................................................................................................................................... 15
Machester DC v Commercial & General Investments ........................................................................................ 15
Acceptance .................................................................................................................................................................. 16
Communication of Acceptance ....................................................................................................................... 18
Felthouse v Bindley [1862] (Ct NS) ........................................................................................................................... 18
Cole McIntyre-Norfleet v Holloway [1919] ............................................................................................................ 19
Household Insurance Co v Grant [1879] EN CA ................................................................................................... 19
Entores v Miles [1955] EN CA....................................................................................................................................... 19
Eastern Power v Azienda [1999] Ont CA................................................................................................................. 20
Electronic Commerce Act SO 2000 c 17 ................................................................................................................... 20
Time and Place ..................................................................................................................................................... 20
Eliason v Henshaw [1819] USSC.................................................................................................................................. 20
Manchester DC v Commercial & General Investments [1969] ..................................................................... 21
Holwell Securities Ltd. v Hughes UK CA [1974] ................................................................................................... 21
Henthorn v Fraser [1892] CA........................................................................................................................................ 21
Acceptance of Unilateral Offers ..................................................................................................................... 22
R v Clark [1927] Aus HC .................................................................................................................................................. 22
Consideration ............................................................................................................................................................ 23
Tobias v Dick & T Eaton Co [1937] Man KB ........................................................................................................... 27
Sufficiency of Consideration ............................................................................................................................ 27
Thomas v Thomas [1842] QB ....................................................................................................................................... 27

1
Stilk v Myrick [1809] EN ................................................................................................................................................. 27
Hamer v Sidway [1891] NYCA ...................................................................................................................................... 28
White v Bluett [1853] EN Ex Ct .................................................................................................................................... 28
Dalhousie v Boutilier Estate [1934] SCC.................................................................................................................. 28
The Seal.................................................................................................................................................................................... 29
Linton v Royal Bank of Canada [1967] HCt CAN.................................................................................................. 29
Past Consideration .............................................................................................................................................. 29
Roscorla v Thomas [1842] QB ...................................................................................................................................... 29
Lampleigh v Brathwait [1615] UK KB ...................................................................................................................... 29
Performance of an Existing Duty .................................................................................................................. 30
Stilk v Myrick [1809] ......................................................................................................................................................... 30
Hartley v Ponsonby [1857] UK..................................................................................................................................... 30
New Zealand Shipping v Satterthwaite [1975] .................................................................................................... 30
Williams v Roffey Bros. & Nicholls [1991] QB CA ............................................................................................... 31
Greater Fredericton Airport Authority v NAV Canada [2008] NBCA ........................................................ 31
Wind River Ventures v British Columbia [2009] BCSC .................................................................................... 31
Foakes v Beer UK [1884] ................................................................................................................................................. 32
Mercantile Laws Amendment Act RSO 1990 ......................................................................................................... 32
Promissory Estoppel ........................................................................................................................................... 33
Central London Property Trust v High Trees [1947] KB ................................................................................. 33
John Burrows Ltd v Subsurface Surveys [1968] SCR ........................................................................................ 33
D&C Builders v Rees UK [1965] ................................................................................................................................... 34
Coombe v Coombe [1951] CA ....................................................................................................................................... 34
Crabb v Arun DC [1976] CA ........................................................................................................................................... 34
Walton Stores v Maher [1988] Aus HC NOT LAW IN CANADA .................................................................... 35
Capacity to Contract ............................................................................................................................................... 35
Section 3 of Sale of Goods Act RSO 1990, c s 1...................................................................................................... 36
Nash v Inman [1908] KB CA .......................................................................................................................................... 36
Toronto Marlboroughs Hockey Club v Tonelli [1976] OR .............................................................................. 37
6. Certainty or Ascertainability of Terms ...................................................................................................... 37
Complete Contract, Incomplete Terms ........................................................................................................ 39
Scammel & Nephew v Ouston UK [1941] AC HL.................................................................................................. 39
Hillas v Arcos [1932] HL.................................................................................................................................................. 39
Incomplete Contract, Agreement to Negotiate ........................................................................................... 39
Walford v Miles [1992] HL ............................................................................................................................................. 39
Empress Towers v BNS [1990] CA – Confined to specific facts in Edper Brascan Corp ................... 40
Edper Brascan Corporation v 117373 Canada Inc. [2000] SCJ ON ............................................................. 40
Terms appear incomplete (might be a contract) .................................................................................... 40
Calvin Consolidated Oil & Gas Co v Manning [1959] SCR................................................................................ 40
British American Timber Co v Elk River Timber Co [1933] BCCA .............................................................. 41
Green v Ainsmore Consolidated [1951] BCSC ...................................................................................................... 41
C. Terms of a Contract .............................................................................................. 41
Interpreting Written Contracts – Parole Evidence Rule......................................................................... 47
Pym v Campbell [1856].................................................................................................................................................... 47
Morgan v Griffith [1871] ................................................................................................................................................. 47
Hawrish v Bank of Montreal [1969] SCR ................................................................................................................. 48
Southern Resources Ltd v Techomin Australia [1990] Supp 21 .................................................................. 48

2
Sattva Capital v Creston Moly Corp. [2014] SCC .................................................................................................. 49
Implied Terms of Contract................................................................................................................................... 50
Codelfa Construction v State Rail Authority [1982] High Ct of Aus ........................................................... 50
Liverpool City Council v Irwin [1976] HL ............................................................................................................... 50
Duty to Perform in Good Faith ........................................................................................................................... 51
Bhasin v Hrynew and Heritage Educational Funds ............................................................................................ 51
Exclusion Clauses..................................................................................................................................................... 52
McCutcheon v MacBrayne Ltd. [1964] HL .............................................................................................................. 53
Parker v South Eastern RY [1877] CA....................................................................................................................... 53
George Mitchell Ltd. v Finney Lock Seeds Ltd. [1983] HL............................................................................... 55
Hunter v Engineering v Syncrude Canada [1989] SCR ..................................................................................... 56
Tercon Contractors v British Columbia [2010] SCC – Most recent Canada ............................................ 56
Defects In Contractual Relations ............................................................................... 57
Misrepresentation:.................................................................................................................................................. 59
Varieties of Misrepresentation ....................................................................................................................... 59
Redgrave v Hurd [1881] UK – Fraudulent Misrepresentation ..................................................................... 59
Esso Petroleum Co v Mardon [1976] EN CA – Negligent Misrepresentation ........................................ 60
Remedies ................................................................................................................................................................. 60
Redican v Nesbitt SCC 1924 ........................................................................................................................................... 60
O’Flaherty v McKinley [1953] Nfld CA – .................................................................................................................. 61
Leaf International Galleries [1950] KB CA ............................................................................................................. 61
Heilbut Symons & Co v Buckleton [1913] HL........................................................................................................ 62
Shanklin Pier Ltd v Detel Products Ltd [1951] KB ............................................................................................. 62
Mistake ......................................................................................................................................................................... 63
Common Mistake ................................................................................................................................................. 64
Bell v Lever Bros (1932) HL .......................................................................................................................................... 64
Solle v Butcher [1950] KB............................................................................................................................................... 65
Great Peace Shipping Ltd. v Tsavliris (International) Ltd. [2002] CA ....................................................... 65
Miller Paving Ltd. v Gottardo Construction [2007] ON CA (obiter) ........................................................... 66
Mutual Mistake .................................................................................................................................................... 66
Raffles v Wichelhaus [1864] UK .................................................................................................................................. 66
Smith v Hughes [1871] QB ............................................................................................................................................. 67
Unilateral Mistake .............................................................................................................................................. 67
Hartog v Collins & Shields [1939] UK ....................................................................................................................... 67
Ron Engineering v R [1981] SCC ................................................................................................................................. 68
Cundy v Lindsay [1878] HL............................................................................................................................................ 68
Ingram v Little [1961] UK QB ....................................................................................................................................... 68
Lewis v Averay [1972] QB CA (probably bad law in Canada) ....................................................................... 69
Mistake as to Documents (Non Est Factum)............................................................................................. 69
Saunders v Anglia Building Society [1971] HL ..................................................................................................... 70
Marvco Color Research v Harris [1982] SCR ......................................................................................................... 70
Frustration ............................................................................................................................................................. 71
Illegality ....................................................................................................................................................................... 71
Common Law Illegality ..................................................................................................................................... 73
Oldfield v Transamerica Life [2002] SCC ................................................................................................................ 74
In the Matter of Baby M [1988] NJSC ........................................................................................................................ 75
Restraint of Trade Clauses ............................................................................................................................... 75
Gordon v Ferguson [1961] NSSC ................................................................................................................................. 75

3
Shafron v KRG Insurance Brokers [2009] SCC ..................................................................................................... 75
Illegality Based on Statute ............................................................................................................................... 76
Yango Pastoral Co v First Chicago Australia Ltd. [1978] AUS ....................................................................... 76
St. John Shipping Corp. v Joseph Rank Ltd. [1956] UK QB .............................................................................. 76
Ashmore, Benson, Pease & Co v Dawson Ltd. [1973] UK CA ......................................................................... 77
Unconscionability and Undue Influence ........................................................................................................ 77
Marshall v Canada Permanent Trust Co [1968] AL SC .............................................................................. 80
Macaulay v Schroader Music [1974] EN ............................................................................................................. 80
Lloyd’s Bank v Bundy [1975] HL ................................................................................................................................. 80
Royal Bank of Scotland v Etridge [2001] HL – Undue Influence ......................................................... 81
E. Enforcement, Breach and Remedy ........................................................................ 82
Privity ........................................................................................................................................................................... 82
Tweedle v Atkinson [1861] UK .................................................................................................................................... 83
New Zealand Shipping Co. v Satterthwaite & Co [1975] AUS Trust Case ................................................ 83
London Drugs v Kuehne & Nagel Ltd. [1992] SCR .............................................................................................. 84
1. Frustration ......................................................................................................................................................... 85
Taylor v Caldwell [1863] UK – Old test .................................................................................................................... 86
Davis Contractors v Fareham UDC –[1956] UK HL Principle Case ............................................................. 86
Frustrated Contract Act, RSO 1990 ............................................................................................................................ 87
3) Remedy............................................................................................................................................................... 87
Damages ................................................................................................................................................................. 89
Ruxley Electronics Ltd v Forsyth [1994] UK HL .................................................................................................. 89
Howe v Teefy [1927] EN.................................................................................................................................................. 90
Groves v John Wunder Co. [1939] USA – Forbes thinks wrongly decided .............................................. 90
Sale of Goods Act, RSO 1990 .......................................................................................................................................... 91
Thompson Ltd. V Robinson Ltd. [1955] UK ........................................................................................................... 91
Payzu v Saunders [1919] KB CA .................................................................................................................................. 91
Hadley v Baxendale [1854] EN..................................................................................................................................... 91
Victoria Laundry Ltd v Newman Ltd [1949] KB CA ........................................................................................... 92
Fidler v Sun Life [2006] SCC .......................................................................................................................................... 92
Jackson v Horizon Holidays [1975] EN CA ............................................................................................................. 92
Whiten v Pilot Insurance [2002] SCC ........................................................................................................................ 93
Specific Performance ......................................................................................................................................... 93
Falke v Gray [1859]............................................................................................................................................................ 93
Co-operative Insurance Society v Argyll Stores [1998] HL ............................................................................ 93
Injunction: .............................................................................................................................................................. 93
Warner Bros v Nelson [1937] EN (Bette Davis Case) ....................................................................................... 94

Formation of the Contract


INTENTION TO CREATE LEGAL RELATIONSHIPS:

Family PRESUMPTION: Family members do not intend their bargains to be legally


Relationships binding contracts (Balfour v Balfour). Extended family included – Jones v

4
Padavatton, Parker v Clark. Closer the relationship, the stronger
Test to REBUT 1) There is a presumption that personal relationships don’t intend to make
(Meritt v Meritt) bargain with legal consequences
2) ONUS is on the party PROVING the contract to rebut the presumption
(Salmon L – Jones)
3) The party shows evidence of extenuating circumstances that, despite the
nature of the relationship, indicate intention to contract: Examples of
evidence:
• The contract is written (Merritt v Merritt; Jones v Padavattan)
• The contract is not made in amity (Merritt v Merritt)
• The Plaintiff performed the contract (Merritt v Merritt)
• Lawyer is introduced into the proceedings (Jones v Padavattan)
• Certainty of terms (Gould v Gould) (Merritt)(Jones v Padavattan –
wooliness reference)
• Severe detrimental Reliance (Parker v Clark)
• Commercial arrangements within family matters (economic
interests) (Roufos v Brewster)
• Existence of informal business relationship (syndicate for profit)
(Simpkins)
• Formality or Nature of the bargain – does it look like a contract
(Merritt) (Simpkins)
4) Objective Test: Has the plaintiff led sufficient evidence that a reasonable
person would assume that there was intention to make a contract?
• If yes Presumption rebutted. Intention to contract made out.
COMMERCIAL 1) The presumption of law is that commercial businesses intend their bargains
RELATIONSHIPS to be legally binding contracts (Carlill v Carbolic Smoke Ball)
Test to REBUT
1) The onus is on the party DISPROVING the contract to rebut the
presumption of law
2) It is a rebuttable presumption
3) This party leads evidence of extenuating circumstances that, despite
the nature of the relationship, indicate NO intention to contract:
• Examples of evidence to rebut the presumption (hard to rebut):
Exaggeration (Carlill)
Intention (Carlill)
Mere Puff (Carlill), deposits (Carlill)
Rose and Frank Clause nullifying the contract – “binding in
honor only” (Rose and Frank Company; Vernon Pools)
Policy Considerations (fraud, courts flooded) (Jones v Vernon)
Certainty of terms (Carlill)
• Examples for presumption:
Sincerity of promise (Carlill)
Contract language
Certainty of terms & less ambiguity (Carlill)
Formality or Nature of Bargain – does it look like a contract
(Carlill)
4) Objective Test: Has the plaintiff led sufficient evidence that a
reasonable person would assume that there was NO intention to make
a contract?
• If Yes Presumption rebutted. No intention to contract.

5
Balfour v Balfour [1919] EN
Facts: Husband promised to pay wife 30 pounds per week for living expenses while away, figures
written out and orally agreed. Differences arose between them and agreed to live apart. She filed for
restitution and filed for alimony on December 16th, 1918
Issues: Is an agreement made in amity a legally binding contract?
Holding ATKIN: Appeal should be allowed; The agreement between a husband and wife is not a
contract.
Reasons: Husband and wife agreements – the parties did not intend that they should be attended by
legal consequences.
RATIO: Arrangements between husband and wife are generally not contracts because the parties do
not intend to be legally bound by the agreements.
TEST: What would a reasonable person see from these circumstances?

Merrit v Merrit [1970] CA USA


Facts: Husband moved in with other woman, agreed to pay 40 pounds per week to pay off
outstanding mortgage, signed documents agreeing when all charges paid on house, he will transfer
property to her sole ownership. She completed payments, husband refused to transfer house to wife.
STAMP J, in favor of wife, husband appealed.
Issues: Is agreement on paper created between husband and wife a binding contract?
Holding DENNING: Appeal should be dismissed. Court compelled specific performance for wife
having sole ownership of the house.
Reasons: Parties were not living in amity but were separated at time of agreement. Husband signed
deal with beneficiary ownership of the house.
RATIO: Agreement between husband and wife out of amity with contractual agreement can rebut the
Balfour Presumption to have no legal relationship. Employ Reasonable Person Test.

FAMILY LAW ACT, RSO 1990, c F. 3


55(1) – A domestic contract [such as a marriage contract, separation agreement or cohabitation
agreement] and an agreement to amend or rescind a domestic contract are unenforceable unless
made in writing, signed by the parties and witnessed.

Jones v Padvattan [1969] CA EN


Facts: Mother/daughter case. Daughter has stable job in Washington, mother convinces her to move
to England and qualify for the bar. Mother agreed to provide daughter with a house and living
allowance. Later on house is bought in mother’s name and she replaces Daughter’s allowance by
letting her rent out rooms and collect rent. 6 years later, daughter hasn’t qualified for bar, daughter
won’t let mother into house. Mother sues for breach of contract.
Issues: Was the agreement a legally binding contract and does the Plaintiff have the right to evict the
Defendant?
Holding DANCKWERTS (FENTON ATKINSON): contract not legally binding because it was made
between family members in amity and daughter (D) failed to show evidence that either party had
intent to enter into a legally binding contract. Uncertainty in terms showed no contract.
SALMON LJ applies the Objective Reasonable Person Test: what the parties said and wrote if were an
ordinary man and woman in such circumstances, would it be intended to form a legally binding
agreement. Found daughter was fully detrimentally reliant on mother but that reasonable time for
studies had elapsed. All judges agreed that P had right to evict D, either because there was no
contract or the terms of the contract had expired.
RATIO: If there is wooliness in a bargain between family members, it supports the presumption that

6
there was no intention for a contract.

Parker v Clark [1960]


Facts: Husband and wife (niece) (P) and uncle and aunt (D) made a deal to have niece sell house and
move in with D, when D dies, they will get the property. Deal documented in letters. Within a year,
realize cannot live together, D throws P out of house, niece and nephew sue.
Issue: Is there a case?
Holding: Court says there was a contract because of the severe detrimental reliance.
Reasons: Apply the four step Objective Reasonable Person Test: 1) Presumption (strengthened by
contract being made in amity) 2) Onus is on niece to prove the contract, 3) Niece selling house shows
Detrimental Reliance, was in writing, P fulfilled their end of the bargain (not very strong argument)
4) Would a reasonable person see a contract in these circumstances? Yes.
RATIO: When there is severe detrimental reliance, a contract can be formed between family
members.

Simpkins v Pays [1955] UK


Facts: Grandma and daughter in house with boarder (P), were all close. Three of them do contest
regularly; send in the stamps in Grandma’s name. Grandma won the big prize, said she was keeping it
alone.
Issue: Is there a contract?
Holding: The household was acting as a joint venture, “informal syndicate for profit” (an informal
grouping of people for profit – Yes, there was a contract.
Reasons: 1) In amity when contract was made, presumption of no contract 2) Onus is on Plaintiff to
prove the contract 3) No writing, no formalities, looks like an informal bargain, they split the costs,
Border delivered stamps to mail 4) Objective Reasonable Test

Roufos v Brewster
Facts: P (Brewster) owns a motel, Roufos is Brewster’s son in law. Brewster’s truck breaks down,
they make plan for D put truck on platform drive to town for parts, load liquor onto truck and hire
someone to drive it back. On way back, D gets in accident. P’s insurer covers father in law but sues D
for negligence for allowing someone else to drive car.
Issue: Presumption family members don’t make contracts, insurer - claiming breach of contract.
Holding: There was a contract.
Reasons: In spite of fact they are family – their bargain was a commercial deal between family
members. Both had business interests in the arrangements, which takes it out of family realm.
RATIO: Arrangements made with commercial/business interests between family members can form
a contract and beat the presumption.

Commercial Relationships

Carlill v Carbolic Smoke Ball Company [1893]


Facts: Carbolic Smoke Ball Group (D) placed add in newspaper stating if person used their ball three
times a day for two weeks and contracted influenza, they could come and claim 100 pounds. Stated
placed 1000 pounds in Alliance Bank to show how serious. P bought ball and used as instructed and
contracted influenza. HAWKINS J found she was entitled to recover. D appeals stating certain ads are
mere puffs.
Issue: Is the advertisement placed in the paper a legally binding contract?
Holding LINDLEY LJ: Contract. Money put in the bank demonstrates sincerity and the D’s
performance of the conditions indicates an acceptance of the offer.
Reasons: The ad contained express promise from the D that they had set aside money for the
occasion; reinforces promise/sincerity. A Reasonable Person would assume that the promise was a

7
legally binding contract. Offer is a continuing offer, the nature of the transaction shows that he does
not expect or require notice of the acceptance of the offer. BOWEN: Performance of the condition is
sufficient acceptance without notification.
RATIO: An ad can constitute a unilateral contract, which can be accepted by fulfilling the conditions
of the contract; no formal acceptance required.

Rose and Frank Company


Facts: Rose and Frank (P) and JR Crompton (D) entered into a contract in 1905 and worked well
together. In 1913, created lengthy document and signed agreement to do dealings with one another –
specified “not to be subjected to legal jurisdiction” and “not a formal legal document” and that it was
to be carried through by each “parties with mutual loyalty and friendly co-operation”. D became
dissatisfied with manner of business of P and refused to continue arrangements. P sued and
judgment sided in P’s favor
Issue: Can a business agreement between businesses not be binding?
Holding SCRUTTON LJ: This is not a binding contract despite that these are commercial parties.
Reasons: While in business, binding contracts are implied – it is possible for the parties not to intend
to include legal relations. If an intention is clearly expressed, there is no reason in public policy why
that effect should not be given to that party.
Ratio: The inclusion of a ROSE AND FRANK CLAUSE can nullify a CONTRACT.
If parties expressly state in an agreement that they do not wish to be bound, the courts must respect
their actual intentions.

Jones v Vernon Pools


Facts: Jones bought Vernon Pools coupon, filled coupon with games and got every game right. Sent
coupon to Vernon Pools, they say never received. On back of coupon states: “Acceptance of the
coupon does not create a legal obligation and is binding only in honor.
Holding: Vernon Pools can rely on clause. Exclusion clause is part of the contract when a significant
effort is made at the formation of the contract.
RATIO: Contract stated to be bound by honor only, not legally binding contract.

Offers
Offer Mechanics
General Rules of Offer: Would a reasonable person interpret the negotiations as having reached a
point where one party merely needs to accept to create a contract?
An offer exists when a reasonable person would see an act of correspondence as an offer. It
is not what the person who made the offer intends, but what we objectively see from what
the person did.
If the reasonable person does not see an INTENTION (previous step) then it is not an offer.

Mechanics of the A. Invitation to Treat (reasonable person does not see a contract has
offer: formed at this stage)
1. Is there an invitation An invitation to enter into negotiations, which both the client and the
to treat or an offer? individual who made the quotation would have to accept in order to create
a contractual relationship.
A display (Pharmaceutical v Boots)
o Offer made when goods presented for purchase
o Offer accepted when cashier accepts money
o Can be collateral – you break it, you buy it
A price quote or price list (Boyer v Duke, Harvey v Facey)
Price quotes in communication between buyer and seller is generally
an invitation to treat (Harvey v Facey)

8
o Exception: A price list is an offer when it includes an order form
(Quebec v T Eaton). Look for: “supply limited” to indicate an offer.
Disclaimer not an offer

B. Offer
In determining if an offer has been made, consider:
Reasonable Person Test: Would a reasonable person think that an
offer had been made? (i.e. How would a reasonable person interpret
the words of communication?) (Harvey v Facey)
An offer must be explicit, clear and reasonably interpreted (Harvey,
Johnston Bros)
Answer to inquiry of price is not an offer to sell (Harvey v Facey)
Volatility of market – more volatile, more likely it is to be invitation to
treat. Reasonable person would not think it’s an offer in such a market
(Johnson Bros)
o ‘Please reply immediately’ – likely not an offer (volatile)
What is the product? (I.e. land or something perishable?)
Offer to world at large is valid (Carlill)
Certainty of terms: the more complex and significant the contract,
more likely it is a contract ‘ready to ship’ – offer (Johnston Brothers v
Rogers Brothers)
Since the good is land, a contract is less likely to be made in a few
sentences of a telegram (Harvey)
2. What type of offer 1. Offer of UNILATERAL contract – promise for a performance.
exists? Acceptance occurs at the same time as the performance. Contract exists
once performance occurs. (E.g. Carlill, Shuey). Acceptance does not need to
be communicated in advance.
2. Offer of BILATERAL contract – Promise for a promise. (E.g. Johnston
Brothers) Acceptance must be communicated.
3. Collateral Contract – A contract that comes into existence as a result of
a main contract. Can be unilateral or bilateral (E.g. The Santanita) Tend to
be implicit.
Can exist between a store and customer – you break it, you buy it
(Pharmaceutical Society of Great Britan)
Can exist between owner and tenderers when both parties intend to
initiate contractual relations by submission of the bid (MJB Enterprises)
Privilege Clauses can allow an owner to award compliant bids that are
not necessarily the lowest price, or to not award any tenderer at all (MJB
Enterprises)
Duration of the offer: Four Ways an offer can be terminated:
3. Was the offer alive at 1. Revocation: Offeror can revoke before offeree accepts
the time the offeree • General Rule: Revocations must be communicated to offeree.
tried to accept? However, it’s sufficient if the offeree gets the information that would
lead a reasonable person to believe that the offer no longer stood
(through knowledge of acts). (Dickinson v Dodds)
• Reasonable Person Test: Would a reasonable person be persuaded,
based on the quality of information that the offer has terminated?
(Dickinson v Dodds)
• Offer can be revoked at any time before it is accepted, even if a
provision says that the offer will be open for a certain amount of time.
There is no contract/bargain that binds the offeror to that promise
(Dickinson v Dodds)
o Exception: option contract – consideration for keeping offer open
until stipulated lapse date (e.g. Deposit) (Petterson v Pattberg)

9
• It is good enough to REVOKE an OFFER to the world-at-large in the
same publication that the OFFER was made, even if offeror doesn’t see
it (Shuey)
REVOCATIONS OF OFFERS OF UNILATERAL CONTRACT:
• An implied term of an offer of unilateral contract is that you cannot
revoke once performance has commenced (Errington). A COLLATERAL
CONTRACT exists to protect offerees in the course of performing the
condition of a UNILATERAL CONTRACT (Errington).

2. Rejection: A rejection terminates the offer. Offeror is free to make


offer elsewhere. (Livingston v Evans)

3. Counter Offer: An implied rejection of original offer – terminates


original offer. Would reasonable person believe it is a counter-
offer?
• A COUNTER OFFER is a REJECTION. The offeror can reinstate offer
after a counter-offer (Livingstone v Evans).
o Rejection of counter does not automatically renew original offer
• Counter offer becomes effective once it is received by the offeree
(Henthorn v Fraser)

4. Time Lapse: An offer can terminate bc it is too old. There may be


a specified lapse date or implied lapse date (e.g. based on rumor,
news, etc.). If no date specified, court must decide what a
reasonable lapse date would be. (Barrick v Clark)
Test for Determining implied lapses (i.e. reasonable lapse time):
Theory 1: (Barrick v Clark). Offer lapses after reasonable period based on:
1. Nature of Contract (e.g. land, perishable food)
2. Circumstances of the offer (e.g. possession taken months away,
can’t use bc of winter)
3. Conduct of the parties (e.g. Does communication indicate
urgency?)
4. Course of business – How normal course of business runs between
parties, this transaction?
• Offeror is seen as REVOKING the offer after the implied lapse date.
Looks at what existed at the time of the offer. (this is theory
applied today)
Theory 2: (Manchester DC v Commercial General Investments)
• If offeree doesn’t accept w/i reasonable time, seen as REJECTING
the offer after implied lapse. Includes an assessment of facts after
offer took place.

TIME LAPSE of OFFERS OF UNILATERAL CONTRACT


• If offeree commences the performance on an offer of unilateral
contract, offeror agrees not to interfere with their capacity to
perform the contract as long as they perform within a reasonable
period of time. (Loring v City of Boston)

Pharmaceutical Society v Boots [1953] UK


Facts: Pharmaceutical Society (P) says that you are not allowed to sell drugs with any poison from
the Poison List without supervision of a registered pharmacist. P claimed that Boots (D) allowed cus
to buy drug with poison in it without supervision. P claims offer is the display of the item and

10
acceptance is picking up item and putting in basket. D claims that display is an invitation to offer,
offer is made when person goes up to till, acceptance when cashier says its yours and you take
money.
Issue: When is a contract/transaction complete?
Holding SOMMERVILLE LJ: Appeal dismissed, agreed that transaction was in fact supervised.
Reasons: A true transaction allows customers to put item in receptacle, have free access to the shop
and then present item desired to assistant at cash, when money is exchanged that is the offer being
accepted.
RATIO: A display is an INVITATION TO TREAT, giving the item to the cashier is an OFFER, the cashier
accepting your money for the item is an ACCEPTANCE

Boyer and Co. v Duke [1905]


Facts: Manufacturer offered a price for his work, may have received and complied with many
applications for quotations in the same day
Issue: At what point is the offer of a price considered an agreement to perform the action/sale for
that price and timeline?
Holding: Price quote is not an offer, it is an invitation to make the offer. Offer is when you made the
order. Acceptance is when the seller says, “Yes, I do have this available for you”
Reasons: MADDEN J: There is a difference between the quotation submitted as the basis for a
possible order and an offer to sell which if accepted creates a contract for breach of which damages
can be recovered.
RATIO: Price quote/list is not an offer – it is an invitation treat.

Quebec Pharmacy v T Eaton


Facts: T Eaton (in Ontario) has a catalogue containing products. It is illegal to sell products in Quebec
without supervision. Pharma says T Eaton is in breach of contract. T Eaton says that a price list sent
out in Quebec is just an invitation to treat – so is a catalogue. Offer is made when order is accepted
when T Eaton fulfills the order.
Issue: Is Eaton’s breaching the law in Quebec?
Holding: Court says that the contract was formed in Ontario. It was accepted when the company
agreed. Catalogue with a pricelist is just an invitation to treat.
RATIO: A catalogue is generally an INVITATION TO TREAT and not an offer but the inclusion of an
order form may indicate that the catalogue is an offer.

Harvey v Facey [1893] CA


Facts: Harvey (P) telegrammed Facey (D) while they were on a train requesting (1) if would sell
Bumper Hall to them (2) What the lowest price was. Facey replied only with the lowest price of 900
pounds. Harvey replied that they agree to purchase for that price.
Issue: Was an offer made to Harvey? Can Harvey treat Facey stating the lowest price as an
unconditional offer to sell to them at the price named?
Reasons: Facey never explicitly answered the question of will sell to Harvey. Only stated the price
they were willing to sell at. Facey is not bound in any respect – he never made an offer.
RATIO: The statement of the lowest price for which a vendor will sell contains no implied contract to
sell at that price to the person making the inquiry.

Johnston Bros v Rogers Bros [1899] ON CA


Facts: P claiming a breach of contract for the sale and delivery of a quantity of flour, P received letter
from D stating “We quote you…” and “We suggest using a wire to order, as prices are changing so
rapidly that they might be beyond reach.” P replied the next day saying we accept at the price and

11
want this much. D replied that the prices have changed and we will give you that much at this higher
price.
Issue: Was there a contract based on the first letter sent from D?
Holding FALCONBRIDGE: No, there was no contract
Reasons: A quotation is not an offer to sell – to quote = to give current market price. Line referring to
rapidly changing prices would not be in a simple offer to sell at a price. A quotation is an invitation to
treat, telegram from P about how much is an offer – no acceptance, instead counter-offered a new
price.
RATIO: Quotation of current prices isn’t an offer to sell.

Carlill v Carbolic Smoke Ball Company [1893]


Facts: Carbolic Smoke Ball Group (D) placed add in newspaper stating if person used their ball three
times a day for two weeks and contracted influenza, they could come and claim 100 pounds. Stated
placed 1000 pounds in Alliance Bank to show how serious. P bought ball and used as instructed and
contracted influenza. HAWKINS J found she was entitled to recover. D appeals stating certain ads are
mere puffs.
Issue: When accepting the contract is performing the condition sufficient acceptance of the offer? Is
notification required?
Holding BOWEN LJ & LINDLEY LJ: Agree that appeal should be dismissed and that P should receive
compensation.
RATIO: An offer can be made to the world at large – if it is an offer of unilateral contract (Unilateral –
when contract is formed, one side has already done the side of the bargain and the other side is
bound to perform)

The Santanita [1895] EN


Facts: Santanita entered into a yacht club and signed document that will obey all rules adopted by
club (one of them being “If a yacht shall foul another yacht – she shall forfeit all claim to prize and
shall pay the damages.” Santanita crashed and sank Valkyrie while getting into position for race.
Yacht club doesn’t sue – they have no damages. P (Valkyrie) stated they entered into this race under
assumption they would perform under these rules, D denied entering into this agreement.
Issue: Is there a contract between two yacht owners since they both signed an agreement with the
yacht club stating that they would pay any damages? Where is the offer and acceptance of the
contract?
Holding: Yes, there was a contract and the sensible thing is that he will pay damages directly to the
yacht fouled
Reasons: The main contracts exist between the defendant and the Yacht club and the Plaintiff and
the Yacht club. The conditions of the main contract state that a unilateral collateral contract exists
between the plaintiff and defendant (and all members of race) whereby you make an offer to
everyone in the race that if you break the rules, you will pay damages.
RATIO: A COLLATERAL CONTRACT can be implied by a MAIN CONTRACT. MUTUAL TERMS FOR
MUTUAL BENEFIT (the parties must have all signed the same MAIN CONTRACT.

MJB Enterprises v Defense Construction [1999]


Facts: Request for proposals (RFP) by Defense Construction (D). P tenders proposal for contract. D
chooses another proposal, other proposal was misrepresented and more expensive than P’s. P argues
that D breached their own tender process. P sues because D did not take their proposal (the lowest
one). D claims a ‘Privilege clause’ – “reserve right to final decision” – “the lowest or any tender shall
not necessarily be accepted”
- Contract B: contract in Request for Proposals, what the bidder agrees to for the proponent

12
- Contract A: in consideration to the bidder there is a collateral contract created (to have D
comply with terms of tender, statements in RFP)
- Only person who was unfairly not selected and lost profits will have a Collateral Contract
Issue: Was there a breach of Contract A for the P? What are the damages?
Holding ETSEY J: Court held that there was a breach of Contract A from the respondents (D) and that
the damages should be awarded based on expected losses – which would amount to the amount of
Contract B.
Reasons: Found that Contract A in this case did not contain a term that stated the tender with the
lowest compliant would receive the work. D argues over the Privilege clause gave ability to contract
anyone – including a non-compliant bid – IACOBUCCI does not find this clause overrode obligation to
accept only compliant bids. By accepting a non-compliant bid – D di breach their contract A.
RATIO: In tender process – two contracts are created. Contract A is the proponent promising the
bidder that he will select a compliant bid. Contract B is the contract between the bidder selected and
the Proponent.

Duration of the Offer

Dickinson v Dodds [1876]


Facts: Dodds (D and appellant) wrote a letter offering to sell property to Dickinson (P) for a price
and offered an end time for response for Friday June 12 th at 9 AM. Dickinson knew he wanted to
purchase on the 11th but did not signify acceptance. Heard later in day Dodds made another offer,
decided in evening to drop a written letter of acceptance to D’s mother in law and have agent stop D
in the train station 7 AM Friday. D replied that property was sold and it was too late. P sued and trial
judge decided specific performance for P.
Issue: Was the letter just an offer, or a fully binding contract? Does revocation of an offer needs to be
communicated to the offeree?
Holding JAMES LJ: There was no binding contract between D and P.
Reasons: The promise in the letter was not binding until a complete acceptance by Dickinson of the
offer. Dodds was free as Dickinson until that point. He left mother in law with an acceptance of the
offer knowing all the while that Dodd’s had entirely changed his mind. An accepted contract doesn’t
exist until the existence of the same mind between two parties.
RATIO: 1) OFFERS can be revoked up until the offer is accepted 2) Revocation needs to occur by
communication, but it may be sufficient that the offeree receives information that a reasonable
person would assume that the offer is no longer on the table. 3) A stipulated lapse date is a bare
promise and not enforceable by law, unless the offeree forms an option contract and offers some
consideration.

Petterson v Pattberg Bad Law [1928]


Facts: D (owner of a bond secured by third mortgage and P property) sent a letter to P (property
owner) stating that if P paid the regular payment on April 4 th, 1924 and then paid in full by May 24th –
five years before the due date – he would reduce the mortgage payment by $780. P made regular
payment and then showed up to pay in May. D refused to take money. D had sold the mortgage bond
to a third person (revoked the offer because he doesn’t own the offer anymore). Trial judge for
plaintiff.
Issue: Did the D revoke his offer prior to it being accepted?
Holding KELLOGG: Decision of Trial Term should be reversed and complaint dismissed. D did
revoke his offer prior to it being accepted.
Reasons: A unilateral contract may be revoked at any time prior to the performance of the requested
action. If the offeror can say ‘I revoke’ before the offeree accepts, however brief the interval of time
between the two acts, there is no escape from the conclusion that the offer is terminated.
LEHMAN (dissent): D made performance of acceptance of offer impossible by refusing to accept

13
payment.
RATIO: A unilateral contract may be revoked at any time prior to the performance of the requested
action.

Errington v Errington [1952] UK CA


Facts: 1930, father bought house for son and daughter in law, they were expected to pay mortgage
and he would transfer property to them when he retired. Wood paid mortgage installments regularly
but found too burdensome, father helped out. Father died, leaving estate to wife (Errington), Wood
split from son and Errington sued for possession, son moves to mom, daughter in law stays in house
and keeps paying mortgage – mother brings a motion to get daughter in law out of house.
Issues: Can a unilateral contract be revoked after the death of the offeror?
Holding DENNING: Appeal dismissed.
Reasons: No express promise between the two parties – couple never promised to pay mortgage
installments. Instead, it was a unilateral contract – the performative act of paying for the mortgage –
meaning it would only be revocable if couple did not make payments. Once performance has started
the offeror cannot revoke the offer.
RATIO: there is an implied term of the contract that once the performance has begun, it cannot be
revoked.
- A unilateral contract cannot be revoked once other party entered on performance of the act,
but the contract would cease to be binding if the party left it incomplete and unperformed.
- Collateral contract operating – in consideration of your commencing the performance, I
agree not to revoke the contract and make the performance impossible.

Shuey v USA [1875] US SC


Facts: Following President Lincoln’s assassination, the Secretary of War issued and published a
$25,000 reward for the apprehension of John H. Surratt, one of Booth’s accomplices.” Plaintiff (St
Marie) aided in capture of Surratt, he saw the offer but didn’t see the revocation. The reward had
been withdrawn (several months prior to Surratt’s capture) in the newspaper. The US paid $10,000
for P’s assistance in capturing and identifying Surratt. P died and his executor sued for the rest of the
money.
Issue: Is D liable to pay the remainder of the award?
Holding STRONG J: Judgment affirmed, they are not liable – he did not actually apprehend Surratt as
required by the reward offer
Reasons: Until the terms of the contract are fulfilled, the contract does not exist. Because the plaintiff
complied with the contract seven months after it was revoked, the defendant and the plaintiff never
entered into a contractual relationship. The judge holds that because the revocation was published in
the same publication in which the offer appeared the revocation is official. P ought to have known
that the offer could have been revoked in the manner it was made.
RATIO: It is good enough to REVOKE an OFFER to the world-at-large in the same publication that it
was made.

Livingstone v Evans [1925]


Facts: 1) Evans (D) wired offer to sell land for $1800/close price (OFFER). 2) Livingstone (P) says
“send lowest cash price, will give $1600.” (REJECTION & COUNTEROFFER). 3) D: “Cannot reduce
price” (REJECTION of COUNTEROFFER & reinstatement of original offer 4) P accepts $1800. D sold
the property to someone else.
Issue: Was there a contract? Does the Defendant’s last response constitute a rejection of the initial
offer?
Holding WALSH: Finding for P, specific performance granted.

14
Reasons: Hyde v Wrench: firmly established that as a part of law of contracts that the making of a
counter offer is a rejection of the original offer. But the telegram from Evans stating “cannot reduce
price” – was a renewal of the original offer – giving the P the right to bind the D to it by subsequent
acceptance of it.
RATIO: Offer is killed by a counter offer – it is determined by a reasonable person test.

Barrick v Clark [1951] (Lapse)


Facts: Barrick owned farmland that Clark wanted to buy, Sept 11 th. Oct. 10th : Barrick – sell for cash.
What is the land worth to you (invitation to treat); Oct 30th: Clark – Will pay $14,500. Send me a
wire. How long will the decision take? (offer); November 15th: Barrick – Will sell for $15,000. Tell me
ASAP. Will give you title by January 1. Trust I will hear from you asap. (counter offer); November
20th: Clark’s wife receives letter. She wrote back saying “Please hold offer open. Clark is on hunting
trip for another 10 days.” No response from Barrick; December 10 th: Clark (letter) – I accept offer of
$15,000; December 11: Clark finds out that Barrick sold the land to Hallman on Dec 3 (revocation of
offer). P sues for performance of contract.
Issue: Did the Plaintiff accept the defendant’s offer within a reasonable time?
Holding ESTEY/KELLOCK: No contract. The plaintiff did not accept the offer in a reasonable time
and offer lapsed.
Reasons: D was under no obligation to extend the time limit on the offer, even though the plaintiff’s
wife asked for a hold of ten days. TEST for REASONABLE LAPSE DATE: 1. Nature of Contract – Land is
not subject to fluctuations in price, therefore longer lapse period is reasonable. 2. Circumstances of
the offer: Offer being made during the winter in Saskatchewan, Tenant living on land until March 1st,
Little can be done with the land on an immediate basis –implies longer lapse date. But significant
demand for land encourages shorter lapse time. 3. Conduct of the parties: Correspondence between
two parties suggests urgency “let me know as soon as possible” which implies immediate response. If
acceptance had occurred on Dec 10th it would have been impossible to transfer deed by Jan 1 st as
discussed in Barrick’s offer.
RATIO: Theory 1 of Implied Lapse Date: An offer lapses after a reasonable period of time based on 1)
the nature of the contract (e.g. an offer of a perishable item might lapse before an offer for a non-
perishable item) 2) Circumstance of the offer (e.g. known demand for an item), and 3) the conduct of
the parties. The offeror is seen as REVOKING the offer after the implied. Only consider what you
know at the time of the offer.

Machester DC v Commercial & General Investments


Issues: For the purposes of determining an implicit time limit on an offer, should the offeror be seen
as withdrawing the offer after a reasonable period of time has elapsed, OR should the offeree be seen
as rejecting such an offer?
Decision: The offeree should be seen as rejecting the offer because the alternative involves too many
uncertainties. This theory allows the inclusion of all of the evidence provided by the offeree up until
the point of the case.
Reasons: BUCKLEY – Two different theories of lapse when there is no specified lapse date:
1. Traditional Theory (Barrick v Clark): A lapse date is an implied term of the offer. If an
offer is not accepted within a reasonable time, it must be treated as
withdrawn/revoked. Looks at things that existed at the time of the offer.
2. Theory Two: If the offeree does not accept the offer within a reasonable time, he mut be
treated as having refused it. Involves an objective assessment of facts and the determination
of whether on the facts the offeree should in fairness to both parties be regarded as having
refused the offer.
RATIO: Theory 2 of Implied Lapse Date: An offeree is seen as REJECTING an offer after the implied
lapse date, looks at circumstance throughout the time period. Courts consider theory from Barrick v
Clark applies (i.e. the offeror is seen as revoking the offer after a reasonable amount of time).

15
Loring v City of Boston
Facts: Boston sends offer on May 1837, in the paper offering award for finding the arsonist setting
fire to buildings of Boston. In January 1841, Loring gives info about the arsonist claims that he
deserves the award. Boston says offer has lapsed.
Issue: Has an unreasonable amount of time passed to allow the offer to lapse, even though this is a
unilateral contract?
Holding: Court said offer had lapsed.
Reasons: Even though the performance may have been started already, an unreasonable amount of
time has passed – therefore the offer should be lapsed.
RATIO: If offeree commences the performance of an offer of unilateral contract, offeror agrees not to
interfere with their capacity to perform the contract as long as they perform within a reasonable
period of time.

Acceptance

ACCEPTANCE: GENERAL RULES: (Felthouse v Bindley)


• Acceptance must be communicated/received in order to be
4.Was the offer effective (i.e. good when received)
accepted? • Generally, silence is not an effective acceptance
o Exception: course of conduct can indicate acceptance (Cole
McIntyre)
• Offeror can’t impose upon an offeree to act/respond or otherwise be
bound by contract
Exceptions to Felthouse v Bindley
• Course of conduct: may amount to acceptance (you can’t sit on an offer
and then by course of conduct, indicate acceptance and then state that
you don’t accept the offer) (Cole McIntyre)
• Custom known to both parties: if the local custom is such that
acceptances are not communicated, court may defer to this evidence
(Cole v McIntyre)
• Past conduct between the parties: if the parties have conducted
business in such a way without notice of a contract (Coly McIntyre)
• Wording of the Offer: is the contract worded in such a way that the
offeror does not require notice of acceptance? (Felthouse) The wording
of the offer is supreme over other factors.
• Postal Acceptance Rule: A contract formed through post is complete as
soon as the letter accepting the offer is put in the mail (Household
Insurance v Grant). This rule applies when there is an intermediary
involved (mail, courier, telegram, messenger) and communication is
non-instantaneous (Entores)

When do you have the right to accept by mail and apply the postal
acceptance rule? (Henthorn v Fraser)
• When ordinary usage dictates that it is permissible. (i.e. regularly use
the mail)
• When acceptance by post is stipulated (Household Insurance)
• When you can show that use of the mail is no less disadvantageous for
the offeror (Manchester DC)
• When the offer is made by mail (Household)
• When the parties used that means of acceptance before
• Reasonable to assume that the ordinary man would use post as the
means to communicate (Henthorn v Fraser)

16
LIMITATIONS ON POSTAL ACCEPTANCE RULE
• Fault Analysis – Exception to PAR – If non-communication of
acceptance is due to the fault of the offeree and they are aware of it, PAR
does not apply (e.g. burning mailbox) (Household Insurance).
• The PAR can be overridden even if the parties involved consider the
post to be an acceptable means of communication, where:
• The express terms of the contract contradict PAR – state they don’t
want a letter (Holwell Securities v Hughes)
• The PAR would result in manifest inconvenience and absurdity and
If PAR does NOT apply, general rule from Felthouse v Bindley applies and
acceptance is good when communicated/received.
Was the 1. No Does it involve an intermediary (e.g. courier, post office?) – if
communication YES, Postal Acceptance Rule Applies (i.e. acceptance is good when
instantaneous? sent). Subject to PAR fault analysis and exceptions.
2. Yes Is it an electronic document (e.g. email)?
• Yes Electronic Commerce Act applies:
o Electronic information or an electronic document is presumed to be
received by the addresses when:
A) SENT, if the recipient indicated the precise mode of
communication (regularly used method)
B) RECEIVED (i.e. when they become aware of info/doc), if
the recipient did not indicate the precise mode of
communication.
o “Presumed to be received” is a rebuttable presumption. The onus is
on the offeror to prove that acceptance was not received and that it
wasn’t his fault.
o Is the statute rebutted?
Yes Common Law Applies: Entores v Miles –
instantaneous communication is only good when received.
Onus is on offeror to prove it was not received.
No Statute applies and determine acceptance based on
statute
• No (e.g.) Fax Common Law applies (Entores v Miles) – Acceptance is
only good when it is communicated/received. If fault of offeror, good as
when sent. If BOTH at fault, it cancels out acceptance is good when
received.
• Contract is made in the location where the offeror receives notification
of the offeree’s acceptance (Eastern Power)
• FAULT ANALYSIS EXCEPTION: If there is reason for the non-
communication of acceptance is the fault of the offeror (e.g. out of fax
paper) and the offeree is unaware (or not notified), then acceptance is
good when sent (Lord Denning, Entores).
• If offeror reasonably believes there is a K and is fault of offeror, there
is a K.
o Offeror makes an offer over the phone and the offeree accepts
but line dead mid-sentence (onus on offeree) – Call back
Contract, Don’t call back No contract
o If on phone, offeree accepts but is mumbled, believes there is K,
onus on offeror to clarify, otherwise there is a K
o If offeror doesn’t receive message and not their fault, but
offeree thinks it went through no contract
• Policy for Entores: already instantaneous so no argument for business
expediency, so good when received

17
TIME AND PLACE PLACE of ACCEPTANCE:
(Stipulations in the • General Rule: ACCEPTANCE must be communicated to the offeror at
offer) the place from which the OFFER is MADE. (Eliason v Henshaw)
o EXCEPTION: When an offeror stipulates a manner/location in
Did the acceptance which it is to be accepted, an acceptance communicated
occur in a elsewhere or by other means is not binding. (Eliason v
manner/location/time Henshaw)
that was different
from what the offeror MANNER OF ACCEPTANCE:
stipulated? • GENERAL RULE: Where the offeror prescribes a method of acceptance,
acceptance by any other method which doesn’t disadvantage the
offeror will be deemed communicated. To rebut this, the offeror would
have to show they were inconvenienced. (Manchester DC)
o Ask: How did you get the offer? Was the manner of the
acceptance stipulated? Would another method of acceptance
disadvantage the offeror?
EXCEPTION: Unless the offeror insisted that acceptance will ONLY be
binding if a SPECIFIC method of acceptance is used (Manchester). This
allows individual to contract out of the postal acceptance rule – as long as
doesn’t disadvantage offeror.
ACCEPTANCE OF General Rule: Acceptances must be communicated (Felthouse)
UNILATERAL General Rule Unilateral Contract: In a unilateral contract, the contract is
CONTRACTS accepted when the offeree performs the conditions of the contract. Notice of
acceptance does NOT have to precede performance of the offer in unilateral
contracts (Carlill)

Acceptance of Unilateral Contracts:


1. Can be communicated by post and PAR would apply (Bishop v
Eaton)
2. Must be communicated within a reasonable amount of time (Bishop
v Eaton), and
3. Requires:
a. Must have knowledge of the offer when performing the
conditions in order for acceptance to be good (Fitch v Snedaker;
R v Clark, AUS)
b. Acting on the offer (i.e. performance) with at least partial
intention/motivation from the reward (R v Clark)
i. Reasons for performance is irrelevant (Williams v
Carwordine)
ii. Character of person performing is irrelevant (Smirnis v
Toronto Sun)

Communication of Acceptance

Felthouse v Bindley [1862] (Ct NS)


Facts: Felthouse (P) negotiated to purchase horse from nephew, was mix up in price (guineas
between pounds). Uncle gave definite offer in Jan to split the difference – stated if heard nothing
more, I will consider the horse mine at offered price. Nephew didn’t respond and no actions made. In
Feb, nephew sold all of his farm stock in an auction and horse, despite nephew’s instructing it be
reserved, was sold. P sued auctioneer Bindley to recover the horse. D appealed.
Issues: Did the nephew accept his uncle’s offer? Or was the horse still his property at the time of the
auction?
Holding WILLES J: Appeal allowed for the D.

18
Reasoning: Nothing had been done at the time of auction to imply the property had changed to the
uncle and nephew had given no formal acceptance. Nephew had intended to sell to his uncle but had
not communicated his intention or done anything to bind himself.
RATIO: An offeree must communicate acceptance to the offeror to make a contract; silence does not
equal an acceptance.
Principles: 1) Acceptance must be communicated to offeror to make a contract; silence from offeree
does not constitute acceptance. Exception: Unless offeror in offer indicates that acceptance need not
be communicated. 2) Offer cannot place on an offeree a duty to respond to or be bound in contract.

Cole McIntyre-Norfleet v Holloway [1919]


Facts: On March 26, 1917, a salesman employed by Cole (D) visited Holloway’s (P) store and
successfully solicited an order for meal (perishable good); agent has a disclaimer on back of offer that
head office needs to accept contract before is formed. D continues to visit store weekly, never
mentioning the previous order. Meanwhile, prices went up 50%. Sixty days after order, P asked for
delivery but D refused claiming head office had never accepted the order and so there was no
contract. P argued that D’s actions communicated acceptance.
Issue: Can silence be considered acceptance? Was contract formed by not informing P of refusal in a
time sensitive manner?
Holding: Yes, there is a contract, normally not but they had ongoing trade relationship and conduct
of Cole indicated acceptance of offer even though he did explicitly say it.
Reasons: Salesman had weekly opportunity to inform buyer of refusal of order. The fact there was
no official refusal, coupled with business-as-usual by both parties is sufficient to provide assent.
RATIO: The basic rule that ACCEPTANCES need to be communicated can have exceptions. Course of
conduct indicates an acceptance of an offer.

Household Insurance Co v Grant [1879] EN CA


Facts: D wanted to buy shares in P’s company - must reply by letter informing that allotment was
accepted. P sent letter through post, it never reached D.
Issue: When is an offer accepted when sent by the post?
Holding: Offer was accepted; time of acceptance is when it was posted in the mail.
Reasons: The post office acts as the agent of both parties (ex: if C is agent of B and A accepts B’s offer
by communication to C, then A and B are in contract). Acceptance is good when delivered to post BUT
acceptance is not good if lack of communication is fault of the offeree.
RATIO: Postal Acceptance Rule: is an EXCEPTION to the general rule of acceptance. A contract in
post is complete as soon as the letter accepting the offer is put in the mail. This rule applies when an
intermediary/third party involved (i.e. mail, courier, telegram, messenger) and communication is
non-instantaneous.

Entores v Miles [1955] EN CA


Facts: Two companies made agreements through Telex machine. Agreement was breached. P sued,
court favored them for not receiving what was agreed on from D.
Issue: When is the contract made in terms of telex and telephone communications?
Holding DENNING: Appeal was dismissed. The contract was not completed.
Reasons: “Where the offeror without any fault on his part does not receive the message of
acceptance – yet the receiver reasonable believes it has got home when it has not – then I think that
there is no contract”
RATIO: General Rule for INSTANTANEOUS COMMUNICATION: Acceptance is only good when
received. EXCEPTION: If the reason for the non-communication is the fault of the offeror (e.g. out of
fax paper) and the offeree is unaware of the non-communication. In that case, good when sent. This is

19
rule for instantaneous communication that doesn’t fall under the Electronic Commerce Act (phone,
fax).

Eastern Power v Azienda [1999] Ont CA


Facts: EP (P) negotiated with ACEA (D) on a joint venture agreement to construct and operate an
electricity generation plant in Italy. D signed a letter of intent and faxed it to EP in Ont. P signed letter
of intent and faxed back to D in Italy. Parties unable to conclude the joint venture, EP brought action
against ACE. Court stated forum non conveniens (another court is better to deal with it); P appealed.
Issue: When is the contract formed through facsimile transmission – does it follow the general rule –
when acceptance is received or the postal exception rule?
Holding MACPHERSON: It is formed in Italy, upon receiving of the acceptance. Follows general rule
of acceptance, not postal acceptance.
Reasons: A form of instantaneous communication – like telephone (Entores), contract is complete
when acceptance is received; offeree could have confirmed within minutes that they had a binding
contract. The acceptance of an offer by the offeree must be notified to the offeror before a contract
can be regarded as concluded.
RATIO: An acceptance sent by instantaneous communication forms a contract when the acceptance
is received.

Electronic Commerce Act SO 2000 c 17


s 19(1): The acceptance of an offer of any other matter is essential to a contract (offer, counter offer,
revocation) may be expressed by (a) electronic means or (b) by an act that is intended to result in
electronic communication, i.e. pushing an icon, clicking
s 22(3): An electronic document is presumed to be received by the addressee when
(a) SENT, if the recipient indicated the precise mode of communication (i.e. authorizes a regularly
used method of communication
(b) RECEIPT (i.e. when they become aware of info/doc), if the recipient did not indicate the precise
mode of communication.
• “Presumed to be received” is a rebuttable presumption. The onus is on the offeror to prove
that it was not received and that it wasn’t his fault.
• Very different from Entores v Miles, which tells us with instantaneous communication, it’s
good when received. This says when instant communication, it’s good when sent if offerer
regularly uses the communication.
• Sets up a set of rules for electronic communication on a presumptive basis. If you can prove
the type of communication is not under the statute or a presumption doesn’t apply then back
to Entores.

Time and Place


Whether you can put stipulations on acceptance

Eliason v Henshaw [1819] USSC


Facts: The plaintiffs offered to purchase from the defendant some flour to be delivered to
Georgetown by a certain time for a particular price. The plaintiffs requested the offer be accepted be
a wagon through which the offer was made. The plaintiffs stipulated a location where the acceptance
was to be delivered. The defendants did not accept the offer by the wagon, nor did they deliver the
offer to the right location. Trial judge for the D.
Issue: Was the offer accepted in the right time, place and manner? Can an offeror stipulate the
manner with which an offer is to be accepted?
Holding WASHINGTON: Appeal accepted. Judgment reversed.

20
Reasons: Acceptance communicated at a place different from that pointed out by the P and forming a
part of their proposal imposed no obligation on binding them. The place to which the answer was to
be sent constituted an essential part of P’s offer. If acceptance is not sent to the requested area, then
it is up to the offeror to decide if they will be bound or not. Stipulating where the offer goes only
holds if the manner of acceptance is not less convenient to the offeror – onus is on offeree to show
that they are inconvenienced.
RATIO: Offeree must follow the terms of acceptance of the offeror (time/place/manner of
acceptance) for an acceptance to be binding.

Manchester DC v Commercial & General Investments [1969]


Reasons BUCKLEY: “I am of the opinion that acceptance is communicated to the offeror by any other
mode which is no less advantageous to him will conclude the contract.” When acceptance is
requested to be returned by post – may reply by telegram or verbal message, or by any means not
later than a letter written and sent by return of post.
RATIO: Two types of means of acceptance:
1) Means of acceptance clearly stipulated in offer as the ONLY means of acceptance. If that is
done in offer, then that’s the only means of acceptance. Must be clear and specific (A way of
overriding the PAR – stipulate ONLY method of acceptance)
2) If offeror doesn’t specify that it’s the ONLY means of acceptance (i.e. it’s a suggestion), other
means of acceptance are fine as long as it doesn’t disadvantage the offeror. To rebut this, the
offeror would need to show prejudice.

Holwell Securities Ltd. v Hughes UK CA [1974]


Facts: Agreement entered into Oct 19th, 1971 – Clause 1: P granted option to purchase certain
freehold property from the D. Clause 2: Option shall be exercisable by notice in writing to D at any
time within six months from the date hereof. P’s solicitor wrote letter to D giving notice to exercise
option – posted, properly addressed, prepaid on April 14th but never reached the D or his address.
Offer expired on April 19th, no further written communications.
Issue: Does the postal acceptance rule make the failure to reach its destination irrelevant and that a
contract still exists?
Holding RUSSEL & LAWTON: Appeal dismissed, no acceptance of contract.
Reasons: The answer might well be that in the circumstance the D had impliedly invited
communication by use of an orifice in his front door designed to receive communications – as long as
he received them. LAWTON: “It does not apply when the express terms of the offer specify that the
acceptance must reach the offeror”
RATIO: The postal exception rule does not apply when notification of acceptance has been specified.
Can preempt the postal exception rule by specifying you want acceptance.
Postal Acceptance Rule can be overridden, it does not apply when:
1) The express terms of offer contradict the postal acceptance rule (Holwell)
2) The postal acceptance rule would result in inconvenience and absurdity

Henthorn v Fraser [1892] CA


Facts: Fraser (D) orally offered to sell property to Henthorn (P) for 750 pounds, with 14 days to
spare. On July 8th, Fraser agreed to sell property to someone else, with option to cancel if he couldn’t
get out of deal with P. D sent a letter of revocation (good when received), which wasn’t received
until 8PM. P posted an acceptance (good when sent) at 4 o’clock. Therefore, postal acceptance rule
applies – contract already formed.
Issue: Was the contract revoked before the acceptance occurred?
Holding HERSCHELL: P successful – order for specific performance made.

21
Reasons: Although P received offer in Liverpool by the D in person, given the specified duration of
days to contemplate and that he resided in a different city, it is ordinary usage of mankind if he
accepted the offer by means of the post. A person making an offer should be considered continuously
making it until he has brought to the knowledge of the person to whom it was made that it was
withdrawn.
RATIO: The postal rule does not apply to revoking an offer.
Postal Acceptance Rule will apply (for accepting offer):
1. Accepted by ordinary usage (it is reasonable to use this type of communication)
2. Acceptance by post is stipulated in contract
3. Offer is given by mail
4. Post is no less advantageous for offeror

Acceptance of Unilateral Offers

Bishop v Eaton [UK]


Facts: Father writes to a friend saying his son Harry is living in the area, if he needs money, give it to
him and will reimburse. Friend tells him that he gave money and sent a letter informing you. Father
says he never got the letter and that he is revoking the offer and the offer had lapsed anyways.
Issue: Is the performance of the conditions sufficient? Did the Postal Acceptance Rule Apply?
Holding: Yes, there was a contract.
Reasons: Court says that the friend did have an obligation to notify the father about his performance.
Here the Postal Acceptance Rule is good (notice is good from time sent), so the friend did notify.
RATIO: If an offeror under unilateral contract is not going to get notice of performance through
performance itself then the offeree has an obligation to notify the offeror within a reasonable time.

Carlill v Carbolic Smoke Ball Co.


RATIO: General rule is that acceptances must be communicated. In a unilateral contract, the contract
is accepted when the offeree performs the conditions of the contract. The performance is acceptance.
The notice of acceptance does not have to precede performance of the offer in a unilateral contract.
Notice of acceptance is contemporaneously given with the performance.

Motivation:

R v Clark [1927] Aus HC


Facts: Crown offered award for info on murder of two police officers. Clark (P) gave evidence but to
clear his own name from the murder charge. Is demanding reward, Crown refuses because there was
no contract between them. Court says you need to be motivated by offer to perform and Clark didn’t
know about the reward at time he performed.
Issue: Was there a unilateral contract? If so, when was it accepted?
Holding ISSACS ACJ: Appeal allowed. Returned to trial decision, no contract. Clark gets no reward.
Reasons: He was acting with reference to a specific criminal charge against himself and not with
reference to a specific general request by the community. There was neither an “acceptance nor
performance” and therefore no contract. Acceptance of a unilateral contract requires the person
accepting and performing the act to be acting on that offer.
RATIO: One cannot accept an offer that one doesn’t know exists or that one has forgotten exists. One
needs an expectation or reliance interest in the reward in order for that reward to be recoverable.

Fitch v Snedaker
Facts: A police officer captures a felon. After having arrested the individual, the officer finds out there
is an offer or reward for the felon. Police officer sues for reward.

22
Issue: Can an individual perform the conditions of a unilateral contract without knowing about the
contract?
Decision: No contract.
Reasons: An individual cannot perform the conditions of a unilateral contract without knowing
about the existence of that contract. It doesn’t matter if you’ve performed a unilateral contract, in
circumstances where you were unaware of the offer.
RATIO: An individual must have the contract in mind (knowledge of contract) when performing
conditions for acceptance to be good.

Williams v Carwordine
Facts: A reward is out for hunt of murderer. Woman is beaten almost to death by murderer.
Individual decides to give information to police about murderer. She knew about reward but was
motivated primarily by fact that she thought she was going to die and not by the reward.
Decision/Reasons: Contract. At the very least, the individual had the reward in her mind, even
though the reward was not her primary motivation for having performed. She may not have been
entirely motivated by the reward because she was dying but she knew of the reward and we would
presume it was at least in her mind.

Smirnis v Toronto Sun Publishing


Facts: Paul Bernado case. Toronto Sun offered ‘UP TO” $100,000 reward for info leading to arrest
and conviction in Bernardo murders. Smirnis is a shady character and comes to Sun and reveals he
thinks it is Bernardo. Wife gave most info leading to it being Bernardo and that he is not proximate
enough to get reward because it didn’t lead to the conviction.
Decision: Contract. You should be getting the money if you are acting on the contract. Apportioned
the reward based on useful information, given only $10,000.
PROF: This makes no sense – apportionment was never part of the contract!
Reasons: Character of individual should not reflect reward he gets. Motive (whether individual is
motivated by greed or by civic duty) should not matter
RATIO: As long as you are partially motivated by offer at time of performance, it doesn’t matter if
other motivations are “good” or not.

Consideration

General Rule: In order for a promise to be enforceable, promisor must have gotten something
5. Was there in exchange for the promise, OR the promisee has to have done something that is
consideration? of detriment to themselves (Dalhousie v Boutilier)
There is no contract without consideration (Tobias v Dick and T Eaton Co)
Sufficiency of Consideration has to be:
Consideration
Real (Thomas v Thomas – 1 pound/year for maintenance, White v Bluett – doing
something you have no legal right to do – complain)
Not illusory – where nothing is moving from promisee to promisor (Eastwood v
Kenyon – moral obligations; White v Bluett – natural love; Thomas v Thomas –
pious respect for wishes; Dalhousie v Boutilier – subscription of others; Stilk v
Myrick – performance of existing duty (for exclusions to existing duty see
below)
Does not have to be adequate (Thomas)
It must be certain enough to enforce, it is a promise (Dalhousie v Boutilier
Estate – general spending not adequate)
Current, not past (Eastwood v Kenyon – reimburse past upbringing cost).

23
Consideration must be contemporaneous with promise (Roscola v Thomas)
o Exception: Where the promisor requested an action from the
promisee (consideration) under the circumstances where
compensation is expected, past consideration(action) is sufficient
consideration (Lampleigh v Brathwait)
Performance General Rule: Performance of existing duty is not sufficient consideration (Stilk v
of Existing Myrick)
Duties
Exceptions:
1. Contractual duties can be considered valid consideration for future contract if
duties change to an extent that original contract is discharged (Hartley v
Ponsonby)
2. An agreement to do an act for a third party that the promisor is under an existing
obligation to do may amount to valid consideration. (Scotson v Pegg; New
Zealand Shipping)
3. If an amending agreement confers a practical benefit or averts a disbenefit
from the promisor (Williams v Roffey Bros persuasive in Canada, Wind River
Ventures v BC)

The exceptions are inapplicable when:


1. Amending agreement puts economic duress upon the promisor (i.e. an
agreement to accept a lesser sum of money in satisfaction of a greater sum of
money is not enforceable bc no consideration – unless given a practical benefit
(The person was obligated to pay the greater sum or money) (Williams v Roffey
Bros, Foakes v Beer, Pinnell’s Case)
2. Amending agreement entered into under duress (Williams v Roffey Bros; Greater
Fredericton v NAV Canada)
Exceptions to 1. The Seal: a sealed document (containing a wax seal, formal language) is
Necessity of enforceable as a contract without consideration. The sealed document need not
Consideration contain the actual seal – Intention will suffice (Linton v RBC)
2. Mercantile Law Amendment Act
s. 16 – Part performance of an obligation either before or after breach thereof,
when expressly accepted by the creditor in satisfaction, or rendered in
pursuance of an agreement for that purpose, though without any new
consideration, shall be held to extinguish the obligation
If the part performance has bee performed (i.e. owe 100 and agree to take
50 and 50 is given ), then statute says that satisfies the obligation even
though there was no consideration given.
In order to get out of statute, must go through estoppel.
ESTOPPEL
Types of 1. Common Law Estoppel
Estoppel 2. Promissory Estoppel
Common Law If you act in a way that reasonably makes the other party believe you have a
Esptoppel contract, then it’s binding (Cole McIntyre, Jordan v Money)
CL estoppel does not apply to representations of intention or future fact (we
will have a contract), only applies to a representation of fact (there is a
contract). (Jordan v Money)
Amending agreements without consideration are representations of future fact
and not present intention therefore don’t result in CL estoppel and therefore
don’t give rise to contract.
History: Since amending agreements w/o consideration were not covered by CL
Promissory estoppel, DENNING created promissory estoppel in High Trees. (e.g.
Estoppel acceptance of a lesser sum doesn’t result in a contract)
Denning- if you have a promise w/o consideration related to existing

24
agreement, that promise is intended to be acted upon and person acts to their
detriment, then equity arises that stops the person from going back and
insisting on the terms of the initial agreement. Not a new contract. Just an
‘equity’.
Promissory Definition: An equity may arise that stops a promisor from enforcing the original
Estoppel agreement even though the amending agreement had no consideration.

In order to rely on promissory estoppel, you need (Central London Property


Trust v High Trees House):

1. An existing contract
2. A promise related to an existing duty that amends the original contract
Promise must be made voluntarily, without economic duress or other duress
(D&C Builders v Rees)
Promise can be inferred from the promisor’s behavior (Crabb v Arun District
Council)
Promise has to have been a result of negotiations/conduct showing obvious
intent not to insist on strict legal rights (not applicable to friendly
forbearances) (John Burrows v Subsurface Surveys)
3. The promise has no consideration
4. The promise is intended to be acted upon and is acted upon (High Trees) to
the detriment of the promisee (Ajayi v RT Bristol – detrimental reliance)

Are conditions met?


Yes The promisor is therefore estopped from denying the existence of the
amending promise, regardless of lack/insufficiency of consideration
No No estoppel. The amending agreement doesn’t apply bc no consideration.

Limitations on estoppel:
Promissory estoppel does not create new causes of action (i.e. cannot sue for
breach of promissory estoppel) Coombe v Coombe = Shield not a sword
Exception: new causes of action can be created in cases of:
a. Future promises when promisee acted to his/her detriment (Walton
Stores v Maher – not the law in Canada)
- Normally it does not apply to the law in Canada, maybe pursuasive
b. Proprietary estoppel (Crabb v Arun District Council – land)
Prof’s Equitable estoppel is likely a dead end in the law
Predictions on Williams v Roffey Bros will probably be brought into Canadian law. If you have
Promissory a practical benefit in absence of duress agreement is enforceable. This maybe
Estoppel even fixes Foakes v Beer bc there is a practical benefit from taking a lesser
sum of money (getting paid earlier)
Need definitive guidance from SCC on promissory estoppel
PROBLEM: How do you deal with an amending agreement without
consideration?
According to Stilk v Myrick, the general rule is that performance of an existing
duty is not good consideration. One party is not bound in contract for the new
promise bc there is no consideration.

Decision tree to get out of Stilk v Myrick performance of existing duty problem:

Requisites:
1. Original contract
2. Amending agreement to original contract via promise to promisee
3. No consideration (most often performance of existing duty)

25
Possible Options:
1. Practical benefit? (Williams v Roffey Bros)
2. Mercantile Laws Amendment Act – no consideration necessary when offeror
agrees to it. (accepting less for more)
3. Promissory estoppel (High Trees- relied to detriment) – last resort

1. Look at the nature of the obligation and determine if it can be construed that
the amending agreement requires additional performance (Hartley v Ponsonby)
Contractual duties can be considered valid consideration for future
contract if duties change to an extent that original contract is discharged
(Hartley v Ponsonby)
2. Was the obligation under the first agreement owed to someone different than
on the amending agreement (i.e. is a third party involved)? (Scottson v Peg; New
Zealand Shipping)
An agreement to do an act for a third party that the promisor is under an
existing obligation to do may amount to valid consideration (Scotson v
Pegg)
3. Does Williams v Roffey Bros apply?
As long as there is a practical benefit and the amending agreement did
not result from duress or extortion, the promise of existing
performance can be valid consideration for an amending agreement
(William v Roffey Bros; Wind Rivers Ventures v BC)
Do you need a practical benefit? (unclear in Canada – GFAA says maybe
not; Wind Rivers Ventures says maybe yes)
4. Was there detrimental reliance on the promise?
Unsure if needed every time, was needed in Wind Rivers Ventures
5. Does the Mercantile Amendment Act apply?
Part performance w/o consideration is ok when expressly accepted or
rendered pursuant to an agreement.
If the part performance has been performed (i.e. owe 100 agree to take
50 and 50 is given), then statute says that satisfies the obligation even
though there was no consideration given.
Act only applies to a completed agreement and “kicks in” once payment
is performed
Responds to Foakes v Beer
o Can try to get out of Mercantile Amendment Act by cashing cheque
and sending a letter very quickly that says the part performance is a
payment on the account and remainder is expected.
6. Is there an equitable estoppel?
An equity may arise that stops a promisor from enforcing the original
agreement, even though the amending agreement had no consideration
(High Trees)
Estoppel is the last resort for dealing with amending agreement w/o
consideration
In order to have equitable estoppel must have (High Trees):
1) An existing contract
2) A promise related to an existing duty that amends the original contract.
i. Promise must be made voluntarily without economic duress (D&C
Builders v Rees)
ii. Promise can be inferred from promisor’s behavior (Crabb v Arun
District Council)
iii. Promise has to have been a result of negotiations/conduct showing
obvious intent not to insist on strict legal rights (not applicable to
friendly forbearances) (John Burrows v Subsurface Surveys)
3) The promise has no consideration

26
4) The promise is intended to be acted upon and is acted upon (High
Trees) to the detriment of the promisee (Ajayi v RT Bristol – detrimental
reliance).
Are the conditions met?

Tobias v Dick & T Eaton Co [1937] Man KB


Facts: Dick (D) invented grain grinder and enters into written agreements with Tobias (P) for sale of
them. P used trickery to get D to sign contract. Since then, D has accepted and acted on it. P had
exclusive rights to sell D’s machine but P wasn’t required to buy them from D. D decided to explore
other options (Eaton) when P wasn’t buying his machines anymore.
Issue: Was there a contract that was formed if it was formed without consideration?
Holding DYSART: There was no contract between 2 parties, and thus no breach occurred when Dick
decided to sell to someone else.
Reasons: The agreement is a standing offer – a one sided bargain. Inventor gains nothing in
exchange for promise. Standing offer is not contractual and is revocable. The offer is accepted to form
a contract when the person places an order and gives consideration.
Ratio: There must be something given in exchange for a promise – that is the mutuality. Something of
value must flow from promisee to promisor. No consideration = no contract. Standing order is not
contractual and is revocable.

Sufficiency of Consideration

Thomas v Thomas [1842] QB


Facts: Widow Eleanor (P) was promised by the deceased to have the house they lived in or 100
pounds. Benjamin (D) and Samuel (now deceased) were the executors of the will. They allowed the
house to go to the widow for life as long as she remained unmarried and paid one pound yearly
towards gross rent and kept house in good repair. When Samuel passed, D refused to execute
conveyance and ejected the P (life interest has substantial value and was much more than the
nominal rent)
Issue: Was there sufficient consideration to enforce the contract?
Holding DENMAN CJ: Appeal dismissed. Decision for the Plaintiff, there was consideration.
Reasons: Law of consideration does not say that consideration has to be adequate JUST REAL. There
was a ground rent payable not to a superior landlord but to the executors – stipulates an agreement.
RATIO: Consideration must be real and not illusory. It must move from the promisee, which should
be getting something in exchange for the promise to the promisor. Adequacy of consideration is
irrelevant.

Stilk v Myrick [1809] EN


Facts: Seamen sailing from London to Baltic and back again; at Baltic two of seamen quit. P agreed to
receive 5 pounds a month in wages, in Cronstadt, captain agreed to split the wages of the deserted
seamen between the rest of the crew. Captain refused to pay P increased wage.
Issue: Did P offer sufficient consideration in the amending part of the contract to bind D?
Holding ELLENBOROUGH: The agreement is void for want of consideration and P can only recover 5
pounds per month.
Reasons: Before they sailed out of London, they agreed to undertake all they could under all
emergencies of the voyage. Desertion of part of the crew is to be considered an emergency of the
voyage as much as their death, remain bound by the terms of their original contract.
Ratio: Performance of a pre-existing duty is not legally a sufficient consideration for an amending
agreement or new contract.

27
Eastwood v Kenyon
Facts: Eastwood (P) is executor of the will of a friend; he was left to raise his daughter as a good lady.
She gets married and promises to repay P for all that he provided. She didn’t. Court said the
obligation to pay because of a moral obligation is not binding in the court of law.
Issue: Is there consideration in a moral obligation?
Ratio: Moral obligation is not binding in law, it is no consideration.

Hamer v Sidway [1891] NYCA


Facts: Uncle promised nephew (P) to pay him $5000 if he refrained from using tobacco, drinking or
swearing until age 21 at a family celebration. P wrote to uncle informing him that conditions had
been fulfilled. Uncle wrote back that he would hold the money until he considered the young man
capable of taking care of it – consider the money on interest. Died without paying P – claim brought
against executor.
Issue: Does a consideration in the legal sense exist between uncle and nephew?
Holding PARKER: Yes it did. Court sees nothing in the sense to determine that the uncle did not
positively benefit from this agreement in the legal sense.
Reasons: The nephew had restricted his lawful freedom of action within certain prescribed limits
upon the faith of his uncle’s agreement.
RATIO: A detriment to the promisee is good consideration. Consideration can be beneficial or
detrimental at the request of the promisor.

White v Bluett [1853] EN Ex Ct


Facts: D (son of John Bluett) complained about not getting as much money as siblings to father.
Father agreed and said if he didn’t complain anymore he would discharge the son of liability to pay
him the amount in a promissory note to him.
Issue: Is an agreement by a father in consideration that his son will not bother him, a binding
contract? Is there valid consideration?
Holding POLLOCK: For the Plaintiff (Executor), no contract created. No consideration.
Reasons: Son had no right to complain, the father might make what distribution of his property he
would like and the son abstaining from doing what he had no right to do can be no consideration. No
consideration when you give up something that you aren’t legally entitled to do (illusory
consideration).
RATIO: No consideration, reciprocal exchange is necessary for consideration – there is no
consideration when give up something that one is not legally entitled to do. Consideration must be
real and not illusory.

Dalhousie v Boutilier Estate [1934] SCC


Facts: Boutilier promised to pay Dalhousie $5000 in a campaign run by the uni to raise funds “to
improve the efficiency of teaching and in subscription to other people, to construct new buildings
and to keep pace with growing needs of its constituency” with terms of payment “as per letter from
Mr. Boutilier”. No letter ever followed, Boutilier fell on economic loss and could not pay. He
acknowledged that he intended to pay and would do so when could afford to. He died, Dalhousie
claimed against his estate for the money. Dalhousie successful at trial. Boutilier estate appealed and
won. Dalhousie appealed to SCC.
Issue: Is a gratuitous subscription promise sufficient to find a binding contract?
Holding CROCKET J: Appealed dismissed with costs.
Reasons: The gratuitous promise did not receive any consideration, and therefore not a binding
agreement. Boutilier did not agree to pay the money for a specific reason, he was not getting a

28
specific benefit out of it. If had donated money for building of a specific monument there could be a
consideration – therefore no binding agreement.
RATIO: A gratuitous promise does not have sufficient consideration to be considered a binding
contract unless the money was given for a specific purpose which can be seen as some benefit to the
promisor. It is illusory otherwise.

The Seal
An early form of action, called covenant, permitted enforcement of promises, even without
any element of exchange, if under seal. The seal, originally molten wax dropped onto the
document and impressed with a die, later took the form of a gummed paper wafer. A sealed
document (also known as a deed, covenant, formal contract, or a specialty) usually contains
formal language and ends with the words, “signed, sealed and delivered.” Through the
passage of time, the requirements of signing, sealing and delivering have become much
attenuated.

Linton v Royal Bank of Canada [1967] HCt CAN


Facts: Document containing words “signed, sealed and delivered” and which has the words “seal” in
parentheses beside space for signature, guarantor signed documents without altering either of those
printed items and delivered to the creditor. Bank added a paper seal afterwards.
Issue: Does the bank adding a paper seal count as a “material alteration”?
Holding: No it does not.
Reasons: It was an alteration that carried out the intention of the parties already apparent on the
face of the document. The document being a deed from the outset, the addition of a paper wafer by
the bank cannot be considered a material alteration affecting the ‘legal incidence’ of the document.
RATIO: When a party signs a document, which by its language purports a deed, he cannot afterwards
deny that it is so.

Past Consideration

Roscorla v Thomas [1842] QB


Facts: P bought a horse from D for 30 pounds, no promise at the time of the horse’s qualities. Later D
did promise P horse was free of vice. Then appeared horse was “very vicious, restive, ferocious.”
Action brought by P on an assumpsit (implied promise) and verdict for P, D obtained a rule nisi to
arrest the judgment on the ground that there was no consideration in this implied promise.
Issue: Did P provide enough consideration to bind D to later promise of quality of horse?
Holding: No contract. No consideration.
Reasons: The precedent sale without warranty, at request of the defendant, imposes no other duty
or obligation upon him. It is clear therefore that the consideration stated would not raise an implied
promise by the defendant that the horse was sound or free from vice.
RATIO: Consideration must be contemporaneous with the promise made. A promise after a bargain
is struck is not good consideration.

Lampleigh v Brathwait [1615] UK KB


Facts: Brathwait killed a man and then requested Lampleigh seek a pardon for this crime from the
King. Lampleigh rode around the country to obtain his pardon, after which Brathwait promised to
pay Lampleigh 100 pounds.
Issue: Can a promise to pay after a request has been fulfilled be binding?

29
Holding: Binding contract found. Judgment for the Plaintiff.
Reasons: Court held that while a mere voluntary promise is not sufficient consideration, there was
prior request and then promise to pay. This is then not a nudum pactum (bare or naked promise –
promise that is not legally enforceable for want of consideration) but rather coupled with prior
request and therefore a binding contract.
RATIO: A promise made after performance can be enforced, only if it was understood by the parties
that they would have some kind of reward prior to the performance.

Performance of an Existing Duty

Stilk v Myrick [1809]


Facts: Seamen quit mid trip, rest of crew promised their pay in if they continue on the ship.
Afterwards, captain refused to give them the excess in pay.
Issue: Is there a consideration for the ulterior pay promised to the mariners who remained with the
ship?
Holding: No contract (in favor of D). No consideration. Obligation not enforceable.
Reasons: Court held that the sailors were already obligated to bring the boat back to London so it
was not good consideration. If you are bound in contract to give a certain performance and enter into
an amending agreement and one of the parties gets more (i.e. the crew) that amending agreement
isn’t enforceable bc one of the parties didn’t get consideration (i.e. the captain)
RATIO: performance of an existing duty is not good consideration for an amending agreement or
new contract. A promise to do what you are already bound in contract to do is not good
consideration. – Likely now the amending agreement would have been enforceable
- Does the amending agreement give to the promisee and promisor? If not there is no consideration
to make the new contract valid

Hartley v Ponsonby [1857] UK


Facts: facts similar to Stilk v Myrick, but reduction in ship crew made operation dangerous to life. It
was incumbent on the P to perform the work; and he was in the condition of a free man (his contract
was ended – it stopped meeting the terms of safe work).
Issue: Did the crew offer valid consideration to enter into bargain to operate ship with reduced
number of crewmembers for a higher wage?
Holding: There is a contract. Additional consideration was given.
Reasons: Crew offered valid consideration to enter into a new contract with the captain bc the
situation resulting from half the crew deserting was substantively different from the original contract
which is therefore null. Sailors no longer obligated to perform bc of dangerous situation.
RATIO: Exception to Stilk. Performance of contractual duties can be valid consideration for a new
contract if the duties change to such an extent because of an external cause that the original contract
is discharged.

Scottson v Pegg
RATIO: Exception to Stilk. If you have a contract between A and B for a performance and C offers B
contract of the same value. B can agree with a third party to do something that they’re already bound
in contract to do for A. The consideration for C is having the contract fulfilled for A. If B doesn’t build
the bridge, he will be sued by two ppl. Obligation for the same performance under two separate
contracts.

New Zealand Shipping v Satterthwaite [1975]


RATIO: Exclusion to Stilk. If I have a contract between A and B and then C offers to pay B for the

30
same performance, that is allowed bc there is good consideration. C has an independent right to sue
B for the performance.

Williams v Roffey Bros. & Nicholls [1991] QB CA


Facts: Williams (P) was carpenter contracted by Roffey (D) to renovate 27 flats in London for a total
contract price of $20,000. After 6 months, 16,200 had been paid. D’s surveyor Mr. Cottrell persuaded
D to pay bonus to P to keep working bc the subcontract was too low – this would alleviate the stress
of D to avoid the penalty clause in the main contract with the Housing Association in terms of delays.
Agreed to pay additional $10,300. One and a half months later, only $1500 had been paid. P sued for
remaining contract price and extra payments agreed to by D.
Issue: Was the variation of the contract made with consideration?
Holding GLIDEWELL: In favor of P, the D’s promise to pay was supported by valuable consideration
because D received a practical benefit.
Reasons: D had a concern about a failure to perform, therefore a practical benefit was created in
ensuring the performance. This benefit was not obtained in duress.
RATIO: As long as there is a practical benefit and the amending agreement did not result from duress
or extortion, the promise of existing performance can be valid consideration for an amending
agreement.

Greater Fredericton Airport Authority v NAV Canada [2008] NBCA


Facts: NAV and GFAA were parties to the Aviation Services Facilities Agreement, which included
governing responsibilities for certain capital expenditures. A $6,000,000 runway extension project
requested that NAV relocate an instrument on a landing strip to the one that was being extended.
NAV concluded that it would be better to replace the navigational aid with another type. GFAA
believed that NAV should pay for the acquisition costs; NAV stated they wouldn’t if GFAA didn’t agree
to pay. GFAA wrote a letter saying they would pay BUT “under protest – feel that they were being
compelled to pay” they signed the agreement under these terms. NAV completed the work and GFAA
refused to pay.
Issue: Was the contract made by GFAA to buy equipment in exchange for NAV continuing to perform
contractual duties sufficient consideration?
Holding: In favor of GFAA, no contract.
Reasons: Applying Williams v Roffey Bros: In supposed contract – NAV gets paid higher price, GFAA
receives a practical benefit. But contract was formed under duress – contract isn’t binding.
RATIO: William v Roffey Bros is good law in Canada. A post contractual modification unsupported by
consideration may be enforceable as long as it is established that the variation was not procured
under economic duress. 1) was the promise made under pressure 2) Did the pressured party have no
option but agreeing.

Wind River Ventures v British Columbia [2009] BCSC


RATIO: Agree to apply GFAA but with a refinement. In order to apply GFAA, need either 1) a situation
where other party relied to their detriment on amending agreement or 2) the party has to receive
some kind of benefit. A real benefit (i.e. an increased performance) or a practical benefit. Overall this
case supports GFAA but need something more than an amending agreement.

Jonathan’s Aluminum v Retail Alley Metal


RATIO: Ontario Court likes the idea of Williams v Roffey Bros. Court concluded that non-completion
of an agreement was not a variation of the contract – there was no consideration, therefore no
binding contract. Court says that does not need to apply Williams v Roffey Bros because the non-
completion was simply a request for an agreement without consideration.

31
Pinnell’s Agreement [1602]
Facts: D owed 10 pounds 8 shillings to P, the P accepted 5 pounds early instead of the full amount of
the debt. Sir Edward Coke: greater sum of money can be collected, unless there was an accord and
satisfaction (consideration). P got money early = BENEFIT. Consideration has to be real, it doesn’t
have to be accurate. If there is an obligation to pay a greater sum and you agree to a lesser sum, it’s
not enforceable – must get a benefit.

Foakes v Beer UK [1884]


Facts: As a result of a previous judgment of the Court of Exchequer, Foakes owed Beer 2090 pounds.
The two parties entered into an agreement on December 21st (not under seal) that Foakes would pay
500 immediately and 150 every six months until he paid off debt and in return Beer wouldn’t take
any action. By June, Foakes has paid off entire principle and sought to leave proceed on the judgment.
Beer claimed she was entitled to interest bc debt wasn’t paid immediately. Foakes claimed there was
a contract with no mention of interest – Beer claimed there was no consideration for it to be a
contract.
Issue: Is Beer entitled to the interest despite their agreement that she would not need to pay it?
Holding: No contract, interest payment due
Reasons: Court decides there is no consideration to bind Mrs. Beer not to sue for the money. Apply
Pinnel’s Case doctrine: “payment for a lesser sum on the day in satisfaction of a greater cannot be any
satisfaction for the whole.” BLACKBURN: Beer got a benefit from not having to execute judgment,
Foakes paying money over time – is it a benefit for Beer not to have to execute on the judgment to get
the money? It is an illusory benefit – Foakes had to pay the money regardless.
RATIO: Agreement to accept a lesser sum of money to serve as satisfaction of a larger amount isn’t
binding, unless the creditor got a practical benefit – early payment, different form of payment.

Mercantile Laws Amendment Act RSO 1990


16. Part performance of an obligation, either before or after a breach thereof, when expressly
accepted by the creditor in satisfaction, or rendered in pursuance of an agreement for that purpose,
though without any new consideration, shall be held to extinguish the obligation.
- If you have an obligation to pay 1000 and accept 500, or if I render 500 to you pursuant to the
obligation, then you cannot collect the balance. However, if there was no acceptance or rendering or
tender, the statute doesn’t apply
- Must be a completed bargain – money is accepted or paid to the creditor
- Act only applies to a completed agreement, not a bargain to take a lesser sum of money. It “kicks in”
once payment is performed.
- Act responds to Foakes v Beer which would hold that if you cash the cheque you still have access to
the remainder. Mercantile Law Amendment would have said that part performance has been
expressly accepted or rendered pursuant to the agreement and therefore the obligation is
extinguished.
- Statute purports to override the Common Law
E.g. A debtor is owed $10000. The person owing the debt send a cheque for $7000 with a letter
stating that “If you cash this cheque, I’ll assume you’ve accepted my offer to give you the money to
extinguish the whole debt.”
Forbes solution: Cash the cheque and send them back a letter v quickly that says that it’s a payment
on the account and we have every intention of getting the remainder. Accepted on account. Not
payment in full.
• Two courses of action:
1) can claim no agreement. There is no contract. It’s not part performance bc there is no agreement
2) Argue that payment wasn’t expressly accepted or rendered pursuant to an agreement. That’s what
the letter is for (try to get out of the Mercantile Amendment Act)

32
Promissory Estoppel

Jordan v Money [1854]


- Cannot use CL estoppel to create a contract where there is no consideration. An amending
agreement does not create a CL estoppel.
- The representation that there will be a contract does not create an estoppel, only the
representation that there IS a contract creates an estoppel.
- These are binding facts DENNING used in High Trees.

Central London Property Trust v High Trees [1947] KB


Facts: Central London (P) leased a block of flats for$2500/yr from High Trees (D) in 1937. Due to
war and heavy bombing in London, P agreed with D to accept a reduced rate of 1250/yr “as the
commencement of the lease.” March 1941, affairs of P were handed over to a receiver who looked at
lease in Sept 1945, saw flats were in full occupancy, even though D was paying reduced rent. P sued
for payment of the full costs from 1945 onwards.
Issue: After making a promise to a promisee to amend a contract that is backed by no consideration,
can the promisor sue the promisee based on the original contract?
Holding DENNING: Judgment for P in the amount requested.
Reasons: Full rent was payable from the time the flats were at full occupancy in 1945. However, full
rents would not have been able to be collected from 1940 bc equity would have estopped P from
collecting. A party who waives a part of performance of a contract may later re-instate that portion if
it would not be unjust or violate the reliance of the other party.
RATIO: The rule of promissory estoppel states, if you have a 1) promise related to the performance
of an existing duty amending a contract; 2) that promise is not backed by consideration; 3) the
promise is intended to be, and is, acted upon; then an equity arises that estops the promisor from
acting on his/her legal rights under the original contract.

John Burrows Ltd v Subsurface Surveys [1968] SCR


Facts: John Burrows (P) sold substantially all of its assets to Subsurface (D) for $150,000. Majority of
purchase was paid in cash, with balance paid in promissory note. D promised to pay interest
regularly (6% per annum payable monthly – any default by D became immediately payable). D paid
interest payments but frequently paid them late. P and D had amicable terms and this was accepted.
Falling out occurred, P expressed interest payment was 36 days late. D tendered a sum of money for
overdue interest. P rejected pay and commenced an action seeking full payment for outstanding
amount. P won at trial, D succeeded in appeal.
Issue: What is necessary for estoppel to apply?
Holding RITCHIE: Appeal allowed and restore learned trial judgment. Appellant is entitled to its
costs.
Reasons: Estoppel does not apply in this case. Estoppel to apply, the conduct of P must amount to a
promise or assurance intended to alter the legal relations between the two and that it is impossible
to infer from the facts of this case. He was simply acting as a friend not entering into any negotiations
over new terms of payment.
RATIO: Promissory estoppel only results from agreements formed as a result of 1) negotiations that
are intended to be binding, and 2) not from a simple, friendly forbearance. It must be clear that the
agreement can be relied upon.

Ajayi v RT Briscoe
RATIO: You must have DETRIMENTAL RELIANCE to plead promissory estoppel. Promisee must
show that they relied and acted on the promise to their detriment. If there is no detrimental reliance,
then the party making the promise can insist on his/her original legal rights after giving notice after a

33
reasonable period of time.

D&C Builders v Rees UK [1965]


Facts: D&C (P) did work for Ree’s (D) shop totaling 746 pounds. D paid 250 on the account and made
no further payment. There was no dispute about the work done at the time. P contacted D twice, to
no reply for the remainder of money. Nov 13th 1964, D was ill, his wife called P stating “300 pounds
settlement. That is all you will get. 300 is better than nothing.” And complained about the work. P
would be in a state of bankruptcy did not accept, informed D’s wife about this and she still pressed
the lesser payment. P accepted cheque and later sued for full payment.
Issues: Can a party accept a lesser amount for satisfaction of a debt and then demand the payment in
full?
Holding DENNING: Appeal was dismissed. For the Plaintiff, should get full payment.
Reasons: Substitute agreements that satisfy the necessary accord can be valid in equity, even if they
do not have consideration, if it would be inequitable to allow the creditor to sue for the money from
the original contract. To satisfy this requirement, an agreement must have been made, the debtor
must have relied upon it and it must be unfair to allow the creditor to claim more money. The
pressure placed on P forced them to accept the agreement = makes the substitute agreement
unsatisfactory.
RATIO: An equitable remedy such as a promissory estoppel must be fair and cannot result from
inequitable actions. Amending agreement cannot be extorted under duress.
- Equity does not permit for people to ask for a defense when they acted inequitably – therefore D
cannot plead a promissory estoppel.

Coombe v Coombe [1951] CA


Facts: Mr. Coombe made an agreement to pay his estranged wife 100 pounds per year. There was no
consideration in exchange for the promise and so no contract was formed. The wife brought an action
to enforce the promise invoking promissory estoppel that she had relied on the promise to her own
detriment. The trial judge held that estoppel could be a cause of action. Mr. Coombe appealed.
Issue: Can a promissory estoppel become a legally binding contract?
Holding DENNING: No, promissory estoppel does not create a binding contract.
Reasons: Cannot plead promissory estoppel as a plaintiff to form a remedy. Can only plead
promissory estoppel as a defendant to stop a plaintiff from insisting on their strict legal rights under
an original contract. “Estoppels are shields, not swords.”
RATIO: Estoppel is only a defence, not a cause of action where one did not exist before. They do not
form contracts. They are shields to defend against enforcement of original contract, they do not
create enforceable rights, they do not form contracts.

Exception to Coombe: Estoppels do not create a cause of action except when:

Crabb v Arun DC [1976] CA


Facts: Claimant was assured that his local council would build a right of way to his land, such that the
land could be partitioned and sold off separately without leaving one part of land landlocked. To
confirm their intentions, council built a fence with a gap for the assured right of way. The claimant
relied on this assurance to sell part of his land, leaving his own landlocked. The council then filled in
the gap and demanded 3000 pounds for a right of way to be built.
Issue: Can promissory estoppel ever become a legally binding contract?
Holding DENNING: Yes.
Reasons: Any sum that could reasonably have been demanded for the cost of construction was set
off against the detriment suffered by the claimant; the net cost being 0 pounds with the council being

34
required to construct the right of way. Where a proprietary estoppel is established, a court will
assess the extent of the equity created and how best to satisfy it.
RATIO: A contract made in negotiation when involving land, may in fact be enforceable. A
representation about there being a contract unsupported by consideration about an interest in land,
creates a proprietary estoppel.

Walton Stores v Maher [1988] Aus HC NOT LAW IN CANADA


Facts: M owned property and W was negotiating a lease of it, granted that M would demolish an
existing building on the property and erect a new one for W. Draft leases were sent between them,
negotiating amendments. In Nov, M informed W that the lease must be concluded within 2 days in
order to organize building supplies. W did not object to any amendments in the next few days, M
began the demolition. Later, W started to have reservations (realized not bound by agreement yet)
instructed solicitors to ‘go slow’ but did not talk to M. W shortly after became aware that demolition
was proceeding. When building was 40% complete, in Jan, W informed M did not wish to proceed. M
sought promissory estoppel.
Issue: Is this a CL estoppel? (i.e. did Walton by their conduct represent that they had accepted the
offer?) Did they represent that there WAS a contract or WOULD BE a contract? What is the difference
between equity and a contract?
Holding MASON & WILSON: Appeal dismissed. W is estopped (prevented) from retreating from its
implied promise to complete the contract.
Reasons: Although here there is no promise/contract, M was entitled to assume the exchange of
contracts was a mere formality. Promissory estoppel extends to promises to future contract.
Promissory estoppel can only enforce voluntary promises when a departure from the parties’ basic
assumptions is unconscionable [not just]: 1) Reliance on an executor promise resulting in the
promisee changing his position or suffering detriment 2) Creation of encouragement by the party
estopped [W] in the other party [M] of an assumption that a contract will come into existence 3)
Reliance by the other party [M] on that assumption to his detriment to the knowledge of the first
party [W]. M had contacted W about start of demolition – W’s inaction enabled the promissory
estoppel.
RATIO: If the circumstances are such that there is an active encouragement of an assumption of an
intention to contract, then it is possible that the promisor is estopped from denying the contract.
- There may be liability in actively encouraging someone that there is a contract if the other person
acts to their detriment.

Capacity to Contract

General Rule: Minors (i.e. infants) and the mentally incapable are not bound by contracts,
into which they enter – even though the other party may be bound by the
contract.
Minors (anyone under the age of 18)
Deemed to lack contractual capacity
Contract is voidable by the minor including when they reach the age of
majority, however, if at that age they accept the contract or act consistently
with its terms then contract becomes binding (Nash v Inman, Toronto
Marlboro Hockey Club v Tonelli)
Exceptions:
1. Contract is valid when the goods that are provided to minors on contract are
necessities (Nash v Inman; Sale of Goods Act)
o Necessities are goods that are suitable to the 1. Condition of life of
that infant and 2. His current requirements (Nash v Inman)
2. Contract for a minor’s services is valid when it is 1. Beneficial to the minor
for the entire duration of the contract AND 2. The benefit is clearly apparent

35
(Toronto Malboro Hockey Club v Tonelli)

Mentally Incapacitated:
3. Individuals are deemed to lack contractual capacity permanently (bc they
have a mental illness) or temporarily (bc the are too intoxicated)
Section 3 of Sales
of Good Act 1990 (1) Capacity to buy and sell is regulated by the general law concerning capacity
to contract and to transfer and acquire property, but where necessaries are
sold and delivered to a minor or to a person who by reason of mental
incapacity or drunkenness is incompetent, s/he shall pay a reasonable price
therefor.

(2) “Necessaries” means goods suitable to the conditions in life of the minor or
other person and to his or her actual requirements at the time of the sale and
delivery.
Decision Tree: If a person is deemed to NOT have capacity, ask:

1. Did the incapacitated party have enough knowledge to appreciate the nature
of the contract at the time?

2. Was the contract for a “necessary” pursuant to s. 3 of the Sale of Goods Act?
Yes They are bound by the contract even if no capacity to contract,
just have to pay for the goods, not service price (Nash v Inman; Sale of
Goods Act)
3. Is the contract for minor’s services?
Yes Contract is valid if it’s beneficial to minor through entire
duration of the contract AAND benefit is clearly apparent (Toronto
Malboro Hockey Club)
4. When the person re-acquires capacity, then they can approve the contract by
expression or by acting consistently with the contract. Or the person can void
the contract (Nash v Inman)
Did the person (re)acquire capacity and approve the contract either
expressly or impliedly?
Yes Contract is valid even though they previously didn’t have
capacity

Section 3 of Sale of Goods Act RSO 1990, c s 1


Capacity to buy and sell is regulated by the general law concerning capacity to contract and to
transfer and acquire property (can’t sell to children under 18, if do, the contract is voidable) BUT
where necessaries are sold and delivered to a minor or to a person who by reason of mental
incapacity or drunkenness is incompetent to contract, he or she shall pay a reasonable price therefor.
- Necessaries means goods suitable to the conditions in life of the minor or other person and to his or
her actual requirements at the time of the sale and delivery.

Nash v Inman [1908] KB CA


Facts: Tailor supplied clothes to D for 145 pounds, the D was an infant at time of sale and delivery of
goods, which was not admitted. He did not pay. P brought action and only claimed 122 pounds – the
cash price of the goods and not the credit price. Learned judge held that there was no evidence that
the goods were necessaries and directed judgment for D. P appealed.
Issue: Did P sell D necessaries of life and therefore deserves payment from an infant?
Holding: COZENS HARDY, MOULTON: Appeal dismissed. Clothing was not necessary.

36
Reasons: It was proved by D’s father that he was an infant and had adequate supply of clothing. The
true basis of an action against an infant for necessaries is not a contract that forms; its real
foundation is an obligation, which the law imposes on the infant/their guardian to make a fair
payment in respect of needs satisfied.
RATIO: In order to receive payment from an infant in a contract in terms of the Sales of Goods Act, P
must prove that the goods supplied were necessaries to the infant in their life.
An exception: when child reaches age of majority and continues with the contract – child’s right to
void the contract goes away. If want to void contract, have to do it very quickly after reaching age of
majority.

Toronto Marlboroughs Hockey Club v Tonelli [1976] OR


Facts: D entered into a contract with P, an amateur hockey club, at age 16. Agreed to play for 4 years
and pay P 20% of earnings during first three years as pro. At 18, D left and started playing for pro
hockey club. P started an interim injunction, but dismissed: Infant’s contracts of service are only
binding if on the whole they are beneficial and this could not be shown on the onerous and one-sided
agreement in question.
Issue: Is the contract of service entered into by D (an infant) binding, is it solely for the benefit of
him?
Holding MORDEN: No it is not binding or enforceable. Contract was not binding at the end of the
contract – was stuck playing minor league and had to pay 20% of salary. Did not confirm contract
when became majority age.
Reasons: Onus is on the P to prove that the contract was for the benefit of the infant before they can
sue to make it binding. The contract keeps him from signing professionally for a long time, limits his
earnings and development.
RATIO: When the contract is for the minor’s services, the contract is valid if the contract is beneficial
to the minor for the entire duration of the contract and the benefit is clearly apparent.

6. Certainty or Ascertainability of Terms

Certainty of The court will make an effort to make the contract certain and enforceable
Terms where the terms are [certain or ascertainable] or [complete or
completable] pursuant to a mechanism that does not fail.
However, the court will not create entire contracts for parties (Scammel and
Nephew v Ouston – hire purchase agreement is too complex and mechanism
failed).

Three Types of Cases (fact patterns)


1. Contract is COMPLETE but terms are UNCERTAIN (i.e. dispute
about the meaning of the language used)
General rule is that courts will interpret the meaning from the document
o Only where there is ambiguous language will courts go “behind the
document”
Factors that will be considered in ascertaining terms (Hillas v Arcos)
o Past dealings bw parties
o Industry standards (or meaning known to both parties)
o Correspondence bw parties before coming to agreement
o Forms the parties have used before
o Plain meaning of language
o Circumstances
Are the terms of the contract ascertainable through a mechanism that
works, OR through the factors above that show an intention to agree on the
term (Hillas v Arcus)

37
Yes Contract is enforceable
No Contract is not enforceable
o Contract is not enforceable where terms are not ascertainable
pursuant to the mechanism given (Scammell & Nephew v
Ouston, agreement to agree, mechanism failed)
2. INCOMPLETE contract but there is an agreement to NEGOTIATE the
terms later.
Are the terms of the contract completable through a mechanism that works?
Yes Contract is enforceable
No Contract is not enforceable. If mechanism failed, no contract.

Agreement to Agree:
An agreement to agree does not work bc the mechanism the parties agreed on
to determine the terms of the contract (i.e. agree at a later date) failed.
Therefore the contract is not enforceable. (Walford v Miles; Scammel & Nephew v
Ousten)

Agreement to Negotiate in Good Faith:


An agreement to negotiate in good faith is another mechanism like an
agreement to agree. If the mechanism fails, then contract is not enforceable
(Walford v Miles)
Exceptions:
An obligation to negotiate in good faith is ONLY enforceable in
situations where it is obvious that a party negotiated in bad faith
(Edpar Brascan Corp v 117373 Canada Inc.) Empress Towers is bad
law.
Where negotiation/determination of terms is to be done by a third
party, and if they are available (arbitrator), the mechanism works
and the contract is enforceable (Calvin Consolidated Oil & Gas v
Manning)
3. INCOMPLETE contract or UNCERTAIN terms, accompanied by a PRE-
CONTRACTUAL document
General Rule: If a pre-contractual agreement is complete and contains all
the prerequisite elements of a contract, the court may hold that the
document is an enforceable contract (British American Timber v Elk River
Timber Co)
You can include a Rose and Frank clause and/or Green Ainsmore clause (i.e.
“subject to a formal contract”) to ensure pre-contractual documents are not
considered contracts. A Green v Ainsmore clause states that the document is
not a legally binding contract until a formal contract is developed and
signed.
The more complicated or unique the agreement, the less likely the court is
to say that the pre-contractual document is a binding contract.

1. Does the pre-contractual document contain all the elements of a contract?


a) YES Court may hold that it’s an enforceable contract (British
American Timber v Elk River Timber Co)
o Was there a Rose and Frank and/or Green v Ainsmore clause?
YES No contract.
NO Could likely argue it’s a contract.
b) NO Unlikely to be seen as a contract.

38
Complete Contract, Incomplete Terms

Scammel & Nephew v Ouston UK [1941] AC HL


Facts: Scammel (P) agrees to purchase a van from Ouston (D) in exchange for another van and “the
balance of purchase price on hire-purchase terms over a period of two years.” D wrote that the van
would be ready for collection “subject to mutual acceptance of the hire-purchase agreement.” Later
the D changes his mind and decides he does not want the transaction because the P misrepresented
the kind of van they were giving the defendant in exchange. D argues no contract bc term “hire
purchase terms” is unclear. The trial judge awarded the P damages. Court of Appeal dismissed the
appeal. D appealed to HL.
Issue: Is there a contract formed between P and D with certain terms?
Holding WRIGHT: Appeal allowed, no contract was formed.
Reasons: The terms were so obscure and so incapable of any definite or precise meaning that the
court is unable to attribute to the parties any contractual intention. The parties never in intention nor
even in appearance reached an agreement – never got beyond negotiations.
RATIO: 1) If court analyses what the parties intended by imprecise terms and can’t figure it out,
contract fails. 2) If parties agrees to agree to terms of the contract later on and then can’t agree, the
contract fails bc parties had a mechanism for determining the terms of the contract and it failed. An
agreement to agree is not a contract unless the parties subsequently agree.

Hillas v Arcos [1932] HL


Facts: Hillas and Company were merchants purchasing timber from Arcos, agreed to purchase
22,000 standards of timber, under the specific condition that they should also have the option of
entering into a contract with Arcos to purchase 100,000 the following year with a 5% reduction
price. Arcos refused the following year. Arcos was successful at trial, Hillas appealed successfully in
CA.
Issue: Was the imprecise term of negotiating the future sale a condition of the contract?
Holding WRIGHT: Appeal allowed for Hillas, a binding contract to sell 100,000 the following year.
Reasons: “Words are to be interpreted so that subject matter is preserved, not destroyed.” The only
thing that had to be negotiated was the price, because this changes yearly.
RATIO: In determining the meaning of imprecise language, the courts will take into account the
following concepts: past dealings between parties, industry standards, correspondence bw the
parties before coming to an agreement and forms the parties have used before. Courts will try based
on evidence to determine what the terms were. If they can. There is a contract on those terms. If they
can’t, contract fails. A contract is enforceable if its terms can be ascertained through a mechanism or
factors that can show intention to agree on certain terms.

Incomplete Contract, Agreement to Negotiate

Walford v Miles [1992] HL


Facts: P was going to buy D’s business; agreement says D won’t negotiate with anyone else. D will
negotiate in good faith with P to complete the terms of the contract. D gets better offer from someone
else, enters into contract to sell someone else. P says breached contract.
Issue: Can you make an agreement to bargain in good faith? If you enter into an agreement to
negotiate in good faith, does that put a contractual restriction on seller or is it a failure of a
mechanism?
Holding: No contract, judgment for D. Mechanism fails.
Reasons: Each party to negotiations is entitled to pursue his/her own interest, so long as he avoids
making misrepresentations. An agreement to negotiate, like an agreement to agree is unenforceable
simply because it lacks the necessary certainty. No shop clause can only be enforceable for a certain

39
period of time.
RATIO: Parties must agree on the mechanism to set the terms of the contract. An agreement to
negotiate in good faith is no different than an agreement to agree to the terms at a later date.
If the agreement doesn’t work because the mechanism fails, then the mechanism to negotiate in good
faith also fails. Both fail bc terms or not complete or completable pursuant to a mechanism that
works.

Empress Towers v BNS [1990] CA – Confined to specific facts in Edper Brascan Corp
Facts: Empress Towers (the proprietor) has a five-year lease with the Bank of Nova Scotia to lease a
space for a bank. The parties have an agreement to agree to the second five-year term for the lease.
Well before the lease is up the Bank presents terms for the second five years of the lease. The week
before the lease is up the plaintiff agrees to the terms but asks for $15,000 additional. Bank of Nova
Scotia sues Empress Towers for not following their “agreement to agree.”
Issue: Is the renewal clause void either for uncertainty or what is fundamentally the same as an
agreement to agree?
Holding LAMBERT: Dismissed claim for writ of possession under Commercial Tenancy Act. Empress
Towers had not negotiated in good faith.
Reasons: An agreement to agree is no agreement at all – no contract
RATIO: Court ruled that landlord had an implied obligation to negotiate in good faith BUT
generally no contract bc there was an agreement to agree. Case caused huge problems: courts
decided that Empress decision should be confined to its own set of facts

Edper Brascan Corporation v 117373 Canada Inc. [2000] SCJ ON


LANE J:
- In the light of subsequent cases, perhaps Empress Towers should be regarded as confined to its
very narrow set of facts, and not as authority for a general proposition that the duty to bargain in
good faith exists whenever a negotiation takes places within an existing contract. To some extent this
seems to have happened
RATIO: An implied obligation to negotiate in good faith is enforceable only in situations where it is
obvious that the party negotiates in bad faith. This overturns Empress Towers.

Terms appear incomplete (might be a contract)

Calvin Consolidated Oil & Gas Co v Manning [1959] SCR


Facts: Parties enter into an agreement and terms are to be mutually agreed to. If decision is
made to make land development for oil and gas a priority, then terms of contract will go
through arbitrator. D tries to walk away from contract on terms that it is not binding. P sues.
Issue: Is an arbitrator a mechanism that works to ascertain the terms of a contract?
Holding: Yes, judgment for the plaintiff. There is a contract UNTIL the mechanism fails.
Reasons: An agreement to agree cannot be appreciated in courts – HOWEVER, a provision
that the new terms can be settled by an arbitrator CAN be enforceable. Saying that if you
can’t agree to terms of contract, will send to arbitration, that is your mechanism for settling
the terms of the contract. The completeness of the terms will be set by a mechanism, unless
the mechanism fails.
RATIO: Only when the mechanism fails and have incomplete terms can we say the contract
fails. Arbitration is the mechanism to have the contract work.

40
British American Timber Co v Elk River Timber Co [1933] BCCA
Facts: Parties entered into what is contended to be a binding and enforceable contract – to purchase
timber limits, dated June 15, 1931. Contract was partially performed. A full formal agreement is
intended in fulfillment of cl. 10 “so soon as the cruise and survey have been completed.” On Sept 5, it
was completed – no formal agreement made; P sued for specific performance of the agreement
executed on June 15th
Issue: Is an agreement that is complete in itself but no formal contract has been drawn up that
embodies its terms, enforceable?
Holding MACDONALD: Appeal must be dismissed, in favor of the P, convinced that they had
concluded a contract and the respondent’s attempt to now recede from it cannot be countenanced.
Reasons: Times of payment are clearly established, purchase-money is to be paid in cash upon
execution of the formal agreement and the formal agreement is to be executed as soon as the cruise
and survey had been met. A contract which has been duly assented to by the parties but in which
they stipulate for a formal agreement has been itself enforceable (Chinnock v Marchioness)
RATIO: When an agreement is complete in itself, the fact that a formal contract is to be drawn up
embodying its terms does not render it unenforceable – the more complex and unusual the contract
drawn up is, the more likely a court is to not find it to be a contract.

Bawitko Investments v Kernals Popcorn


Facts: Parties negotiate letter of intent – franchise fee, renewal fee, no guarantees, deposit, rest of
terms in draft sent. Kernals returns deposit and back out. Franchise sites British American Timber.
RATIO: Agreement to agree is not a contract. The more complicated the agreement the less likely for
courts to say it is a ‘contract.’ In CONTRAST TO British American Timber. The contact is worthy of
negotiation and the parties haven’t provided another mechanism for completing the terms.

Green v Ainsmore Consolidated [1951] BCSC


Facts: A letter outlining some detail clauses for an agreement was approved by the P and signed by
appropriate officers of D’s company. It requested that P submit to us a draft form of agreement and
concluded with words “This memorandum shall be subject to a formal agreement of sale and
undertaking being prepared, satisfactory in form to the solicitors of both parties”
Holding: Letter was not an enforceable contract.
RATIO: Can use a Green v Ainsmore “subject to contract” clause to invalidate an informal contract.
The clause states that the document is not a legally binding contract until a formal contract is signed
and developed.

C. Terms of a Contract
1. Parole Evidence Rule
PAROLE GENERAL RULE:
EVIDENCE • When interpreting the meaning of a contract, the court will first look at the
RULE: written substance of the document (i.e. within the “four corners” of the contract).
• Evidence of a pre-contractual agreements, what the parties said, documents that
preceded or are subsequent to the contract are only relevant if the terms are
ambiguous.
• Applies to written and oral contracts.

Application of PER (PER)


• Parole evidence rule (PER) applies to documents agreed by the parties to be a
memorialization of the contract.

41
o E.g. Documents that have clear terms and are signed by both parties
o Or reasonable evidence showing agreement to unsigned document

1. Did the parties intend for the document to be a memorialization of their


agreement?
• YES
o PER applies. Intention of the parties, terms and meaning should be
interpreted from the “four corners” of the document. No outside evidence
should apply.
• YES, BUT TERMS ARE NOT CLEAR
o PER does not apply then (Southern Resources v Techomin Australia) If
there is ambiguity on the fact of the contract, that is an exception to PER.
Court will look at other factors (Hillas v Arcos) to determine what the
parties intended.
• NO
o PER does not apply when parties did not intend terms of contract to be
enforceable until a condition precedent was met (Pym v Cambell)
Show evidence that the document was never intended to be a
contract until a condition precedent was met (Pym v Campbell)
2. Is there evidence of a collateral contract?
• Can prove there is a second, independent collateral contract (Morgan v Girffiths –
rabbits) and this does not violate PER. Court will look at collateral contracts
(nws “entire agreement clauses”) where collateral contract was 1) intended and
2) does not substantively change the original contract
o Limitation: collateral contracts cannot modify or conflict with the
original contract (Hawrish v Bank of Montreal)
o An “entire agreement” clause allows the court to presume that the
entire agreement exists within the “four corners” of the contract.
This presumption is rebuttable, if the other party can prove a
clear collateral contract (Gallen v All-State Grain)

EXCEPTIONS TO THE PAROLE EVIDENCE RULE (TERMS):


1. Evidence that shows there was no agreed memorialization (Thorne v
Borthwick)
2. Evidence that proves the document is not a contract: The PER only protects
documents that are contracts on their own (mistake, misrep.). You can prove
that a document is not a contract in the first place by submitting evidence to
the fact (outside the four corners of the contract), which demonstrate a
condition precedent must be fulfilled prior to the existence of a contract (Pym
v Campbell)
3. Existence of collateral contract: A verbal collateral contract can always be
proven and written or oral evidence to prove such is not barred by the PER.
The terms of the collateral contract must not conflict with the terms of the
main contract (Morgan v Griffith – rabbits)
• Must prove that the parties intended for the terms outside the document (Heilbut)
• EXCEPTION: a collateral contract offends the PER if the collateral contract
conflicts with the terms of the main contract and cannot be accepted (Hawrish v
Bank of Montreal)
• EXCEPTION: An “entire agreement clause” allows the court to presume that the
entire agreement exists within the “four corners” of the contract. This
presumption is rebuttable, if the other party can prove a clear collateral contract
(Gallen v All-State Grain)

EXCEPTION TO THE PAROLE EVIDENCE RULE (MEANING):

42
1. Ambiguity: If the words of the contract are ambiguous (patently or latently),
then the PER is set aside and the court will permit evidence that clarifies the
terms of the contract (refer to factors in (Hillas; Scammel) (Southern
Resources)
• Latent Ambiguity: There is no obvious inconsistency as words have a clear
definition, but parties claim there is a different meaning to their words. Generally
not an exception to the PER (Southern Resources v Techomin Australia)
• Patent Ambiguity: When there is an obvious inconsistency in the meaning, the
courts will look at what the parties have intended and allow external evidence
(Scammel – Hire Purchase Terms)

NEW LAW: Sattva Capital Corp: MAY RENDER PER OBSOLETE – interpretation
considered ‘in light of the factual matrix’ – knowledge that was reasonably or ought to
have been known by both parties
Ledcor Construction v Northbridge Insurance [2016] SCC – cut back at
SATTVA – interpret factual matrix only if it is a negotiated contract

2. Implied Terms of Contract


Implied Implied terms of K involve the courts adding terms that are not expressly agreed
Term of upon by the parties. It is easier to imply terms in oral or partially written K than a fully
Contract written K. The more formal and complex, the less inclined a court will be to imply
terms.

Before a court will imply terms of a contract, they will have:


• Ensured they are not creating a contract for the parties
• Looked at the certainty of terms
• Applied the Parole Evidence Rule – it is an exception to PER
Implying terms is a LAST RESORT solution and is rarely done.

Three types of implied terms (Different test for each)

1. Terms Implied in FACT


• Should Reflect Intention of parties: Both parties would have included this term in
the K had they turned their minds to it (Codelfa Construction)

5 Step TEST for implied terms (Hard to satisfy) (Codelfa)


1. Terms must be equitable and reasonable (easy to satisfy)
2. The term must be necessary to give business efficacy to the transaction
• No term will be implied if the contract is effective without it. K must not work
without it to apply
3. Term must be so obvious it goes without saying (High Standard)
• However, if it’s not obvious, why is it not in the contract?
4. Term must be capable of clear expression
• Should be able to express the term in a short and clear manner
• If cannot articulate what the term should be then what argument do you have?
5. Term must not conflict with the writing not contradict any express terms of the
contract
• Similar to Hawish v BMO (where oral evidence conflicted and a collateral
contracts was not formed)

2. Implied in LAW (tenancy/employment contract)


Not reflecting intention of parties, but reflecting substance of what is necessary to be
in K of this type. It is a matter of law that certain kinds of K contain certain essential
terms; they are implied by the nature of the K (Liverpool City Council).

43
TEST:
1. Implied term must be necessary to make contract make sense.
2. Implied terms do not have to be obvious, they must be:
• Reasonable and equitable
• Minimum terms for business efficacy
• Capable of clear expression
• Must not contradict any express terms of the contract

3. Implied by statute
• Legislature may imply terms in a K as a matter of law. It is illegal to exclude
operation of these implied terms in the K. (Sales of Goods Act/Consumer Protection
Act)
Sales of Goods Act, RSO 1990, c S 1
(1) Warranty of Title: implied term that seller has good title and ability to
deliver good product
(2) Warranty of Merchantable quality: implied term that seller warrants that
the goods are of “merchantable quality” and will work
(3) Fitness for purpose: Implied that good is fit for purpose
Consumer Protection Act, 2002, SO 2002, c 30
• s 31 – Cannot contract out the implied warranties of the Sale of Goods Act in the
case of a transaction with a consumer (can between two businesses)

3. Duty to Perform in Good Faith

Bhasin v Hrynew: Cannot intentionally mislead in a contract. Have the duty to perform in good faith.
• SCC says there is a general Organizational Principle of the Law of Contracts: parties of the contracts
must perform their contracts in good faith, honestly and not arbitrarily
o Appropriate regard for the legitimate contract interest of the other party (don’t undermine
contractual agreements in bad faith)
o Must not lie or knowingly mislead about contractual performance

General Organization Principle is not:


1) An implied term – therefore cannot be contracted out (but can contract down, with clear words,
not just an entire agreement clause)
2) Isn’t a fiduciary duty: do not need to put other’s interests first; don’t have to forgo advantage, even
if it causes intentional loss to the other party
3) Doesn’t require “positive disclosure”

Obligation to Perform in Good Faith:


1) Termination for cause in an employment contract
2) Termination of franchise right (good faith and with good reason)
3) Payment of an Insurance Benefit (Whitten v Pilot Insurance)
4) Exercise of a contractual discretion (if one side has right in contract to do something at their will, it
has to be exercised for a proper purpose)
5) Must not lie or knowingly mislead about contractual performance

Greater Vancouver Sewage and Drainage Authority v Wastek: Need active dishonesty; can’t just
state you don’t like the outcome, have to look at what bad faith is in terms of what is in the contract
Styles v Alberta Management Corporation (AIMCO): Performance is in bad faith when exercise a
discretion (means of behavior) that is completely contrary to the reasonable expectation of the other
party

4. Exclusion Clauses
Exclusion Any terms that excludes or limits the possibility of getting a normal remedy for breach

44
Clauses of contract. Four types of exclusion clauses:

1. Complete Exclusion of Liability: excludes all liability for breach of K, negligent


performance, or intentional failure to perform (McKutheon v MacBrayne Ltd)
2. Limitation of Liability Clause: in the case of a breach, liability is limited to a certain
amount (Parker v Southeastern Railroad)
3. Exclusion of types of remedy: limits type of remedies that are available for a breach
(i.e. right to bring a litigator, within a certain amount of time (George Mitchell Ltd. v
Finney Lock Seeds; Hunter v Syncrude – time)
4. Exclusion of statutory rights: parties can agree that certain statutes do not apply to
the K
o Subject to legislation (i.e. Consumers Protection Act) that might prevent contracting
out.

In order to enforce an exclusion clause, it must be a part of the contract.


The PER does not preclude one from adducing evidence that the exclusion clause is not
part of the contract. The onus is on the party arguing that the exclusion clause should
not apply (Tercon Contractors Ltd)

1. Was the exclusion clause really a party of the contract?


A. You signed the agreement with the clause
o General: A signature on a contract is conclusive of applicability (L’Estrange v
Graucol)
o The terms must be contemporaneous with the formation of the contract (Olley v
Malborough)
o Exclusion clause is valid if both parties negotiated the clause and make explicit
agreement about who will accept liability. No excuse if you agreed to take on
the risk. (Harbutt’s Plasticine v Wayne Tank)
B. You did not sign the document but assented to it
o If the written contract is a memorialization of agreed-upon terms, the written
contract need not be signed for the exclusion clause to apply. The parole
evidence rule applies (Harris v Great Western Railway Co. – accepting ticket)
C. You did not sign the contract
o Where past dealings between parties show that the exclusion clause on the
usual contract was known to the agreeing party, the contract need not be
signed for the exclusion clause to apply, BUT must be evidence of knowledge of
clause in past dealings (McCutcheon v MacBrayne)
o If the party knows that the document contains conditions, the conditions will
apply even though the party may not know the specific conditions (Harris v
Great North Railroad)
o If the party knows that the document contains writing, but does not read it to
find out that it contains conditions, he is bound by an exclusion therein.
Ignorance is no defence. (Parker v The South Eastern Railway Co.)
o If the party does not know there is writing, whether or not the exclusion clause
applies depends on:
a. Is it reasonable for the person claiming no knowledge to make that claim
credibly given: (1) the nature of the document and (b) the knowledge of the
person?
b. YES Has the person relying on the clause done what is reasonable in the
circumstances to bring the limitation clause to the person’s attention?
(e.g. bold type, different colors etc.)
Yes Exclusion clause is part of the contract.
No Exclusion clause is not part of the contract
Test from Parker v The South Eastern Railway Co; applied in Union
Steamships v Barnes)

45
D. If a signing party is under a disability and the issuer knows of the disability, the
issuer is under an obligation to take reasonable steps to bring the conditions to the
signing party’s attention (Thompson v London, Midland and Scottish Ry Co.)
E. Exception: Exclusion clause is not contemporaneous with K (Olley)
• Can I prove that the exclusion clause was not part of the K when I signed or assented
to it? (Olley)

Assuming the exclusion clause was part of the contract, what interpretational
techniques should be used?

2. Interpreting Exclusion Clauses: does it apply to the circumstances?


1. As a matter of interpretation, does the clause apply to the circumstances established
in evidence (Tercon Contractors v BC – Most recent SCC)
• Construe the statute very strictly (contra proferentum) against the party
relying on it to see if it applies to the breach
• Before you can rely on the exclusion clause, the damage that has occurred has
to be caught by the exclusion clause
2. If the damage is included in the exclusion clause: two methods of interpretation:
(follow method two)

METHOD ONE: HL
1. Construe the clause in the context of the contract as a whole (Photo Production Ltd v
Securicor Transport Ltd)
2. Interpretation of an exclusion clause will involve regarding the contract in its entirety
to determine what the exclusion clause was intended to limit and under what
circumstances
3. Factors considered:
• Nature of the contract
• How the contract is supposed to work
• Risks that appear to be assumed
• Rewards of the parties

3. METHOD TWO: (Test from Tercon Contractors v BC)


1. If damage is included in the exclusion clause, was it fair and reasonable or
unconscionable at the time of the contract? (George Mitchell Ltd v Finney Lock Seeds)
2. In determining whether the clause is fair, consider (George Mitchell, Denning’s factors)
• What is the nature of the contract?
o Standard form contracts are less likely to be deemed fair, while
negotiated contracts are more likely to be deemed fair
• Whether there was equality of bargaining power
o Standard form implies an inequality of bargaining power, while
negotiated contracts imply an equality of bargaining power
• A consideration of the risks and rewards of the contract
o A smaller payment may entail a smaller assumption of risk (Photo
Production Ltd v Securicor)
o A smaller payment may imply that the exclusion clause is fair
o A smaller risk may imply that the exclusion clause is fair
• The nature and circumstances of the breach
o Where the breach is intentional, the clause is less likely to be
applicable
BINNIE (minority): thinks there should be a more narrow unconscionable test:
• Was there a grossly unfair bargain?
• Was there inequality of bargaining power?
• Did someone take unfair advantage?

46
• No legal advice?
• All of these conditions must be met to not have exclusion clause apply
3. If it applies and is fair/equitable, should the court nonetheless refuse enforcement of
the exclusion clause based on an overriding issue of public policy? (Burden of Proof:
on party seeking to avoid enforcement of the clause (Tercon Contractors v BC)
Traditional Denning Approach:
Denning’s Rule of Fundamental Breach: If a party has fundamentally breached a K,
that party cannot rely on the exclusion clause (Karsales Harow v Wallace)
• A breach of a condition that does to the root of the consideration
Reid’s Rule of Construction: Construe the exclusion clause in context of the contract as
a whole, if determine that parties bargained for it to apply in exact situation – then it
applies (Suisse Atlantique, Photo Production)
• Overruled Denning’s theory of fundamental breach
• DENNING took HL’s ruling in Suisse and Photo and interpreted it in a different way,
creating the modern approach above

Interpreting Written Contracts – Parole Evidence Rule


Parole Evidence Rule “Four Corners Rule”
• Parole evidence rule precludes any admission of evidence outside the words of the written
contract that would add to, subtract from, vary or contradict a contract that has been wholly
reduced to writing

Pym v Campbell [1856]


Facts: Inventor (Pym), 2 engineers (ensuring the product works) and the purchaser (Campbell) set
up a meeting to solidify selling the invention to Campbell. Pym arrived late and the two engineers
had left, it was agreed if they could find the two engineers, the deal might be made. Only one was
found and agreed – they drew up a paper that both Campbell and Pym signed and it was agreed if
Abernithie approved the invention the paper agreement is valid. Abernithie did not approve. Trial
judge for the defendant – could not operate as an agreement until Aberithie agreed.
Issue: Did a contract form with a written document signed by only parts of the agreeing group?
Holding: Decision for the defendant, not an agreement.
Reasons: ERLE J:
• Of the view that it was never meant to be an agreement at all. They were to sign the
memorandum of terms for convenience, yet it was not an agreement until Abernethie was
consulted.
Ratio:
• A paper that is signed with the expressed intention that it should not be an agreement, the other
party cannot fix it as an agreement upon those so signing.
• Evidence to vary the terms of an agreement in writing is not admissible but the evidence to show
that there is not an agreement at all is admissible.

Morgan v Griffith [1871]


Facts: The Plaintiff became tenant to defendant in 1867, on oral terms that included the signing of a
lease. The plaintiff discovered that the land was overrun with rabbits and when the lease was
presented demanded that they be destroyed in order to sign, defendant promised to destroy the
rabbits. 1868, the lease was again tendered – plaintiff asked to have the promise incorporated into
the lease, which Defendant refused, although he repeated his promise – on this the plaintiff signed.
Plaintiff agreed not to destroy the rabbits and allow the defendant to hunt. Rabbits were not
destroyed by Michaelmas 1870. Plaintiff brought an action and the defendant pleaded the parole

47
evidence rule. Trial judge admitted oral evidence of his promise and the plaintiff got a verdict.
Issue: Although not in the lease, is the oral promise considered part of the agreement?
Holding: Yes, the verbal agreement was entirely collateral to the lease.
Reasons: KELLY CB:
• Was founded on good consideration, the plaintiff unless promised the destroying of the rabbits,
would not have signed the lease
Ratio:
• When there is nothing in the lease that negatives the collateral contract in the lease – the
collateral contract can be enforced.
• As long as the collateral contract being alleged doesn’t conflict with the contract in writing.

Hawrish v Bank of Montreal [1969] SCR


Facts: Hawrish signed guarantee on a bank loan for company. Agreed with bank manager it would
only be for up to $6000 amount until they got the guarantee from directors and it was for present
debt, afterwards, the Directors of company would take over the debt. Problem is the guarantee says
that it is for present and future debt and it will continue to exist until debt is repaid. The bank got the
guarantee from the directors, however they go bankrupt. The bank goes after Hawrish for the money;
he claims there is a collateral contract
Issue: Can the oral evidence be used as a collateral contract – that the bank was going to get the
money from the directors?
Holding: Appeal dismissed – the collateral agreement (oral) cannot stand as it clearly contradicts the
terms in the written contract.
Reasons: JUDSON JA:
• A collateral agreement – a promise to do something to enter into a contract – another form of
consumer protection. The guarantee was to be immediately effective. The oral evidence is in
complete contradiction of the written document. There must be a clear intention to create a
binding contract – can’t claim collateral contract in conflict with written guarantee.
Ratio:
• There must be clear intent that the parties are creating a separate agreement (collateral
agreement). A collateral agreement cannot be established where it is inconsistent with or
contradicts the written agreement.

Southern Resources Ltd v Techomin Australia [1990] Supp 21


Facts: In a signed deed relating to mineral exploration and production, it was stated that the
respondent was entitled to receive payment of three percent net smelter returns (NSR) in respect of
the minerals produced from an area known as the Blue Development. Appellant is claiming that the
respondent is not entitled to be paid a three percent NSR in respect of all minerals produced from the
Blue Development Area but is entitled to be paid 3 per cent NSR only in respect of minerals produced
by the use of geophysical technique ONLY and not including gold.
Adding these two details on to the contract, that wasn’t stated within the contract – if appellant can
prove that the agreement is ambiguous, can throw away parole evidence rule and get behind the
contract.
Issue: Is this unstated condition to the signed deed valid?
Holding: Appeal should be dismissed
Reasons: BRINSDEN J:
• Appellant is just trying to explain the meaning of the phrase three per cent NSR by varying the deed
by adding to it an additional provision that the respondent is only able to enjoy its entitlement if
minerals have been produced by the use of geophysical technique
• “It proposes to attach a quality or condition to the phrase three percent NSR” and not to identify it

48
• An exception to PER is AMBIGUITY – if the words in the contract are ambiguous
• Court says that there is no ambiguity in the written contract – the contract must be interpreted as
the words in the document
Ratio:
• Cannot attach an unstated or undetermined addition/meaning to a signed agreement

Gallen v Allstate Grain Co [1984]


If there is a real agreement outside the contract that does not contradict the written contract and the
only thing standing in the way is the “Entire Agreement Clause” than a collateral agreement can be
argued and can argue Morgan v Griffin

Sattva Capital v Creston Moly Corp. [2014] SCC


Facts: Plaintiff and Defendant agree that D will pay a finder’s fee of 1.5 US million to the acquisition
of a molybdenum mine in Mexico. The fee would be payable in shares of the defendant. A dispute
arose about the date for determination and thus, the number of shares to be issued to the plaintiff
(the date between writing and signing of document let the share value rise, negotiator wanted shares
that were worth more). Plaintiff argued for a date set out in the definition of “market price” in the
agreement and thus should be 11.5 million shares. Defendant argued that the share price should be
determined to the date elected by the plaintiff as the “maximum amount” of the finder’s fee, couldn’t
exceed US 11.5 million – meaning the P should get 2.5 million shares. Arbitrator wanted to look
behind the contract to determine its meaning.
Issue: Did the arbitrator commit an appealable error of law in looking at extrinsic evidence of the
intentions of the parties in interpreting the contract?
Holding: The Parole evidence rule does not apply to preclude evidence-surrounding circumstances
when interpreting the words of a written contract.
• No, the arbitrator did not make an error of law – the arbitrator should be allowed to look at the
surrounding circumstances.
Reasons: ROTHSTEIN J
• Consideration of the surrounding circumstances recognizes that ascertaining contractual
intention can be difficult when looking at words on their own
• Looking at surrounding circumstances cannot be used to deviate from the text such that the court
effectively creates a new agreement
• It is about seeing something that reasonably ought to have been within the common knowledge of
the parties at the time of execution of the contract
• Parole evidence rule precludes any admission of evidence outside the words of the written
contract that would add to, subtract from, vary or contradict a contract that has been wholly
reduced to writing
• Surrounding evidence is not precluded in the parole evidence rule because they are facts that are
reasonably known to both parties before the date of contracting
• Cannot let surrounding circumstances overwhelm the written contract – must always be
grounded within the words of the contract
• When interpreting the words of the contract – should interpret the words considering the factual
matrix at the time of making the contract
Ratio:
• The surrounding circumstances can be used to interpret the terms of a contract, as long as they
don’t overwhelm the words of the agreement – Factual Matrix
Notes Forbes: SCC made a mess here: stated can look at factual matrix when interpreting a contract
without ambiguity.
• Look at terms of a contract ONLY/SOLELY when it is a non-negotiated contract
• If it is a negotiated contract, the surrounding facts may be relevant

49
• Ledcor Construction v Northbridge Insurance [2016] SCC – cut back at SATTVA – interpret
factual matrix only if it is a negotiated contract

Implied Terms of Contract


When are we going to allow people to imply terms into a contract? Where does it not offend parole
evidence rule and we are not making bargains for the parties?

1) Terms implied in fact: (reflects actual intention – terms that they HAD to have wanted in
the contract without stating it, common knowledge)

Codelfa Construction v State Rail Authority [1982] High Ct of Aus


Facts: State Rail Authority (respondent) engaged Codelfa (appellant) to excavate tunnels, within 130
weeks from March 7, 1972 onwards, 3 shifts per day. Local residents obtained an injunction – no
work on Sundays or between 10 pm and 6 am. Rail Authority refused to meet claim of increased costs
because there was nothing of the sort in the contract. Dispute was submitted to arbitration.
Arbitration found that Respondent should grant reasonable extension of time as well as indemnify
Codelfa for any additional costs and expenses reasonably incurred. Codelfa appealed to the High
Court against some of the answers given to other questions raised in the case and Authority appealed
against the decision to imply a term into the contract.
Issue: Can a term not in the contract be implied into the contract?
Holding: They lost, no implied term.
Reasons: MASON, BRENNAN JJ
• The court holds that the term is not necessary to the contract and to its business efficacy. P took a
risk that was inherent to the contract itself. The bidding process implied this risk. The term,
therefore, is not so obvious as to go without saying. The contract is further not particularly capable
of clear expression. Lastly, if the contract is a fixed-price contract, then altering the price or
implying a term that alters that price conflicts with the writing of the contract.
Ratio: In order to imply a term into a contract, it must meet the following conditions:
1. Implication of the term must be reasonable and fair.
2. The term must be necessary to give business efficacy to the contract, such that the
contract would not function without the implied term
3. The term must be so obvious, as to “go without saying”
4. The term must be capable of clear expression
5. The implied term cannot conflict with the writing of the contract
Codelfa fails on #2, 3, 5

2) Terms implied in law (Is this a term that the law implies into contract, presumed
intention from the nature of the contract) Narrow

Liverpool City Council v Irwin [1976] HL


Facts: Maisonette lent to the Defendants contained no covenants on the part of the council. Within 18
months of living there, the tower was no longer livable- no elevator, garbage in staircase, no lights in
staircase. They claimed that the landlords breached the covenant for quiet enjoyment and for repair.
Court judge made an order for possession but gave the defendant’s damages, the council appealed
Issue: What obligations do the landlords have to the implied easements of the tenants?
Holding: Appeal must be dismissed. Judgment for D.
Reasons: WILBERFORCE
• To imply an absolute obligation to repair would go beyond what is necessary legal incident and
would indeed be unreasonable.
• An obligation to take reasonable care to keep in reasonable repair and usability is what fits the

50
requirements of the case.
• Such a definition involves recognition that the tenants themselves have responsibilities – what is
reasonable to expect of the landlord has a clear relation to what a reasonable set of tenants
should do for themselves
• Court says you have to keep reasonable care to keep the facility in a reasonable repair – this is the
term that is necessary and is implied as a reason of law
• The lease is so void of terms that it is not functional without implied terms. Court can imply terms
to make the contract work – give it business efficacy
Ratio:
• A landlord’s obligation in unwritten covenants can only be implied as a necessity and must be
reasonable in nature.
• Courts saying that this the minimum that is required to keep the contract in necessary business
efficacy, even if the parties did not intend any such terms

3) Implied by statute (Sales of Goods Act)

Sale of Goods Act, 1990


The Sale of Goods Act implies three warranties on the sale of goods:
1. WARRANTY of TITLE: In transactions of the sale of goods, warranties of title are implied,
seller has the right to sell
2. WARRANTY of MERCHANTABLE QUALITY: If you get a sale of goods, the property you get
must be a piece of good property – implied warranty that the good will fit the purpose
3. WARRANTY of FITNESS FOR PURPOSE: If a storeowner gives a purchaser an item in
response to the purchasers demands, then there is an implied warranty that the product is
fit for the purpose
• “This is what you need”
Consumer Protection Act: Any attempt to negative the implied warranties from the Sale of Goods Act
through an exclusion clause in a consumer transaction is void.

Duty to Perform in Good Faith

Bhasin v Hrynew and Heritage Educational Funds


Facts: Bhasin (P) was the sale of an educational savings plan for the defendant, Heritage – they had
an agreement for automatic renewal of the contract for successive three year terms, unless one of the
parties gave 6 months written notice of termination. Hrynew was also a plan marketer under a
similar contract and the Fund’s best salesman, he wanted Bhasin’s territory and proposed a merger.
Bhasin declined. Heritage told Bhasin that Hrynew was required to audit Bhasin – which was untrue.
Bhasin was ultimately not renewed, he lost business and he brought an action against Heritage for
breach of contract and Hrynew for the tort of inducing breach of contract.
First trial it was decided that there was an implied term of the contract that decisions about renewal
would be made in good faith and that Hrynew had induced the breach. ABCA held that trial judge had
erred in implying a term of good faith as to renewal, implied in law, applying a term that doesn’t pass
the Imply Term Test. Bhasin sought SCC appeal.
Issue: Was there an implied term of good faith in the contract that was breached? Can a new duty for
good faith be created?
Holding: Heritage was found to be liable for breach of contractual duty to perform in good faith.
Reasons: CROMWELL J:
• The organizing principle that parties generally must perform their contractual duties honestly and
reasonably and not capriciously or arbitrarily
• Good faith performance does not engage duties of loyalty to the other contracting party or a duty to
put the interests of the other contracting party first

51
• Common law places great weight on the freedom of contracting parties to pursue their individual
self interest – in the legitimate pursuit of economic self interest
• The duty of honesty in contractual performance is a general doctrine of contract law that applies to
all contracts – the parties are not free to exclude it
Ratio:
F. There is a general duty of honesty in contractual performance. Parties must not lie or
otherwise knowingly mislead each other about matters directly linked to the performance of the
contract. This does not impose a duty of loyalty or of disclosure or require a party to forego
advantages from flowing from the contract
G. SCC says there is a general Organizational Principle of the Law of Contracts: parties of the
contracts must perform their contracts in good faith, honestly and not arbitrarily
o Appropriate regard for the legitimate contract interest of the other party (don’t undermine
contractual agreements in bad faith)
o Must not lie or knowingly mislead about contractual performance
General Organization Principle is not:
1) An implied term – therefore cannot be contracted out (but can contract down, with clear words,
not just an entire agreement clause)
2) Isn’t a fiduciary duty: do not need to put other’s interests first; don’t have to forgo advantage, even
if it causes intentional loss to the other party
3) Doesn’t require “positive disclosure”
Obligation to Perform in Good Faith:
1) Termination for cause in an employment contract
2) Termination of franchise right (good faith and with good reason)
3) Payment of an Insurance Benefit (Whitten v Pilot Insurance)
4) Exercise of a contractual discretion (if one side has right in contract to do something at their will, it
has to be exercised for a proper purpose)
5) Must not lie or knowingly mislead about contractual performance

Exclusion Clauses

L’Estrange v Graucob [1934] KB


Ratio: If you sign a contract, you cannot claim ignorance of exclusion clause

Karsales (Harrow) v Wallace [1954] ENCA


Facts: Bought a plunker of a car, exclusion clause says they are only liable for a certain number of
dollars
Denning: Rule of Law (Fundamental Breach): Exclusion clause does not apply where there is a
fundamental breach of contract

Swisse Altlantique v NV Rotterdamsche (Photo Production v Securicor) 1967 HL


BAD LAW
Decision: House of Lords say when you look at an exclusion clause, Rule of Interpretation, Strict
Construction: look at the contracts as a whole, how was the clause meant to apply in the specific
circumstances that have arisen; Look at:
• Risk Reward, Intention (was there an intentional breach), Equality of Bargaining Power, Standard
Form of Contract, was the exclusion clause meant to apply
• Is it fair and reasonable to enforce the exclusion clause
• Denning keeps getting struck down: Denning believes – Rule of Law that exclusion clause never
applies if contract isn’t performed in its fundamental aspects.
• If there is a material breach of contract, a party should be able to set aside the exclusion clause
aside and sue on breach of contract. This provides

52
McCutcheon v MacBrayne Ltd. [1964] HL
Facts: Appellants had used the respondents’ service (MacBrayne) to carry their car on their carriers
(ferry) – they had signed a long written document with conditions all three times beforehand. The
document was long and Appellants had admitted to never reading the full contract. The date at issue,
Oct. 8th, 1960, the Respondents’ boat negligently hit a rock and sank, carrying the appellants livestock
and they had forgotten to get P to sign the contract
Issue: Can it be stated that the previous contracts they had been reflective of the current contract –
meaning that the Respondents were not negligent for P’s loss?
Did the Respondent have an estoppel in the expected contract?
Holding: Appeal should be allowed. They must abide by the contract they made – which is no
contract at all.
Reasons: LORD DELVIN:
• There can be no conditions in any contract unless they are brought into it by expression.
Incorporation or implication. They are not brought into it simply because one party has inserted
them into similar transactions.
• If haven’t signed – must look at past dealings and determine if past dealings realistically created no
knowledge of the exclusion clause
Ratio: You must abide by the contract that you made
• Previous dealings are only relevant if they show knowledge of the terms
• If can say from previous dealings – that the knowledge of the exclusion clause existed – then you
can apply exclusion clause

Harris v Great North Railroad


Facts: Where someone gives you a document and it’s a receipt or a ticket of some sort that has
conditions on it that says there is an exclusion clause. Person puts ticket in pocket and doesn’t read it.
Issue: Are they bound by the clause?
Ratio: If you put a document in your pocket and you KNEW THAT IT CONTAINED CONDITIONS, then
the court considers that you have assented to the conditions as though you signed it. Allegations of
ignorance about the content of the conditions are not a defence.

Parker v South Eastern RY [1877] CA


Facts: Parker and Gabell checked their luggage on a train. They were given tickets with a number on
one side, and small print on the other side. The small print stated that the railway would not be
responsible for bags lost worth more than 10 pounds. Both respondents had received the tickets
before but had never read the small print. They both lost their bags and brought actions against
South Eastern for the value of the bags and their contents. Both were greater than 10 pounds. In the
lower courts both respondents were successful, this appeal by South Eastern.
Issue: Are the respondents bound by the terms on the back of the tickets?
Holding: Appeal allowed; new trial ordered.
Reasons: MELLISH:
• There is no definite law in situations like this, for it depends on the specific circumstances
• If the person receiving the ticket does not know that there is writing on the back of the ticket, then
he cannot be bound by its conditions.
• However, if he knew that there was writing and he either neglected to read it or read it and did not
think that it contained conditions of the contract, then he is bound by its terms, as long as the ticket
was delivered to him in a manner that gave him reasonable notice that there were conditions on it
• Thus, the judge states that a new trial must be awarded and the jury must determine if there was a
reasonable notice that the writing contained conditions
BRAMWELL:
• If the P sees the writing and either does not read it, or reads it and does not object, he must be held

53
to consent to its terms and be bound
• He states that this is a question of law and therefore is not for a jury to decide – believes verdict
must go to South Eastern here
Ratio:
• If a P does not see writing that contains “conditions” of the contract and no reasonable effort was
made to ensure he was aware of it, then he is not bound by its terms; if he does see if and either
does not read it, or does not think that it contains conditions, then he will be bound by its terms so
long as the defendant delivered it in a manner that gave him reasonable notice that there were
conditions on the ticket.
TEST:
1) If party knows document contains writing but does not read it to find out the conditions, he is
bound by any exclusion therein. It is no defence to allege ignorance.
2) If the party does not know there is writing, whether or not the exclusion clause applies depends
on:
• Is it reasonable for person claiming no knowledge to make that claim credibly given a) the nature of
the document and b) the knowledge of the person?
• If yes, has the person relying on the clause done what is reasonable in the circumstances to bring
the limitation clause to the person’s attention? (i.e. Bold type, different colors). If they have,
exclusion clause is part of the contract. If they failed to do so, exclusion clause is not part of the
contract.

Thomson v London, Midland and Scottish Railway Co [1930] KB


Ratio: If you are aware of the person’s disability, then you have to do what you can to bring the
exclusion clause to the attention of the individual. If you are not aware of the person’s disability, then
question is whether the individual did everything reasonable to bring the exclusion clause to the
attention of the person entering the contract.
Facts: Suppose the ticket holder is blind, does not speak English, or cannot read?

Union Steamship v Barnes [1956] SCR


Facts: Barnes lived in Hamlet not accessible by road. Only way to get out was Union Steam Ship’s
boat. They sold him a ticket in the dark, he didn’t read it and then fell through a hole and broke both
his legs.
Holding: Judgment for Union Steamship
Reasons: The legend was on ticket in bold face. There was no extra caution required to tell Barnes
about the limitations of the ticket bc they did meet the reasonableness standard.
Ratio: APPLICATION OF THE Parker v South Eastern Ry Co TEST:
• As long as you have taken reasonable precautions to bring exclusion clause to the attention of
person entering the contract, they will apply.

Olley v Malbourough Court [1949] KB


Facts: Olley’s arrived at a hotel and agree to price. In the room, there is a disclaimer saying the hotel
is not liable for any goods lost or stolen. Ms. Olley’s fur coat goes missing.
Holding: Judgment for P. Hotel liable for lost coat.
Reasons: Exclusion was not part of the contract at the time it was made and can’t be added in after.
Ratio: Exclusion clause must be made available to you prior to the formation of the contract. Must be
contemporaneous.

Harbutt’s Plasticine Co v Wayne Tank [1970] EN CA


Facts: Harbutt’s wants to upgrade plasticine factory with new machine. They agreed to take on risk
with their insurance of there being a problem in the performance of the contract. Wayne Tank is
excluded from any liability under the performance of the contract including their own negligence.
Fire started and factory burned down.

54
Holding: Judgment for Wayne Tank
Reasons: DENNING:
• The exclusion clause in this case is valid because both the parties negotiated the clause and
agreed about who would accept liability.
Ratio: An exclusion clause is valid when both parties negotiate and make an explicit agreement
about who will accept liability.

Photo Production Ltd. v Securicor Transport Ltd. [1980] HL


Facts: Photo Production Ltd (P) sued Securicor Transport Ltd (D) after Securicor’s employee started
a fire at P’s factory to warm himself while at work and accidentally burnt it down. D argued that an
exclusion clause in its contract meant that they were not liable. P argued that the clause could not
apply under the doctrine of fundamental breach, that the breach of contract went to the root of the
contract and invalidated the whole agreement, and extinguished the exclusion clause.
Issue: Does the exclusion clause apply in this circumstance?
Holding: Yes, the exclusion clause does apply in this circumstance. Judgment for D.
Reaons: HL – Denning’s rule of law fundamental breach is not the law and not the way to
interpret the exclusion clause. Interpretation of an exclusion clause will involve looking at the
contract in its entirety to determine what the exclusion clause was intended to limit and under what
circumstances, and if it applies to this breach (factors considered: the nature of the contract, how the
contract is supposed to work, the risks that appear to be assumed, and the rewards of the parties).
Here, contract was for a small amount and it’s not unreasonable that the D shouldn’t be subject to
huge liabilities. Look at clause in light of whole.
Ratio: Interpretation of an exclusion clause will involve regarding the contract in its entirety to
determine what the exclusion clause was intended to limit and under what circumstances (factors
considered: the nature of the contract, how the contract is supposed to work, the risks that appear to
be assumed and the rewards of the parties).

George Mitchell Ltd. v Finney Lock Seeds Ltd. [1983] HL


Facts: D (Finney Lock Seeds) sold P (George Mitchell) 30 lbs. of defective cabbage seed. Sixty-three
acres of crop failed and P sued for loss of production. The delivery came with an invoice, which
outlined two limits: (1) liability was limited to any defective seeds sold (2) Liability excluded for
loss/damage/consequential loss/damage from use of seed
Issue: If the exclusion clause applies, was it fair, reasonable and not unconscionable at the time the
contract was made?
Ratio: DENNING:
1. In determining whether the clause is fair and not unconscionable (Denning’s Factors):
i. Whether the contract was in standard form or negotiated
• Standard form contracts are less likely to be deemed fair, while negotiated contracts are
more likely to be deemed fair
ii. Whether there was equality of bargaining power
• Standard form implies an inequality of bargaining power, while negotiated contracts imply
an equality of bargaining power
iii. A consideration of the risks and rewards of the contract
• A smaller payment may entail a smaller assumption of the risk (Photo Production Ltd v
Securicor)
• A smaller payment may imply that the exclusion clause is fair
• A smaller risk may imply that the exclusion clause is fair
iv. The nature and circumstances of the breach
• Where the breach is intentional, the clause is less likely to be applicable

55
Hunter v Engineering v Syncrude Canada [1989] SCR
Case for the Death of Fundamental Breach
Fats: Gear Boxes ordered, the defects were discovered after warranty had lapsed. Have to claim
within 24 hours if think the product was defective; didn’t find out in time but didn’t work at all.
Breach of the implied term of working goods in the sale of goods/consumer act and that the
exclusion clause should not work – it is a fundamental breach
Reasons: DICKSON:
• No fundamental breach, it should be abandoned.
• Unconscionability doctrine: Ask agreeing with DENNING: Is it unconscionable to rely on the
exclusion clause? (George Mitchell)
WILSON:
• Adopt strict construction (Photo Production v Securicor) or adopt a reasonableness approach.
Construing the exclusion clause to the circumstances as a whole.
• Fundamental breach should be available as a residual mechanism
Ratio: Fundamental Breach is NOT THE LAW IN CANADA (but lower courts still apply it. Adopt
DENNING articulation of exclusion clauses (George Mitchell v Finney Lock Seed)

Tercon Contractors v British Columbia [2010] SCC – Most recent Canada


Goes with Denning from George Mitchell – Unconscionable test
Facts: BC Ministry of Transportation issued request for proposals for building new highways. Tercon
submitted a bid as did Brentwood The RFP included a broadly worded exclusion clause excluding
liability on the part of the Province for any damage claims arising “as a result of participating in this
RFP.” Brentwood sent submission stating that it wanted to join bid with another company, which was
ineligible for bidding process. Gov’t never responded and ultimately came down to Tercon and
Brentwood. Brentwood was selected and Tercon sued the Ministry for damages on the basis that
acceptance of an ineligible bid constituted a breach of contract and that the breach had cost it an
award of the project.
Issue: Does the exclusion clause bar a claim for damages for breach of the tendering contract?
Holding: (5/4) No. Exclusion clause does not apply. It included circumstances that were different
than the breach.
Fundamental breach is dead.
Reasons: CROMWELL
• SCC says construe the clause very strictly. The exclusion clause did not cover the damages being
sought by Tercon (i.e. damages for allowing an unqualified person to bid).
• Cromwell “laid to rest” the doctrine of fundamental breach (no explanation given)
• “Rule of construction” doesn’t appear to apply, it is irrelevant
• We determine from looking at George Mitchell v Finney Lock Seeds
BINNIE: minority Proposed Test: Simply ask if it is unconscionable
• The only time you can’t rely on the exclusion clause is if it is unconscionable in basic sense:
o Was there a grossly unfair bargain? Was there inequality of bargaining power? Did someone
take unfair advantage? No legal advice? All of these conditions need to be met
• More courts are applying the Binnie decision- going to be unusual to not meet this test – may be in a
cold winter in contracts again
Ratio: Test:
Use the following steps to interpret exclusion clause:
1. As a matter of interpretation, does the clause apply to the circumstances established in evidence?
• Courts construe the statute very strictly (contra proferentum) against the party relying on it to
see if it applies to the breach.
2. Unconscionable? If it applies, was it unconscionable at the time the contract was made (i.e. unfair)
(courts chose to apply Mitchell test)
3. Public Policy: If it applies and is valid, should the court nonetheless refuse enforcement based on an

56
overriding issue of public policy? (Burden of Proof: on the party seeking to avoid the enforcement
clause, i.e. Poison baby formula with exclusion clause)
Courts have been applying the Binnie test more often – Simply ask – is it unconscionable.

Defects In Contractual Relations


What are the things that can cause an otherwise good contract to fail/be voided/unenforceable?

1. Misrepresentation
Types of Misrepresentation renders a contract voidable (subject to bars), not void.
Misrepresentation 1. Fraudulent Misrepresentation (Tort of Deceit)
• Statement is known to be wrong, or made recklessly, without any care as
to whether it is true (Redgrave v Hurd)
• Remedy is rescission (subject to bars of rescission), or tort damages in
fraud (Redgrave v Hurd – solicitor partner)
2. Negligent Misrepresentation (Tort Damages)
(1) Person having a special knowledge or skill, (2) Gives advice in the area of
expertise, (3) To an individual or group of which P is a member, (4) Knowing
that the advice will be relied and acted upon, or should have known owed
a duty of care (Hedley Byrne)
• Remedy is rescission or tort damages for negligent misrepresentation
(Esso Petroleum v Marden)
3. Innocent Misrepresentation (No Tort)
• A representation that is neither fraudulent or negligent (O’Flaherty v
McKindley)
• Remedy is rescission (subject to bars of rescission), but no tort damages
(Heilbut Symons & Co)
• Can argue that the representation be converted into a term of collateral
contract, in which case, the P can claim contract damages (Leaf v
International Galleries – Denning)
o If a representation is made in the course of dealings for a K for the
very purpose of inducing the other party to act on it, and it actually
induces them to act by entering into K, that is prima facie ground for
inferring a collateral K
Elements of an 1. Misrepresentation must be material
Actionable Mis- • Ask: Is it an inducing factor to the contract?
representation o The representation must be material in that it induced the
formation of the contract; allegations that the Plaintiff failed to do
his due diligence is no defence. (Redgrave v Hurd)
• The misrepresentation does not need to be the only factor inducing the
other party to enter into the contract, it just needs to be an inducing
factor (Redgrave v Hurd)
• TEST: Would the misrepresentation have induced a reasonable person to
enter into the contract?
o Yes Then presumption is that misrep induced that person.
Presumption will apply unless representor can show that the
plaintiff 1) knew the falsehood or 2) Clearly didn’t rely on the
representation (Redgrave v Hurd)

2. It must be a positive misrepresentation (Actually stated)


• Failure to speak (i.e. silence) is normally not a misrepresentation
• Exceptions:
o A Fiduciary Relationship (trustee, director)

57
o In a contract of utmost good faith (insurance), contractor has
positive duty to represent any factor that the contracted may need
to know
o Contract of Issuance of Shares

3. Misrepresentations must have been false when acted upon.

4. Representation must be one of fact, not opinion


• A representation – that induces an individual to enter into a contract –
can be held as a warranty sounding in damages, if that representation is
one of fact and not opinion (Bisset v Wilkinson)
• A statement of law by a non-lawyer is an opinion. Presumption is that
you don’t know the law. Statement of law doesn’t general result in a
misrepresentation unless given by a lawyer.
• Statement of fact – if you possess expert knowledge – it is fact and not
opinion (Esso Petroleum)

Remedies for Misrepresentation


• Condition: part of contract, substance of consideration; remedy is rescission
• Warranty: lesser term, affirmation of certain facts; remedy is damages
Rescission: Definition: A contract may be voidable subject to the equitable doctrine of
rescission, which says the innocent party can treat the contract as though it’s at
an end. This is subject to bars of rescission, which creates a loss of the right to
rescission.

GENERAL RULE: The remedy for misrepresentation is rescission (subject to


damages exception below).
• In the case of fraudulent misrepresentation the bars will not work as
aggressively because courts do not want to compensate fraud.
• Exception: Mistake which renders contract void to begin with – no property
passed.

Bars to rescission:
1. Fully executed contract: A fully executed contract is a bar to rescission in
the case of innocent misrepresentation, unless it renders the subject of the
sale different than what was contracted for (Redican v Nesbitt)
2. Inability to make restitution: Rescission cannot be granted unless the
parties can be restored substantially to their original positions. Rescission
will be available if one of the parties can grant substantial restitution
(O’Flaherty v McKinley)
• Where complete restitution cannot be granted, then the court allows part
restitution and part compensation
3. Intervening third party rights: An intervening third party in the property
(e.g. car was already sold to someone else, so can’t make restitution) removes
a right to rescission (O’Flaherty v McKinley)
• Exception: if the third party is not innocent, they do not have good title
4. Fully executed contract for a conveyance of land (Redican v Nesbitt): Once
you close a land deal and register the deed, you lose your right to pursue
rescission remedy for any representations. Before buying land, the contracted
needs to perform his due diligence in inspecting land. There must be finality
in a transaction of land.
• Exception: If the misrepresentation was fraudulent, then the individual
may still be able to sue for rescission in contract (Regrave v Hurd)
5. After a reasonable period of time has passed (sale of personal property

58
only): An individual loses their right to rescission if the individual does not
exercise this right within a reasonable period of time, in which the individual
has the opportunity to investigate. This occurs in the case of innocent
misrepresentation. (Leaf v International Galleries)
6. Affirmation: A decision to keep the goods, express or implied despite
knowledge of misrepresentation (Have to move on it)
Damages • Damages is the remedy when rescission is not possible or appropriate. You
must convert the misrepresentation into a term of a collateral contract.
o E.g. “in consideration of me entering this contract, you represent the
painting as a Constable.” Can try to do this unless the terms of the
collateral contract conflict with the original contract.
o You will have to convince the court that there are two contracts,
which may be difficult because they may ask why you didn’t just put
all the terms into one contract.
• A collateral contract is a means of obtaining damages for innocent
misrepresentation.
• In order to claim damages on a collateral contract, must have:

1. Real Intention:
• Plaintiff must show that the parties had a real intention to create two
contracts (Heilbut Symons & Co v Buckleton)
2. No conflict with main contract
• The collateral contract must not contradict (Hawish v Bank of Montreal)
Easier to prove if a third party is the representor: In a specific situation, a
third party to a contract can sue for fraudulent misrepresentation through a
collateral contract. A collateral contract can exist where A represents to B that
they should get C to use a specific product that A sells. In consideration of A
getting C to use B’s product, B represents that the product will be effective
(Shanklin Pier v Detel Product)

Misrepresentation:
Varieties of Misrepresentation

Redgrave v Hurd [1881] UK – Fraudulent Misrepresentation


Facts: Mr. Redgrave, an elderly solicitor advertised for a partner to join the business and buy the
accompanying house. He told Hurd that the practice brought in 300 pounds income when it was only
200 pounds. He said Hurd could check records, but he didn’t (in fact showed no business). Mr. Hurd
did not inspect the papers until he realized the truth just before completion of the agreement; he
agreed to buy half interest in the practice and a house. He had signed the contract but refused to go
through. Mr. Redgrave sued for specific performance and Mr. Hurd counterclaimed for rescission
based on fraudulent misrepresentation.
Issue: Should the contract be rescinded on the grounds of fraudulent misrepresentation?
Parole Evidence Rule Issue – nothing in contract referencing the revenue amount of the legal practice
Holding: Yes, fraudulent misrepresentation. Contract is voidable and set aside. No damages, the D
didn’t plead that P knew it was fraud, but D was freed of the contract/
Reasons: The contract should be rescinded on these grounds. An individual who induces another to
enter into a contract through fraudulent misrepresentation cannot claim the defence that the other
failed to exercise due diligence in order to escape liability. Where the misrepresentation goes to
inducing another to enter into a contract, the law assumes the individual entering into the contract
was induced by the representation to do so. This is rebuttable when the individual knew the
statement to be false or did not act in reliance of it.

59
• Exception to the PER: it is a voidable contract because of the misrepresentation
Ratio:
1. In cases of fraudulent misrepresentation, the damage awarded in contract is rescission, while the
damage awarded in tort is damages (i.e. money). You can only receive contract damages if you can
imply that there is a collateral contract implied in the bargain
2. Fraudulent misrepresentation occurs when the maker of the misrepresentation
a) Knew the representation was false
b) Does not know whether the representation is true, has no basis for belief in its truth and does
not believe it is true
3. If the misrepresentation would induce a reasonable person to enter into a contract then the law
assumes the individual entering into the contract was induced by the misrepresentation to do so
a) The misrepresentation does not need to be the only factor inducing the other party to enter
into the contract; it just needs to be an inducing factor.
b) Exception: This inference does not apply where the individual knew the statement to be false
ahead of time or did not act in reliance of the false statement
If an individual makes a false representation, alleging that the individual could have proven the
representation false through due diligence is no defence.

Esso Petroleum Co v Mardon [1976] EN CA – Negligent Misrepresentation


Facts: Negligent misrepresentation and collateral contract. Esso builds gas stations and before they
build, they do a thorough survey to monitor traffic in a particular area to see how much gas will be
pumped. They conclude that this station will pump 2000 gallons per year and want to have someone
come in and run it. Mardon rents the service station expecting revenue based on that number. No
term of lease indicates any specific volume. Esso did their survey on basis of creating entrances and
exits off of the main road and the city subsequently said no, that they would need to use the side
road, Esso agreed. Mardon took over and spent years trying to make it work but it wasn’t financially
viable. Esso then brought an action for possession against Mr. Mardon.
Issues: Should P recover in breach of an implied warranty (contract) or negligence (tort) under
Hedley Byrne? The deal that the amount of gasoline pumped out was necessary for the formation of
the contract.
Holding: Judgment for P Denning states can get contract or tort damages
- Tort damages for negligent misrepresentation
- Or contract damages – represented a collateral contract that a specific amount of gasoline
would be pumped in addition to first contract (lease property for money)
Reasons: Denning: P can successfully recover for either a breach of an implied warranty (contract)
or negligence (tort). The motivation behind damages is that the injured party be restored to their
original position. P cannot recover twice. When D found out that it based its study on the wrong
facts, it had a duty to alter the study accordingly. Esso professed special expertise and breached
their DOC.
• Hedley Byrne Requirements: When do we have negligent misrepresentation: Need a special
relationship (experts) = ESSO, based on their special skill, they tell Mardon that the station will
pump this much, he reasonably relied on statement and induced the formation of a contract = if
meet these requirements, then have a duty to exercise their special knowledge with reasonable
care.
Ratio: If negligent misrepresentation induces the formation of contract, person may have a tort
remedy, as long as they can fulfill Hedley Byrne requirements

Remedies
Rescission:

Redican v Nesbitt SCC 1924


Facts: Nesbitt sold house to Redicans without their prior inspection. The keys were exchanged for

60
the cheque and when the Redicans saw the property, noticed there had been misrepresentation
about the number of bedrooms, electricity, etc. and ordered a stop payment on the cheque. Nesbitt
sued for payment and the Redicans defended by suing for rescission. Trial judge found it was an
innocent misrepresentation but since contract had been executed, it couldn’t be rescinded.
Issue: Can the court rescind the contract on the purchaser’s claim of misrepresentation?
Holding: Appeal allowed. New trial ordered. Will allow the transactions in the land deal to be set
aside bc there was fraud and people should not be allowed to benefit from fraud.
Reasons: Court cannot rescind contract because (a) the contract was completed (i.e. when the
cheque was given for the keys), and innocent misrepresentation after the completion of a contract
does not support rescission, and (b) the misrepresentation is not significant enough to consider the
vendor’s consideration false.
Ratio: BAR TO RESCISSION:
• A fully executed contract is a bar to rescission in the case of innocent misrepresentation, unless
the misrepresentation is of such a substantive degree that it renders the consideration for the
contract false consideration. If misrepresentation is fraudulent then this principle does not apply.
An individual should not be able to profit from fraud.

O’Flaherty v McKinley [1953] Nfld CA –


Inability to return other party back to original is bar to rescission
Facts: P brought a car from D that D innocently misrepresented as a 1950 model, when it was
actually a 1949 model. P drove the car 7000 miles and then found out that the car was 1949. P sues
for rescission of the contract and for damages. She sues for the money she paid for the car and for the
insurance and repairs she made to it while she owned the car. D says can’t make restitution bc put
7000 miles on it
Issue: Can P obtain a rescission of the contract even though the contract was complete and P’s
misrepresentation was an innocent misrepresentation?
Holding: Yes, judgment for P, rescission granted.
Reasons: P can obtain a rescission of the contract even though the contract was complete and D’s
misrepresentation was innocent. The difference between cars is too substantial for the two cars to be
equal consideration. D’s claim that rescission cannot be granted because of the miles put on the car
change its character substantially does not hold up because the judge considers the car “still a young
car” and was not devalued by its use. The substantial restitution is “good enough” in this case
because of the fraud.
Dissent: The buyer is afforded a reasonable period to inspect the goods after purchase to determine
if they are the goods that the vendor represented them as being. If the buyer does not inspect the
goods within that time, then the buyer must accept the responsibility and looses her right to sue. In
this case a reasonable time elapsed and P forfeited her right to sue (Leaf v International Galleries)
Ratio: Rescission cannot be granted unless the parties can be restored to their original positions.
Rescission will be available if one of the parties can grant substantial restitution. Exception: Where
complete restitution cannot be granted, then the court allows part restitution and part compensation.

Leaf International Galleries [1950] KB CA


Facts: “Salisbury Cathedral” by John Constable was what Leaf thought he was buying on March 8,
1944 from International Galleries. International Galleries said it was a Constable. Leaf paid 85
pounds. Five years later when he tried to auction it, Leaf was told that it was not a Constable. He
claimed rescission of the contract against International Galleries, to get his money back.
Issue: Can the judge grant rescission of the contract even though P has held the innocently
misrepresented painting for a number of years?
Decision: Judgment for D. No right to rescind.
Reasons: Denning: Transaction cannot be rescinded because too must time has passed. Five years is

61
not a reasonable time. If the contract had stated it was a Constable, under the Sales of Goods Act, the
buyer is deemed to have accepted the goods, “when after the lapse of a reasonable time, he retains
the goods without intimating to the seller that he has rejected them.” Denning holds that if you can’t
set contract aside for breach, then shouldn’t be able to under rescission.
Ratio: BAR TO RESCISSION: If K is for a sale of goods, right to rescind is barred after a reasonable
period of time (Sale of Goods Act)

Damages

Heilbut Symons & Co v Buckleton [1913] HL


Facts: Heilbut Symons & Co were rubber merchants who were underwriting shares of what they
claimed was a rubber company. Buckleton called up manager at Heilbut to inquire about the shares.
In response to the questions, the manager stated that they were “bringing out a rubber company.”
Based on this statement, Buckleton purchased a large number of shares. The shares turned out not to
be for a rubber company at all. The shares later dropped bc of a problem with the rubber trees and
Buckleton brought action for fraudulent misrepresentation and also for damages for breach of
warranty that the company was a rubber company whose main object was to produce rubber.
Issue: Was there a collateral contract to the main contract to take the shares in consideration that
the company was a rubber company?
Holding: VISCOUNT HALDANE LC: Judgment for the D. No collateral contract.
Reasons: Just because the vendor states a fact of which the buyer is ignorant doesn’t mean that there
is warranty. Intention of parties is only deduced from the totality of the evidence. At common law,
there is no right to damages for an innocent misrepresentation. Courts often try to sidestep this by
finding a collateral contract where there was really only an innocent misrepresentation. In this case,
the statement was made in response to an inquiry – representation as to specific thing, nothing more
– no evidence either party thought contractual liability would attach to it as a collateral contract.
Ratio:
• In order to obtain damages on a collateral contract for innocent misrepresentation, the plaintiff
must show that the parties had a real intention to create two contracts (i.e. the representation
must be clear and separate)
• Collateral contracts by their nature must be rare. If parties intend the language to be a contract,
they would have put it into a contract. Must prove that it’s inherently likely for two contracts
rather than one. You can prove a collateral contract and not offend the Parole Evidence Rule.
Collateral Contract can’t be proven though if it conflicts with the language of the main contract. It
must be a real inducing factor. Now have good ammunition for misrepresentation if you follow
the collateral contract route. Damages for breach of the collateral K.
• Have to overcome the factual issue of if the two parties intended to form a contract, can you
actually prove that there was a representation made in a way where there was an intention to
make a collateral contract.

Shanklin Pier Ltd v Detel Products Ltd [1951] KB


Facts: Shanklin Pier Ltd. hired a contractor to paint Shanklin Pier. They spoke to Detel
Products Ltd. about whether a particular paint was suitable to be used, Detel assured them
that it was and that it would last for at least 7-10 years. On the basis of this conversation
Shanklin Pier Ltd instructed the contractors to use a particular paint, which they did. The
paint started to peel after three months and Shanklin Pier attempted to claim compensation
from Detel Products. No direct contract between the two companies, only between Shanklin
Pier and the contractors and between the contractors and Detel Products. P sues based on
collateral contract.
Issue: Is P entitled to recover damages through a collateral contract?

62
Holding: McNair J: Yes, judgment for P. Plaintiffs are entitled to recover damages for
breach of collateral contract.
Reasons:
• Collateral contract was: “in consideration of telling our painter to use your Detel paint,
you represented that the paint would last at least 7 years.”
Ratio:
• In a specific situation, a third party to a contract can sue for fraudulent
misrepresentation. A collateral contract can exist where A represents to B that they
should get C to use a specific product A sells. In consideration of B getting C to use A’s
product, A represents that the product will be effective.
Notes:
• This case is easier because there is no main contract to call into question the collateral
contract – don’t have to prove the real and inducing factor of the two parties entering
into two contracts
• There is no conflict between two contracts – there is no Entire Agreement Clause – there
is no MAIN contract between the two – the main contract is between Detel and Painters.

Esso Petroleum Co v Mardon [1976] CA


Issue: Should P recover in breach of an implied warranty (contract) or negligence (tort) under
Hedley Byrne?
Reasons: P can successfully recover for either a breach of an implied warranty (contract) or
negligence (tort). When D found out that it based its study on the wrong facts, it had a duty to alter
the study accordingly. Esso professed special expertise and breached their duty of care.
Ratio: If negligent misrepresentation induces the formation of contract, person may have a tort
remedy as long as they can fulfill Hedley Byrne requirements (special knowledge, reliance). An
innocent representation gives no rights to damages. A representation – that induces a person to enter
into a contract – can be held as a warranty sounding in damages. In order to hold that representation
as a warranty, the representation must be one of fact and not opinion (Bisset v Wilkinson).

Mistake
Parties don’t have the same view of the facts
Mistake means no contract – void contract – it never existed

1) What type 1. COMMON MISTAKE: Both parties share the same fundamental mistake.
of mistake is EQUITABLE COMMON MISTAKE: probably bad law in Canada
it? 2. MUTUAL MISTAKE: the parties have different understandings of the terms and
neither is aware of the other’s understanding.
3. UNILATERAL MISTAKE: One party is mistaken, while the other is not. Generally,
the other party knows that the mistaken party is mistaken.
4. MISTAKE AS TO DOCUMENTS: One party is fundamentally mistaken about the
document they signed (non est factum)
5. FRUSTRATION: parties are mistaken as to some future assumption. When the
assumption is untrue, performance of the contract may be excused.
(1) Common DEFINITION: Both parties share the same fundamental mistake. (McCrae v
Mistake Commonwealth Disposal)

GENERAL RULE: A common mistake must go to the substance of the thing


contracted for, not the quality/character of that thing (Bell v Lever Bros)
If mistake is about quality/character, the contract remains binding UNLESS
the quality is a term of the contract or there is misrepresentation (i.e. sick

63
horse not enough). Refer to the difference between the barren cow and the
fertile cow (Sherwood v Walker – not good law)
Very difficult test to prove.
Buying something you already own Cooper v Phibbs
Buying something that doesn’t exist, i.e. Corn fed pigs Coutourie v Hastie

TEST FOR COMMON MISTAKE: (Great Peace Shipping v Tsalviris; overruled Solle)
The following elements must be present if common mistake is to void a contract:
1. Mistake must go to existence of a vital attribute of subject matter of
contract (lower attribute than Bell v Lever)
2. Mistake must render performance impossible
It must go to the heart of consideration, higher standard than Bell
3. Mistake can’t be the fault of the party pleading the mistake
4. At formation of contract, neither party accepted the risk – no warranty

REMEDY: The contract becomes void. All property passed through the contract is
returned, even if a third party has rights to it.
(2) Equitable GENERAL RULE: A contract can be set aside in equity if (Solle v Butcher)
Common 1) The parties were under a common mistake about the facts or their relative
Mistake rights; would they have entered K if they had known?
2) The mistake was fundamental
3) The party seeking to set it aside was not at fault for the mistake
4) Would it be unfair to give remedy? Neither party must be seen to have accepted
the risk of the mistake
Denning creation for cases where it would be inequitable to not allow contract
to be voidable due to mistake. (Common Law Common Mistake is difficult to
prove)

REMEDY: The contract is voidable (not void) (subject to bars of rescission)

APPLICATION IN CANADA:
This is potentially bad law in Canada. The higher threshold set out in Great
Peace Shipping probably renders this so.
Miller Paving Ltd. v Gottardo Construction says equitable common mistake is not
dead in Ontario – likely to give more flexibility and fairness while protecting
third parties
If you can prove equitable CM, and this gives a good remedy to set K aside, this
is a good way to go. But if that doesn’t work bc of bar to rescission, will need to
plea common law common mistake.
(Look to short summary for 3,4, 5 – not in this summary)

Common Mistake
Mistake is shared between parties – both are wrong

Bell v Lever Bros (1932) HL


Facts: Lever Bros hired Bell to be the CEO of their company for 5-year contract. After 3 years, they
don’t need him anymore so they make a contract to pay him 30,000 pounds to terminate the
employment. Later on in the year, Lever Bros find out that Bell had been making his own profits on
the side without disclosing them to the company and therefore bring an action against him for the
return of 30,000 pounds arguing that there is a common mistake: Both thought they were entering
into contract to end a five year employment contract when in fact the contract was terminable for
cause and that this is a mistake as to the substance of the contract.
Issue: Can P terminate the contract and get back their money due to common mistake?

64
Decision ATKIN: Judgment for D. No mistake as to substance of contract, only mistake as to quality;
i.e. Lever Bros can’t recover their money.
Reasons: The mistake went to the quality of the thing contracted for and not the substance of the
contract. D thought they were terminating employee with a five-year contract, when in reality they
were getting rid of an employee who they had actual cause to terminate. Lord Atkin gives an example
of a sale for a sick horse, compared to the sale of a sound horse. If both parties believe that horse is
sound, but it turns out the horse is sick, then the party buying the horse has no remedy.
Ratio: A common mistake must go to the substance of the thing contracted for, not the quality of the
thing contracted for. This makes common mistake very difficult to prove.

Sherwood v Walker – US LAW


Facts: Sherwood (P) contracted to purchase a cow from Walker (D). Walker showed Sherwood a cow
that he believed to be barren. Sherwood agreed to purchase the cow for $80. If the cow had been
fertile it would have been worth $750. Walker later discovered that the cow was with calf and
refused to complete the transaction. Sherwood sued. At trial, Walker showed that at the time of the
sale, both parties had believed the cow to be barren and both knew that the value of a fertile cow was
much higher than that of a barren cow. The judge instructed the jury that it was immaterial whether
the cow was barren. The jury ruled in favor of Sherwood and Walker appealed.
Issue: Can a common mistake regarding the substance of the subject matter of a contract render a
contract unenforceable?
Holding: No contract. A common mistake regarding the substance of the subject matter of a contract
may render that contract unenforceable.
Ratio: A common mistake regarding the substance of the subject matter of a contract may render
that contract unenforceable.

Solle v Butcher [1950] KB


Facts: P and D are friends and partners in business. D rents out flats and keeps one for him. The
parties have always assumed that the flats were not subject to rent control. P and D have a falling out.
P is still the landlord of D in the one flat. D goes to the rent control office and asks if the flat is subject
to rent control. Turns out it is and rent should only be 180 pounds. P asks that the lease to be set
aside.
Issues: Can P set aside the lease on the ground that both P and D made a mistake about whether the
flat was rent-controlled?
Holding: Judgment for D (P has option of rescission or paying full rent).
Reasons: DENNING: Two types of common mistake: 1) Common Mistake at Common Law – renders
the contract void 2) Common Mistake at Equity – renders the contract not void, but voidable – for
this you have to prove it was important to the parties at the time contract was made. Rescission is
available as a remedy as long as there is no bar to rescission. In this case, P was responsible for
letting D know that the flats were subject to rent-control. It would be inequitable for P to benefit from
the mistake that he made. On this basis, P cannot recover.
Ratio: Equitable Common-Mistake: A contract can be set aside in equity if the parties were under a
common mistake about facts or their rights, provided that the mistake was fundamental and that the
party seeking to set it aside was not himself at fault.

Great Peace Shipping Ltd. v Tsavliris (International) Ltd . [2002] CA


Facts: D needs someone to assist to bring a ship back to port and to save the crew, because the ship
may be going down. D concludes that P has a ship that could help. They make a contract for five days
of salvage. D finds out that P’s ship is 400 miles away and not 35 miles away. D finds someone closer
and contracts with them, D then cancelled the contract with P and refused to make any payments. P
brought an action claiming $82,500 for payment of the contract or for damages. D disputed the claim

65
on the basis that the contract had been formed under a fundamental mistake (both parties thought
the 2 vessels were in close proximity and they weren’t) and that the contract should be void or
voidable and they should be entitled to rescission.
Holding: Judgment for P (that D did not rescind K indicated that performance was possible, even
though it was not ideal)
Reasons: Court says that Solle v Butcher was wrongly decided and that common mistake at equity
doesn’t exist and instead they will apply Bell v Lever Brothers. Equitable common mistake is a dead
end and now contract either a valid contract or is void for common mistake. Does that mean that
Canadian courts will no longer follow Solle v Butcher? It is unclear what Canadian courts would do
bc haven’t have many cases in this area since Great Peace Shipping BUT this case has taken a lot of
flack from academic writing criticizing it for killing Denning’s ideas and also bc it makes it almost
impossible to get a remedy for common mistake.
Ratio: New Test for Common Mistake: The following elements must be present if common mistake
is void to a contract:
1. Mistake must go to existence of a vital attribute of subject matter of contract (may be broadening
Bell v Lever Bros)
2. Mistake must render performance impossible
3. Mistake can’t be the fault of the party pleading mistake
4. Neither party assumed the risk of the mistake – Ex: in Sherwood v Walker – Walker assumed the
risk, if he paid small price and cow bore calf he did well, if not he paid a lot for a beef cow
At formation of contract, neither party accepted the risk – no warranty.

McCrae v Commonwealth Disposal Commission [1951] Aus


Commonwealth selling material to highest bidder from WW2
McCrae bids the highest, goes all the way to get it, there is no tanker
Commonwealth said there is a mistake – no tanker = no existence of contents = void contract
Courts said you cannot plead mistake if you made it
McCrae was able to plead breach of contract

Miller Paving Ltd. v Gottardo Construction [2007] ON CA (obiter)


Reasons: Suspect that you are within your rights to argue an equitable common mistake in the area.
Court does say that when talk common law common mistake, you look at Great Peace Shipping and
go through the four elements of it. On the facts of the case, En CA says that there isn’t common law
common mistake because performance of the contract wasn’t impossible. When the D found out
about the mistake, they didn’t say no K but kept it in place to see if there is someone closer.
- We elect not to follow that equitable common mistake doesn’t exit
Ratio: Equitable common mistake is not dead in Ontario.

Mutual Mistake
Parties have a different view of the contract and each of their views is reasonable (Raffles v
Wichelhaus).
If court finds a mutual mistake – Contract is VOID
Test: Have to decide if the parties are reasonable in the mistake being claimed

Raffles v Wichelhaus [1864] UK


Facts: Contract made for the shipping of cotton. Two different ships in India called the Peerless. P
thought ship was coming in October on first ship. D thought ship was leaving in December on second
ship. P sues for breach of contract when cotton does not arrive on the first Peerless ship. D claims
there never was a contract because of mutual mistake.
- Latent Ambiguity: ambiguity on what ship that would have cotton was discovered late
Issue: If there is latent ambiguity, does K exist? If there is latent ambiguity, can parole evidence be

66
submitted to determine what P meant and what D meant?
Holding: MARTIN & POLLOCK: No contract. Judgment for D.
Reasons: Both parties had different but reasonable views of the terms. Contract is void. Document
has latent ambiguity related to the time of the ship sailing. Not apparent on its fact but as soon as you
realize there are two ships sailing from Bombay, there is latent ambiguity and parole evidence can be
given to show that D and P meant different ships. Plaintiff gets no damage.
Ratio: Mutual mistake regarding an essential term of the contract means contract is void.

Smith v Hughes [1871] QB


Facts: P selling oats. D thought oats were old and bought a large quantity of them for his horse to eat
(can only eat old oats). Upon arrival, he realized they were new. D tries to send them back but P says
he never said they were old and the sample he showed was new oats. Hughes refused to pay and
Smith sued for breach of contract for the amount delivered and for damages for the amount of oats
that were still to be delivered.
Issue: Should we set this contract aside for mutual mistake?
Holding: Judgment for P, there was a contract.
Reasons: BLACKBURN: For mutual mistake, both views must be reasonable and D’s claim here is not
reasonable bc he agreed to buy the specific oats that he saw, thus representing he was contracting on
P’s terms. You can’t later try to opt out of a contract bc it wasn’t what you thought it was, if there is
no misrepresentation.
Ratio: If one of the parties intends to make a contract on one set of terms and the other intends to set
a contract on another set, then the parties are not in agreement and there is no contract. Where one
of the parties has an unreasonable view of the facts, then the contract exists on the terms of the
individual with the reasonable perception of the facts.

Unilateral Mistake
One party is mistaken, while the other party is not. In most of these cases, the mistake was
caused by the party that was not mistaken
Most of the time, party making mistake knows about mistake they are making
Very often, these are fraud cases and engage fraudulent misrepresentation and K is subject
to rescission (subject to bars) and tort damages through tort of deceit
Someone has been defrauded, the crook is gone and the property is in the hands of another
party
The misrepresentation remedy is no longer useful
Tort damages are no longer available – crook is gone
1) One party is mistaken and other party knows – “Snapping up a Mistaken Offer”
2) “Rogue Cases”: Fraudulent mistakes – one party is duped and therefore mistaken

Hartog v Collins & Shields [1939] UK


Facts: The subject matter of K is the purchase of hare skins. D mistakenly offered skins at a price per
pound. P accepted offer. Negotiation in past had always been at a price per piece – 3 pieces per
pound. As a result, D lost a considerable amount of money.
Holding: Judgment for D. Plaintiff should have known from the facts that the D was mistaken. Can’t
just snap up offers.
Reasons: We conclude on the facts that P knew about the mistake. D was mistaken, P didn’t cause the
mistake, but P knew about the mistake. The price was significantly too low. Court knew the buyer
had to be aware. One party was mistaken and the other party knew, therefore, K should be set aside.
Precedent: This case has become a highly relevant precedent in the modern context of e-shopping
on the internet, when online retailers sometimes get the published price wrong and receive
hundreds of online orders (automatically accepted) before they discover their error - e.g. advertising
a £299 television on the website for £2.99. Retailers can avoid having to supply at the mistakenly low

67
price if the court finds that the would-be purchasers must have known that the advertised price was
clearly a mistake.
Ratio: If one party is aware, or should reasonably have been aware, of the other party’s mistake in
making an offer, then the contract is void for unilateral mistake and the offeree cannot “snap up the
mistaken offer.”

Ron Engineering v R [1981] SCC


Facts: Crown calls for tenders – P steps forward. D accepts tender from P. P forgot to add cost. Crown
accept P’s offer anyways (lowest). P refuses to accept. D keeps deposit and sues for difference. Court
says there was a collateral contract that stated if for whatever reason P doesn’t perform contract,
they forfeit their deposit
Reasons: Told of mistake before acceptance can’t accept void. But Crown is entitled to keep
deposit (collateral contract)

Cundy v Lindsay [1878] HL


Facts: Thief named Blenkarn sends Lindsay a letter wanting to buy hankerchiefs, which they
manufacture. Lindsay thinks that the crook is a company they know of with the same name and so
sells them. Thief then sells them to Cundy, an innocent third party. Thief ends up in jail and has no
money, so what can Lindsay do? No option for rescission (Cundy already sold the hankerchiefs) and
he can’t bring claim against crook because crook doesn’t have property anymore and has no money.
Lindsay alleges unilateral mistake bc he wants a property claim (to get this, must be a unilateral
mistake contract void, so then he can sue in property)
- Lindsay has to show that his mistake of believing he was dealing with a company was
FUNDAMENTAL to the contract
- There are property rights still owed to the original owner bc she pleaded unilateral mistake –
voided contract – gets property back from the hands of the purchaser
Issue: Was the identity of the crook fundamental to Lindsay? Did Lindsay mean to contract with the
sender of the letter, or only with the company, Blenkarn?
Holding: Contract is void, Judgment for Lindsay
Reasons: The mistake was fundamental – Lindsay meant to contract with the company Blenkarn,
therefore there was a unilateral mistake, contract is void, and Lindsay can sue Cundy for converting
his property.
Ratio: Mistake in correspondence: In correspondence, a unilateral mistake as to real identity of
the other party to the contract will make the contract void, if the facts indicate that the mistake as to
ID is fundamental (e.g. you meant to contract with X instead of a thief)

Ingram v Little [1961] UK QB


Facts: Ingram sisters sold car to a man with a faulty identity (Mr. Hutchinson) whom paid by check.
This man then sold the car to Little, Ingram now sues Little for the return of the car or damages for
tort of conversion. Trial judge said that P’s mistake as to the identity of person she was dealing with
prevents formation of contract, so P should recover. D appeals.
Issue: Is there an actionable unilateral mistake here? (There is a bar to rescission bc of 3 rd party
intervention, so they need to plead unilateral mistake to make contract void and get property rights)
Holding SELLERS: Contract void for mistake. Judgment for P. Ingram gets car back.
Reasons: You start with presumption that the party intended to contract with the person in front of
them. Here sisters only intended to deal with Mr. Hutchinson and the mistake was fundamental to
the transaction, they used due diligence to check up on him.
Ratio: Face to Face Mistake: In cases of face to face negotiations, the same result may be reached as
in Hartog, as long as the appearance is still that the mistake is fundamental, and the innocent party

68
has undertaken some level of diligence to indicate that it is. The presumption in these kinds of
negotiations is that the parties are not fundamentally mistaken but you can rebut that presumption.
Presumption: in face-to-face negotiations, intend to deal with the person in front of you
Rebuttable: Show that you made an effort to verify the identity of that person, attempted to do some
level of diligence, therefore there is a material mistake.

Lewis v Averay [1972] QB CA (probably bad law in Canada)


Facts: Lewis sold his car to a man, a “rogue” pretending to be the actor from Robin Hood, and the
check he got was stolen and worthless. The “rogue” then sold the car to Averay under Lewis’ name
and got away with the money. Lewis is now suing Averay for damages for tort of conversion.
Issue: Was there a contract under which the car passed property from Lewis to the rogue? What is
the effect of a mistake by one party as to the identity of the other?
Holding: Contract. No fundamental mistake (2 of 3 judges) and all three deny remedy.
Reasons: Denning: Unilateral mistakes do not make a contract void, but rather makes it voidable,
subject to bars of rescission. Here there was a bar to rescission, and therefore no remedy. (This is
NOT the law in Canada, however). On this basis, the original seller should bear the loss caused by the
mistaken identity.
- Denning does not believe that a mistake of an identity should make a contract void, it should be
voidable – bar to rescission is that the car was given to a third party.
Ratio: Face to Face Mistake: When two parties have come to a contract, the fact that one party is
mistaken as to the identity of the other does not mean there is no contract, or that the contract is null
and void from the beginning. It only means that the contract is voidable, i.e. liable to be set aside by
the mistaken person, so long as he does so before third parties have in good faith acquired the rights
under it (i.e. no bar to rescission). On this basis, the original seller should bear the loss caused by the
mistaken identity.

How do we distinguish these two car-cases?


In Ingram v Little: there was a positive act taken to establish that you thought identity was important
(Ingram checked the phonebook to verify that a person named Hutchinshon lived on the named
street). If you take action to verify that the statements made were true then the courts are more
likely to accept it as evidence that the identity of the person was material to the contract being made.

What about Denning’s judgment?


Arguably more equitable in the result (innocent third party would not suffer) BUT, following Great
Peace Shipping, we would likely say now that there is no equitable remedy for mistake. (This
could be argued though because there is no binding judgment in Canada.

Mistake as to Documents (Non Est Factum)


- In rare circumstances, someone can say they were mistaken as to the documents that were
signed and as a result that person can get out of the contract.
- In order to plead Non Est Factum, must be able to show that the document is of a different
class of document than what you intended to sign. (e.g. you intended to sign a contract and
signed a guarantee)
- In order to disentitle you from pleading non est factum, you had to show negligence on the
person who signs the document. There must therefore be a duty of care.

Thoroughgood Case (300 years ago)


Thought he was signing a release of rent, so he could evict his tenant. After tenant has read the
document to him as a rent release – it was actually a document to give his property away
SIR EDWARD COOK: says you are not bound by this document – if Thoroughgood had not had
the document read to him and shown that he intended to exert his due diligence than he would
not have been released from the duty to carry out the contract

69
Saunders v Anglia Building Society [1971] HL
Facts: Mrs Gallie, who had broken her spectacles, signed a document without first informing herself
of its contents. She was lied to by her nephew’s business partner, Mr. Lee, that the documents were
merely to help her nephew. In fact, she signed papers allowing the nephew’s business partner to
grant a mortgage over property in favor of Anglia Building Society. When the business partner
defaulted on the mortgage, Anglia Building Society claimed to foreclose and repossess Mrs. Gallie’s
house. Mrs. Gallie died before the litigation reached the HL, and was represented by Saunders as the
beneficiary of the estate (i.e. Wally). Argue that deed should be taken away from the building society
bc Gallie was mistaken about what she was signing.
Issue: Can the contract be set-aside on a plea of non est factum?
Holding: Judgment for D (Anglia)
Reasons: PEARSON: agrees with Denning about class/content issue and adopts his formulation that
in order to plead non est factum, the mistake has to be a document that was radically or
fundamentally different than what was intended. Even if Gallie asked for document to be read to her,
they wouldn’t have read the right thing. So Gallie was not careless. However, the document was NOT
radically different or fundamentally different than what she intended. They agreed with majority in
the court of the appeal.
- Other two judges – Gallie loses bc doc she signed is exactly the type of document she intended to
sign. She intended to transfer the house for the benefit of Wally and that’s what she did, only
difference is the intervening fraud by Lee.
Denning at appeal level: A plea of non est factum should be about whether the document is
radically different from what was intended to be signed. You can disavow the document while the
property is in the hands of the other party, but once it’s in the hands of a third party, you can’t. That is
a bar to rescission.
Ratio:
1. The essence of the plea of non est factum is that the person signing believed that the document
was different in character or effect than what was actually signed.
2. A person pleading non est factum must have taken steps to determine what the document was
about
3. A plea of non est factum can only apply to those who are blind, illiterate, or through no fault of
their own do not have a real understanding of the document.
To make the pleas, there must be a fundamental difference between what the claimant signed and
what she thought she signed. It must be left to the courts to decide what exactly this means.

Prudential Insurance v Cugnet 1956 SCC


Cugnet doesn’t read a document and signs it. Turns out to be a deed to the land he owns
Holding: Judgment for Cugnet. Non est factum.
Reasons: Cugnet didn’t owe a duty of care to Prudential Insurance and what he signed was a
different class of document, so Cugnet is free to plead non est factum.
Ratio: A plea of non est factum only functions where the document is of a fundamentally different
class and not when it has different content. – what you need to plead non est factum is negligence not
carelessness

Marvco Color Research v Harris [1982] SCR


Facts: Harris’ own a house and have one child, a daughter who has a fiancé. He is trying to buy out
business partner and asks Harris’ for help. To do so, they mortgage their house. As it progresses, he
needs more money. Goes back to Harris’ and says that there was a typo in the mortgage so re-
executes the mortgage, this time for a higher amount. Harris did not read the document before he
signed it. He later sought to avoid liability on the basis that he did not understand the nature of what
he had signed due to the fraudulent misrepresentation of his son-in-law.

70
Holding: Contract. Judgment for Marvco
Reasons ETSEY: Harris was careless in not properly understanding the document, which disentitles
him from a plea of non est factum.
Ratio: Carelessness will disentitle a plea of non est factum
Forbes: we don’t need to decide between if carelessness standard or negligence disentitles plea,
need to decide DENNING/PEARSON that the document was radically different

Frustration
Even occurs after the formation of the contract that makes the performance of the contract
radically different
Will say that performance is excused
A mistake occurred that nobody thought would happen

Knell v Henry
P rented balcony to family so they could have a big event for the coronation event on the street
Family had a right to bring the party to the balcony on a certain date
Prince gets appendicitis
P says that you can still sit on the balcony on that date, just no coronation

Illegality
When engage in illegal conduct renders contract unenforceable (not void because
leads to connotation that you can regain property rights)

REMEDY Where a contract is found to be illegal at common law, contradicts a statute, or


includes an illegal restraint of trade clause, that contract will be unenforceable –
different than void or voidable.
• Unenforceable means no recovery of damages
• “Public policy is a very unruly horse, and once you get astride it you will never
know where it will carry you” Richardson v Mellish
• “With a good man in saddle, the unruly horse can be kept in control. It can leap
fences put up fictions and come down on the side of justice.” (DENNING:
Enderby v Football)
1. What 1. Illegal at common law
type of a. Contracts that offend public policy
illegality is b. Contracts with restraint of trade clauses:
it? • Non-competition clauses
1. Sale of business
2. Employment (Gordon v Ferguson)
• Contracts of tied selling agreements (sell 1 type of product only for a
reasonable period of time)
2. Statutory Illegality
a. Type of contract or performance is illegal based on statute

Two Concepts to Consider:


1. What is the effect of the illegality on the contract – contract/terms are
unenforceable.
2. What happens to property delivered under an illegal contract – property is
returned (?); what is best for the baby (in the matter of Baby M)
Common Does the contract fall into one of the existing categories below?
Law • Yes Contract is illegal and unenforceable. Property cannot be recovered from an
Illegality illegal contract.
• No Contract is legal and enforceable. Property returned.

71
Contracts Illegal at Common Law:
1. Contracts to commit a crime or a tort: no profit from wrongdoing (Oldfield)
a. EXCEPTION: Innocent named beneficiaries of insurance policies should be
allowed to collect (Oldfield v Transamerica Life)
2. Contract to defraud the revenue authorities (Alexander v Rayson)
3. Contracts that promote corruption in public offence (Parkinson v College of
Ambulance – knighthood w donation)
4. Contracts for an immoral purpose (Andrews v Parker, Wilkinson v Osborne)
a. Consider the societal standards at the time
& Contracts that are contrary to public policy (In the Matter of Baby M)
• TEST: Does the contract contravene “some defined and governing principle
which the community as a whole has already adopted either formally by law
or tacitly by its general course of life?” (Wilkenson v Osborne)
Illegal Restrictive covenants give rise to tension in the common law between the concept of
Restraint of freedom to contract and public policy considerations against restraint of trade, which
Trade interfere with individual liberty and limit the exercise of trade.
Clauses • Presumption that restrictive covenants are prima facie unenforceable but will be
enforced if shown to be reasonable in reference to the interests of the parties
concerned and to the interest of the public (Shafron v KRG Insurance)
• Ambiguous restrictive covenants are by definition not reasonable, unless the
ambiguity can be resolved (Shafron)
• A restraint of trade clause is either reasonable or not. If found to be
unreasonable, the clause will not be enforced. The courts will not read it down to
make it reasonable (Gordon v Ferguson)

Test for determining if restraint of trade clause is enforceable (Shafron v KRG


Brokers): Applies to:
- Vendor Contracts (Sale of Business): generally be more reasonable than
employment K bc the purchase price paid includes goodwill and there is more
equal balance of power
- Employment Contracts: generally less reasonable than restrictive covenants in
buyer/seller of business. This is because employees are not paid goodwill and
have an imbalance of power – more likely to be seen as illegal
- Tied Selling Agreements: Brewery interest loans/expertise to renovate bars in
return for exclusive sale of its products until loan repaid

1. Has non-compete been drafted in a way that is clear and unambiguous?


a. An ambiguous and unclear clause is prima facie unenforceable
b. Construe non-compete clauses strictly against employer/purchaser. They are
enforceable if tightly drawn
c. It’s easier to enforce in the sale of business than an individual in an
employment relationship
2. Is the restraint reasonable between the parties?
a. Will the customers follow the employee? (Gordon v Ferguson, below too)
b. Does the employee have confidential info that he can take to competitor?
3. If yes, is the non-compete clause more broadly drawn than necessary to the
purchaser/employee’s valid business interests, in terms of geography and time?
a. How much distance is the right amount of distance?
b. How long is the right amount of time?
c. The broader the geographical area and the longer the restraint of trade clause,
the less likely is the court to find the clause reasonable and therefore
enforceable
4. If meets above, is the non-compete clause contrary to the public interest?

72
a. How does it effect essential services?
b. Does it create a monopoly that will result in unreasonable price increases?

Does the clause meet all the requirements of the test?


YES Clause is legal and full contract is enforceable. Property returned.
NO Only the non-compete clause is illegal and unenforceable, not the entire
contract; contract proceeds as though clause was not there. Property is not
recoverable
Statutory General Rule:
Illegality A contract that is expressly or impliedly prohibited by statute is unenforceable,
regardless of the intentions of the parties to it St. John Shipping Corp v Joseph Rank Ltd

Contract is Illegal at Formation:

1. Expressly Prohibited: Prohibited writing of contract – it is illegal to buy guns


– therefore can’t write a contract to do so
2. Impliedly Prohibited:
• Statute does not explicitly state illegal, but is aimed at doing so – it states that
there are penalties for engaging act; and it impliedly prohibits the contract if the
statute makes an offence out of entering into contract
• Otherwise, whether the statute impliedly prohibits the contract can be determined
by:
Test for Indirect Illegality: (Yango Pastoral v First Chicago Australia)
1. What is the purpose of the statute?
• What is the intent of the legislation or mischief the legislation trying to
solve?
2. Who is the statue trying to protect and is the person in this class?
• If the plaintiff is protected under the statute than it should be enforceable
3. Is there a penalty for breach of a statute, and does the penalty relate to
entering into this type of contract?
• If yes, and the penalty does not relate to the contract, then unlikely
contract will be found to be illegal
• If statute does cover the contract, it’s likely meant to address them. Statute
prohibits the contract and t/f contract is illegal and unenforceable.
If yes and the penalty does not relate to the contract, then unlikely the contract will
be found to be illegal.

Contract is illegal in formation:


The contract itself is legal but the performance of the K is involved an illegality
1) Illegal Performance intended by One:
• If (A) intends, at the time he makes K, to perform illegally and does perform
illegally then (A) cannot enforce but (B) still can. If unintended by both, then K still
stands. (St John Shipping)
o If one intends to perform illegally then they can’t enforce contract,
but other party can
2) Intended by Both:
• If both parties make a deal and know or ought to have known that the only way
it can be completed by illegal means, then neither can enforce (Ashmore, Benson,
Pease & Co)

Common Law Illegality


Contracts that are contrary to public policy
Contract is unenforceable

73
Oldfield v Transamerica Life [2002] SCC
Facts: Oldfield died after swallowed a condom with cocaine in it while smuggling from Bolivia. Wife
was beneficiary of estate. P claimed the proceeds of a life insurance policy under which she was the
named beneficiary. He died while smuggling balloons of cocaine out of Bolivia (one burst in his
stomach). His actions were contrary to both Bolivian and Canadian Law. D refuses to pay on public
policy grounds.
Issue: Whether P can claim life insurance policy, even though he died while committing an illegal
act?
Holding: Contract. Wife should be able to recover. Beneficiary of the policy is an innocent third party
and had nothing to do with illegality.
Reasons: P can claim under her ex-husband’s life insurance policy even though he died because of
committing an illegal act. Public policy vitiates insurance claims where the claim results from the
insured’s illegal act or the claimant’s illegal act. This rule extends to claimants claiming under the
insured’s estate, in the case where death results from the illegal act. However, this does not prevent
separate beneficiaries from claiming under the insurance policy of the insured.
Ratio: The general rule is that “ a person should not be allowed to insure against his or her own
criminal act irrespective of the ultimate payee of the proceeds” (Brisette Estate v Westbury Life
Insurance). Exception: An innocent beneficiary named in an insurance policy should not be
disentitled to insurance proceeds where the insured dies while committing a criminal act and does
not intend the loss.

Alexander v Rayson
Property is leased and a side contract for 750 pounds to do side work to maintain the property, so
that the landlord can get less property taxes
Tenant walks away from lease and landlord sues
Court sees the contract as unenforceable because it was designed to defraud revenue agency
Pay cash is less in contracting as per cheque = if need to bring a claim, court may see you as
helping in tax fraud (Alexander v Rayson)

Parkinson v College of Ambulance


Facts: Parkinson wants to be knighted, college says they have influence over gov’t and if he gives
them a donation, they will try to get him knighted. He sues to get money back when he does not
become a knight.
Holding: Court won’t help him enforce a contract for a bribe
Ratio: An agreement to promote corruption in public office is illegal and court won’t help enforce
that.

Andrews v Parker
Illegal contract to defraud authorities – tax purposes
Ratio: Idea of morality changes over time and shifts. Must make Ks illegal bc of public policy reasons
that relate to current social values.

Richardson v Mellish
Ratio: Public policy is an “unruly horse”, you should be reluctant to mount this horse, lest it run away
with you – public policy is a dangerous thing and treating Ks as illegal bc of public policy is not a good
thing – it changes over time

Wilkinson v Osborne
Holding: A contract that interferes with the impartial judgment of a public official, including
members of Parliament, will be illegal
Ratio: Public policy, which court is entitled to apply as a test of validity of a K, is about values

74
adopted as a whole by the community. Public policy reflects societal values and norms. E.g. we as a
society think that K’s of this type should not be enforced. Not what judge thinks, supposed to be judge
reflecting what society thinks

In the Matter of Baby M [1988] NJSC


Facts: Sterns contract with Whitehead to give them her child in exchange for 10K and payment of
medical expenses. She has the baby and then changes her mind. Goes into hiding. Sterns sue for
specific performance of the contract (i.e. make her give the baby). Trial judge says it’s a good contract
and doesn’t offend societal values. Won’t give specific performance until determining the best
interests of the baby, which was Sterns. Whitehead appeals.
Issue: No contract. Whether K should be upheld or voided for public policy reasons
Holding: Contract is illegal and contrary to public policy. Baby goes to Sterns
Reasons: K is illegal bc it denies baby natural mother, ignores impact on both sets of parents, best
interests of baby. Public policy is against it. However, judge also ruled that it was in best interest s of
the child to be with Sterns.
Ratio: Contract is illegal if it violates public policy.

Restraint of Trade Clauses


Contrary to public policy
Restraint of employment clause:
o An employee non-competition restraint of trade clause: can’t work for another
employer if employed, must be tested to see if it is a reasonable restraint
o A vendor non-competition restraint of trade clause: there is value in your business
but only if you don’t set up shop down the street
o A sale of business contract Not construed as strictly, person extracting restraint
has paid for it, whereas in employment non-competes, they are restraining future
work needs to be stricter
Tied Selling Agreement
o Where a company forces another company to only sell their product

Gordon v Ferguson [1961] NSSC


Facts: Employer hires a junior doctor in his office. Worries that patients become attached to him and
he will take the business with him. Includes a non-compete clause in contract, stating that if he
leaves, he won’t practice medicine in the same city for five years.
Reasons: Employer has a valid business interest to protect. Courts look at where the client’s are
drawn from and hold that he can’t shut the junior doctor out of practicing in an area that’s outside
where the doctor draws from. Also how long do you think you can say that person can’t practice
around here? E.g. Might be difficult to believe that after 2 years, people would follow… can’t be too
broadly drawn. Public concern: will there be sufficient medical services in the town if you shut the
employee out?
Ratio: Non-compete clauses can’t be too broadly drawn (i.e. the time limit restriction) and should
factor in the public interest (i.e. other doctors in town?)

Shafron v KRG Insurance Brokers [2009] SCC


Facts: D sold his insurance brokerage for cash and submitted to a restraint of trade clause that stated
that he would continue working there, but would be unable to seek employment in the same field
with another brokerage in the “Metropolitan City of Vancouver.” The business was later sold to P, but
D’s employment continued under similar circumstances. Later, D left the company and began
working for another brokerage. P sued D for breach of K.
Issue: Whether the restraint of trade clause in K is enforceable on its face

75
Decision: Judgment for D. Non-compete is unenforceable.
Reasons: Restraint of trade clause in K is not enforceable because of the language describing its
geographical limits is ambiguous (“The Metropolitan City of Vancouver”). However, but for its
ambiguity, the restraint of trade clause might be enforceable because it occurs within the context of a
vendor/buyer relationship, and not simply an employer/employee relationship.
Ratio: Test for determining if restraint of trade clause is enforceable:
General Rule: Construe non-compete clauses strictly against the employer/purchaser. They are
enforceable if tightly drawn.
• Is non-compete has been drafted in a way that is clear and unambiguous?
• It’s easier to enforce a non-compete in the sale of the business over an individual. In a business,
you buy the goodwill of the business and also the parties are on a more equal playing field.
1. Is the restraint reasonable between the parties? Will the customers follow the employee?
2. If yes, is the non-compete clause more broadly drawn than necessary to the
purchaser/employee’s valid business interests, in terms of geography and time? How much
distance is the right amount of distance? How long is the right amount of time?
3. If yes to the above, is the non-compete contrary to the public interest? How does it affect
essential services? Does it create a monopoly that will result in unreasonable price
increases?
The broader the geographical area and the longer the restraint of trade clause, the less likely
is the court to find the clause reasonable and therefore enforceable.

Illegality Based on Statute


• Either the contract is illegal in creation or the performance is illegal

Yango Pastoral Co v First Chicago Australia Ltd. [1978] AUS


Facts: Statute says its illegal to bank in AUS unless you have a banking license. Yango signs up for a
guaranteed loan before banking license is complete. The loan was secured by a mortgage,
incorporated a guarantee given by the other Ds. Yango’s loan goes into default and they say bank
shouldn’t be able to enforce guarantee bc loan was signed up for before the bank got their license.
Impliedly the K is illegal. When D defaulted on the loan, P sued Ds (Yango and guarantors).
Issue: Whether Banking Act on its proper construction, prohibited the making or performance of K
Holding: Contract. Judgment for P (bank).
Reasons: The purpose of the legislation was to ensure that the banks would carry on their business
in a way that keeps them solvent. The purpose is to protect individuals who are putting money in the
bank and not to protect those borrowing from the bank. In fact, D’s actions contradict the purpose of
the statute in that not paying loans tends to make banks insolvent. Further, the penalty imposes a
fine for every day that the bank carries business without a license, not for every loan contract the
bank makes without a contract. On this basis, the statute did not mean to prohibit these contracts.
The conclusion is that the bank pays the fine, but the court enforces the contract.
Ratio: In implied illegality cases, you have to apply a layered test.
TEST for indirect illegality:
1. What is the purpose of the statute, on its proper construction? Is the statute meant to protect the
person claiming the illegality?
a. If the plaintiff is protected under the statute, then is should be treated as enforceable
2. What is the penalty of the statute?
If yes and the penalty does not relate to the contract, then unlikely the contract will be found to be
illegal.

St. John Shipping Corp. v Joseph Rank Ltd. [1956] UK QB


Facts: P loaded his ship so that it was submerged 11 inches below the load line. The fine imposed on
P for doing so was a maximum of 100 pounds per inch, pursuant to the Merchant Shipping Act. P was

76
charged 1100 pounds for its infraction. P made an addition 2295 pounds for overloading, which
covered the fine and actually caused P to profit from wrongdoing. Ds being upset that their cargo
could have been potentially ruined by P’s negligent overloading, withheld the additional $2295 in
order to punish P further for their infraction. P sues in breach of K.
Issue: Whether D can set aside K because P performed the condition of K in an illegal manner
(overloading the ship beyond its load line)
Holding: Contract. Judgment for P.
Reasons: Contract itself wasn’t illegal, only the performance was and wasn’t done intentionally so.
Contract enforceable, and D must pay the shipping costs. The contract to ship your materials on that
boat was not illegal – it was the overloading = the performance of a contract in a certain way
Ratio:
General Rule:
A contract that is expressly or impliedly prohibited by statute is unenforceable, regardless of the
intentions of the parties to it. Application depends on proof of parties’ intentions to break the law at
the time the contract was made:
a. If one party intends to perform illegally then they can’t enforce contract, but other party can
b. If both parties agree to an illegal performance, then neither can enforce the contract
Express Prohibition by statute:
1. If K is expressly prohibited by statute or becomes an expressly prohibited K through the
performance of its conditions, then it is unenforceable.
Implied Prohibition by Statute:
1. If the statute impliedly prohibits the contract, the contract will also be unenforceable (St.
John’s Shipping)
• It will impliedly prohibit the contract if the statute makes an offence out of entering into
the contract
2. Otherwise, whether the contract is impliedly prohibited must be determined by:
• 1) By construing the statute
• 2) Asking what is the intention/purpose of the statute, and
• 3) Whether it was intended to protect the person asking to have the contract set aside
• 4) Whether there was a penalty for breach and whether it relates to the making of the
contract (Yango Pastoral Co. v First Chicago Australia)

Ashmore, Benson, Pease & Co v Dawson Ltd . [1973] UK CA


Facts: A piece of engineering equipment was being moved on a truck. It was too heavy and ended up
tipping over and causing damage to the equipment. Company (Ashmore) is trying to get damages
from the movers (Dawson) but movers say the load was too heavy for the vehicle (contrary to the
Road Traffic Act which had a 30 ton limit, and this load was 35 tons) and that the performance of the
contract was illegal.
Reasons DENNING: Contract is legal but performance is illegal. Contract is unenforceable b/c they
had to have known that it was illegal at the formation of the contract. It doesn’t matter if you agreed
to load the material illegally or not, it is a matter of if you should have KNOWN, on a reasonable look
at the facts. Person who was involved ensuring loading happened to the particular company had to
have known that it would be done illegally.
Ratio: DENNING: Neither party can enforce the contract is the parties agreed to the illegal
performance OR they must have known that the contract had to be performed illegally.

Unconscionability and Undue Influence


Duress A contract is set aside if one of the parties induced another party to enter into
such a contract through threats of physical violence
Previously thought could impose as much economic duress as one wanted, but it
may now be recognized: GFAA v NAV Canada, Williams v Roffey Bros
Remedy: Voidable Contract, Rescission

77
Undue When parties contract because one party had undue influence over another, whether
Influence the party with influence knew it or not.
In the past, needed to be in a special relationship (doctor, parent, fiduciary,
spouse)
Now no longer need to be in one of those categories if one can satisfy part 1 of
test for undue influence (trust/confidence)
TEST FOR UNDUE INFLUENCE Royal Bank of Scotland v Etridge
1. If one party placed sufficient trust of confidence in the other party (if
special relationship, this automatically is satisfied)
2. Other party uses that to their advantage (questionable contract)
o If 1 and 2 are satisfied, undue influence is presumed
o Rebuttable presumption: contract is only good if the stronger party
can show that:
1.The weaker party had actual ability to exercise free judgment
(independent legal advice or expertise) OR
2.The contract is fair
Remedy: Rescission
Unconscion- Where there is no threat of physical violence (duress) and no category of undue
ability influence relationships, but a party with a greater power overreaches and induces a
weaker party to enter into an improvident bargain, then the contract may be set
aside (Marshall v Canada Permanent, Macaulay v Schroeder Music)
Remedy: Rescission

Unconscionability Test (Macaulay v Schroeder Music, Marshall v Canada


Permanent)
1. The individual with the power advantage need not be aware of the
disproportionality between himself and the person who is entering into the
contract for the contract to be rescinded. It is enough that there is:
a. An inequality of bargaining power resulting from some “special
disadvantage” and
b. An abuse of the inequality, driving an improvident transaction for the
weaker party
c. Can be negated if there is independent legal advice or if party shown
to have expertise (CIBC v Rowett)
Situations in which unconscionability has been recognized:
1. An elderly infirmed man, with limited capacity to understand business
transactions (Marshall v Canada Permanent)
2. Insurer gets an injured insured on painkillers, to sign a waiver and settle on
unfair agreement (Pridmore v Calvert)
3. Young/musician/performer, with significant cash needs and who is
unrepresented, signs an improvident agreement with a record company
(Macaulay v Schroeder Music)
4. Duress and unconscionability: Husband gets an emotionally distraught wife on
sedatives and drinking brandy to sign a disadvantageous separation agreement
(Mundinger v Mundinger)
5. Bank takes man’s farm away to pay for son’s default loan through inequality of
bargaining power – no ILA (Lloyd Bank Limited)
6. Bank allows wives of several men charge interest in their family homes for
security on husband’s loans, doing so under undue influence and without ILA
(Royal Bank of Scotland)
Independent PRINCIPLE: These are automatically special kinds of cases bc the transactions are
Legal Advice generally entirely improvident. The guarantor receives no direct benefit but could be
(ILA) forced to assume a terrible consequence.

When there is evidence of inequality of bargaining power or undue influence,

78
bank must show that the weaker party received ILA in order for contract to be
enforceable DENNING – Lloyd Bank Limited v Bundy

TEST FOR LOAN GUARATEES: (Royal Bank of Scotland v Etridge)


When an individual guarantees something from which he receives no direct benefit,
independent legal advice is required, and that advice must involve a satisfactory
description of the risk in a face-to-face meeting, and the creditor must advise of any
unusual features of the obligation being guaranteed.

Presumption: When there is a special relationship, undue influence is presumed.


• Presumption is rebuttable with a certificate of ILA for guarantor
• Burden of proof is on person in stronger position to prove the bargain is fair
1. Did the person who gave the guarantee get the sort of advice that allowed them to
make a free decision? (Or an opportunity for ILA that they declined?)
• Steps to get a certificate of Independent Legal Advice:
1. Someone has to explain the documents to the person
2. Someone must explain the seriousness of the risk of default and the extent of
her liability
3. Someone has to explain that there is a choice and that the individual can
decline to act as a guarantor (even though person wouldn’t get the loan)
4. These steps must occur in the absence of the other party
5. ILA can be given by a lawyer that is associated with both parties, but the
guarantor must be offered the choice to get advice elsewhere – in ON has to
be from an independent lawyer
6. The individual must also act in the best interest of the guarantor
Have above requirements been met?
• YES The criteria for a certificate of ILA is satisfied and presumption is
rebutted. No undue influence in transaction, t/f no unconscionability
• NO Presumption of undue influence remains. Contract is voidable
o BUT was the person in a position to have enough knowledge to take
care of themselves? (worked in field – CIBC Mortgage)
YES Presumption rebutted. No undue influence in
transaction, therefore no unconscionability
NO Presumption of undue influence remains. Contract is
voidable.
2. No Did the financial institution know about the undue influence?
• YES Perhaps they should have insisted on ILA. May be voidable
• ILA does not always save the K, but may be fatal without it (Royal Bank of
Scotland)
• If knew about the undue influence, ILA doesn’t cleanse the presumption
of undue influence
• NO Harder to prove that institution was at fault.

CIBC Mortgage Corp v Rowatt [2002] ON CA


Ms. Rawhit is a real estate agent, there was a presumption of undue influence of her husband to
secure her interest in the family home to cover debt in his business
Court found that she had the knowledge due to her profession to know what she was doing
Presumption relationship to husband affected her choice
Rebuttal She had the knowledge to look after herself

Pridmore v Calvert
Person gets in car accident, insurance person shows up at door to help him out, says sign this
release form and I will cut you a chq for $10,000 right away

79
Person signs form, turns out he has long term injuries and would have needed more
Is there unconscionability? Was this an unfair bargain?
Court says he is not bound by the agreement. Person is entitled to place contract aside and sue
for real damages.

Mudinger v Mudinger ON CA 1968


Wife on sedatives, husband gives her glasses of brandy and gets her to sign a settlement
agreement that is unfair
Gross and overarching use of power? Unfair bargain?
Contract is voidable, and takes issue to court properly

Marshall v Canada Permanent Trust Co [1968] AL SC


Facts: P made a deal with an elderly man in a nursing home to buy his farm. After the contract was
worked out, Canada Permanent Trust, acting as Mr. Walsh’s committee for his estate, returned the
cheque bc Mr. Walsh was mentally incapable of transacting business or of protecting his interest. P
sued for specific performance of contract and sale of land.
Holding: Contract is voidable. Defendant entitled to rescission. Judgment for D.
Reasons: Marshall had stronger bargaining power. Does not matter if he knew about Mr. Walsh’s
condition or not bc power imbalance still existed. Transaction was unconscionable, unequal
bargaining power.
Ratio: This is an example of UNCONSCIONABILITY. The individual with the power advantage need
not be aware of the disproportionality between himself and the person who is entering into the
contract for the contract to be rescinded. It is enough that there is:
1. An inequality of bargaining power resulting from some “special disadvantage” and
2. An abuse of the inequality, driving an unfair transaction for the weaker party

Macaulay v Schroader Music [1974] EN


Facts: Young song-writer enters into a contract with music publishers that was exploitative and was
basically a restraint of trade for 10 years (company got full copyright, most of the royalties, all the
bargaining power, etc.)
Holding: Contract is unconscionable and is therefore set aside.
Reasons: It’s an unfair K because the music company has considerable power over the young artists.
Ratio: This is an example of UNCONSCIONABILITY. Influencing factor: Young, musician/performer
with significant cash needs and who is unrepresented, signs an improvident contract.

Loan Guarantee Cases


These are automatically a special kind of case because the transactions are generally entirely
unfair. The guarantor receives no direct benefit, but could be forced to assume a terrible
consequence

Lloyd’s Bank v Bundy [1975] HL


Facts: Bundy’s son’s business is going under and son’s bank says Bundy needs to act as son’s
guarantee and mortgage his home. Bundy agrees to help his son in whatever way he can. Problem is
that the bank knew that the son’s business was going to go bankrupt regardless of Bundy mortgaging
his house and they just wanted to lessen their losses. So, son goes bankrupt, bank wants to sell the
house and they bring action against Bundy to evict him.
Holding: DENNING: Contract is voidable due to unconscionability BUT if Herbert had had
independent legal advice, it would have been allowed.
Reasons: Transaction was unconscionable bc bank was simply saving themselves on the loan.

80
Shouldn’t have taken security away from Bundy when they knew the loan was going to fail.
Guarantee is unenforceable and set aside bc Bundy did not get legal advice. There is an inequality of
bargaining power based on:
1. Consideration from the bank was grossly inadequate
2. The relationship between the bank and Bundy was one of trust and confidence. Bundy relied
on and trusted the bank and the bank failed that trust
3. Relationship between son and Bundy was such that Bundy’s natural love and affection had
much influence over him.
Conflict of interest between bank and Bundy, the father should have gotten independent advice.
Ratio: A guarantee is not legally enforceable if the guarantor is a volunteer or a family member to the
debtor, unless that guarantor receives independent legal advice about the risks. We need real,
independent legal advice so that the guarantor knows the risks involved in their decision – a
volunteer needs to present a certificate of independent legal advice

Royal Bank of Scotland v Etridge [2001] HL – Undue Influence


Facts: A wife charged the interest in her home in favor of a bank as a security for her husband’s debt.
She later asserted that she signed the charge under undue influence of her husband. The bank tried
to enforce the document she signed, but wife’s defence was that the transaction was procured
through her husband’s undue influence.
Holding: Contract voidable bc of undue influence
Ratio: TEST FOR LOAN GUARANTEES
Presumption: When there is a person in a special relationship involved in a loan guarantee, the
presumption is that there is undue influence and the transaction is tainted.
• Presumption is rebuttable: with a certificate of independent legal advice for the guarantor
• Burden of proof is on the person in the stronger position to prove the bargain is fair
When an individual guarantees something from which he receives no direct benefit, independent
legal advice is required, and that advice must involve a satisfactory description of the risk in a face-
to-face meeting and the lawyer must advise of any unusual features of the obligation being
guaranteed.
1. Did the person who gave the guarantee get the sort of advice that allowed them to make a
free decision? (Or an opportunity for independent legal advice that they declined?)
• STEPS TO GET CERTIFICATE OF INDEPENDENT LEGAL ADVICE
1. Someone has to explain the documents to the person
2. Someone must explain the seriousness of the risk of default and the extent of her
liability
3. Someone must explain that there is a choice and that the individual can decline to
act as a guarantor (even though person wouldn’t get the loan)
4. These steps must occur in the absence of the other party
5. Independent legal advice can be given by a lawyer that is associated with both
parties, but the guarantor must be offered the choice to get advice elsewhere
6. The individual must also act in the best interest of the guarantor
• Have the above requirements been met?
o YES The criteria for a certificate of independent legal advice is satisfied and
presumption is rebutted. No undue influence in transaction
o No Presumption of undue influence remains. Contract is voidable.
Was the person in a position to have enough knowledge to take care of
themself? (i.e. worked in the field – CIBC Mortgage Corp v Rowatt
YES Presumption rebutted. No undue influence
NO Presumption of undue influence remains. Contract is
voidable
2. Did the financial institution know about the problem?
• YES voidable

81
E. Enforcement, Breach and Remedy
Privity
• The only people who can enforce a contract are the people who gave consideration to the
contract

General Rule Only the party who gives consideration can sue under a K. A third party cannot sue
of Privity through a contract which it is not a party, even though the contract may be
expressed for a third party’s benefit (Tweedle v Atkinson)
How do you Enforcement by other Party:
get around Other party to K can enforce specific performance. Damages are not appropriate
the general remedy bc the benefits are going to a 3rd party.
rule of • If party to K is dead, estate could sue for specific performance (Beswick v
privity? Beswick)

Three exceptions to the general rule of privity:


1. Trusts:
• Where a specific benefit on a contract is “held in trust” for an individual, that
individual can sue on that trust even though he is not party to the contract.
Requirements to use trust law:
o Both parties acknowledge that there is a benefit that accrues to 3rd party and
that the benefit is being “held in trust” for the benefit of the identifiable third
party that it be directly enforceable under trust law
o Trust law does not require a party to be named, but has to be identifiable (can
be unborn)

2. Agency (New Zealand Shipping Co v Satterthwaite & Co)


• Through the use of agency, a third party to a contract can shelter under the
protection of the contract (Person in contract acts as agent to 3 rd party)
• TEST 4 Conditions must be met to argue agency to gain the benefit of the
exclusion clause: (New Zealand)
1. Contract must make it clear that the third party is to be protected
2. Contract must make it clear that one party is contracting in two capacities:
As the principal under the main contract
As an agent for the third party in negotiating a separate contract
3. The agent has the authority to contract on behalf of the third parties, or the
third parties must later ratify the contract
Did the third party give permission for person to contract on their behalf?
Person must have given authority or you need to ratify that authority
after the fact
Consider: If you are affiliated with the third party in some way? (e.g.
subsidiary company) Do you know them? Pre-existing relationship?
o If not, should likely use a trust. They will always want to shelter
under the clause but the issue is whether or not they need to send a
notice of ratification
4. Third parties must give separate consideration to the contract
Scotson v Pegg – Agreeing with a second party to do what you’re already
bound to do with a first party is not good consideration. There is also extra
liability
RESULT: TWO CONTRACTS: - a second collateral contract is effectively created bw
the original party and third parties, which allows the third parties to shelter under
the conditions of the original contract.

82
Does the contract meet all four of those standards?
YES Can bypass the general rule of privity by creating two contracts
NO General rule of privity applies and third parties cannot enforce a contract that
they gave no consideration to.

3. Principled Exception to Employees (London Drugs v Kuehne & Nagel)


Application: Iaccobucci: court can apply principled exception (small excursions into
law) to general law of privity to render fair and equitable results.
• Applies in narrow circumstances – only to employees who are performing the
exact contract that contains the limitation clause.
Principled Exception Test: If the following conditions are met, there is a principled
exception to the law of privity, which allows employees to shelter under exclusion
clauses if:
1) Clause expressly or impliedly extends to the employees (it is clear this
happens)
2) Employee must be acting in the court of employment and providing the very
service provided for under the contract

Iaccobucci created this test around Greenwood Shopping Center Beatty:


• Employees working w welding torches burnt down mall, SCC said employees
can’t benefit from exclusion clause bc not performing exact contract – built
London Drugs around this
Does the contract meet these conditions?
YES Court is entitled to make principled exception to general rule of privity to
make fair and equitable results
NO General rule of privity applies and employee cannot shelter under the
exclusion clause.

Beswick v Beswick [1968] EN CA


• Mr. Beswick has a nephew and he is joining him in the business, when he dies he says nephew will
get the business, if he provides monthly income to his aunt for the rest of her life
• Nephew stops making payments bc the aunt can’t enforce the bargain
• Aunt is also the executor of uncle’s estate – acting on behalf of uncle, she sued for the enforcement
of the contract for specific performance

Tweedle v Atkinson [1861] UK


Facts: Father and father-in-law agree to provide a financial benefit to P, the son. Father dies and
father-in-law says he’s not going to provide the benefit. Son can’t enforce K bc he didn’t give
consideration to the original agreement.
Issue: Can the third party beneficiary enforce the contract?
Holding: Son cannot enforce the contract bc he didn’t give consideration to the original contract.
Reasoning: Only someone who has given consideration to contract can enforce, so although the
contract is for the P’s benefit, he cannot enforce it or sue on it. – Forbes: there are a lot of issues with
this
Ratio: General rule of privity is that a third party cannot sue through a contract to which it is not a
party, even though the contract may be expressed for the third party’s benefit.

New Zealand Shipping Co. v Satterthwaite & Co [1975] AUS Trust Case
Facts: Shipper and owner of drill press enter into K of shipping. Will ship drill press and K will be
performed when it’s delivered. Shipper needs a stevefore to unload boat and a delivery person to
perform the contract (third parties to contract). There is a limitation of liability clause in K between

83
shipper and owner. While being unloaded, stevedores drop and damage drill press.
Issue: Can stevedores shelter under the exclusion clause? Privity problem bc these ppl are third
party beneficiaries in the contract.
Reasons: In this case the contract was set up originally so that the exclusion clause protects all
parties to the performance, including third party beneficiaries. Have to make the contract clear that it
is intended to include third party protection. Contract has to say that shipper is contracting in two
capacities 1) as shipper 2) as agent of third party. Have to show that the carrier has the capacity to
make that contract or the third party agrees – have to agree to have them act as their agent.
Ratio: TEST – 4 conditions must be met to argue agency to gain benefit of the exclusion clause:
1. Contract must make it clear that the third party is intended to be protected
2. Contract must make it clear that carrier is contracting in two capacities: on its own and as an
agent for other people (third parties). As principal and as agent.
o Similar to a trust. You are acting in capacity to get exclusion clause for your own
benefit but also as an agent to enforce the exclusion clause for the benefit of the
stevedores. Court says that this must be made clear in K
3. The agent has to have the authority to contract on behalf of the third parties, or the third
parties must later ratify the contract
o If you enter into K on someone else’s behalf, they must have given you the authority
to do so as their agent on their behalf. Person must have given authority or you need
to ratify that authority after the fact
o Consider: Are you affiliated with the third party in some way? (e.g. subsidiary
company) Do you know them? Is there a pre-existing relationship?
o If not, you should likely use a trust. They would always want to shelter under the
clause, but the issue is whether or not they need to send a notice of ratification
4. Third parties must give consideration to the contract

At the end of this, there are really two contracts:


1. K between shipper and owner AND
2. K between shipper and stevedore.
o The second one is a collateral contract. In consideration of the stevedore unloading the drill
press, shipper agrees to limit their liability (you give a limitation of liability)
o It’s a collateral contract – collateral to main contract AND it’s a unilateral contract – unload
/promise limited liability
o But agreeing to perform a K that you’re already bound to perform is not good consideration.
Scottson v Pegg – BUT consideration here is unloading. Agreeing with a second party to do
what you’re already bound to do with a first party IS good consideration. There is also extra
liability
Court says if K is set up right and does all four of the requirements, then you can get around the rules
of privity by creating two contracts

Create a principled exception to the law of Privity


o SCC saying this isn’t working properly – going to change the law

London Drugs v Kuehne & Nagel Ltd. [1992] SCR


Facts: Owner of transformer and owner of a storage company have a K where storage company will
store transformer with a limitation of liability clause for damage over a certain amount. Two storage
company employees try to move it and damage it. Owner of transformer recognizes that there is a
limitation of liability clause, so he sues the employees. Union appears on behalf of employees.
Contract says limitation will apply to storage company and warehouse men. – There is no language
implying trust or agency – can’t create a collateral contract. Trust would have been easier plan (the
benefit of exclusion clause held in trust for all the employees as opposed to getting the agreement for

84
the act of an agency for all the employees)
Issue: Does limitation clause apply to workers?
Holding: IACCOBUCCI: Employees can rely on exclusion clause even though they were not privy to
the contract.
Reasons: “A Principled Exception to the Law of Privity” (new creation). There is clear identity of
interests bw employer and employees and no reason for denying the employees the benefit of the
contract. 1) Clause expressly or impliedly extends to the employees 2) If clause expressly or
impliedly extends, which it does here bc it says “warehouse men” and they are doing what was
supposed to be done under contract, there is a principled exception which entitles them to shelter
under the exclusion clause
Ratio: Principle Exception Test: If the following conditions are met, there is a principled exception
to the law of privity which allows employees to shelter under exclusion clauses if:
1) Clause expressly or impliedly extends to the employees
2) Employee must be acting in the course of employment and providing the very service
provided for under the contract
If so, courts are entitled to make principled exceptions, small excursions into the law, when
necessary to make fair and equitable results. This principled exception applies in narrow
circumstances – only to employees who are performing the exact contract that contains the
limitation clause.

Greenwood Shopping Center v Beatty [1980] SCC


Facts: Car repair place in shopping center has a limitation of liability clause in K with shopping
center. Employees are welding and start a fire that damages shopping center. Shopping center owner
sues the employees bc of limitation clause
Reasons: Employees cannot shelter under limitation clause bc not performing a duty that is
protected under the contract and exclusion clause
Ratio: In order to shelter under the exclusion clause, employee must be acting in the course of
employment and providing the very special service provided under the contract.

1. Frustration
• When will we allow a contract void when there is nothing wrong with the actual contract –
something happens to make the contract significantly different than was intended

DEFINITION: Frustration results from a mistake as to future matter. The Contract is valid, but
further performance may be excused when the future unanticipated event occurs.

OLD TEST (Taylor v Caldwell) – Implied term of K that performance excused when there is a future
event that renders performance impossible.

TEST FOR WHEN CONTRACT IS FRUSTRATED: (Davis Contractors v Fareham UDC)


On construction of the contract, in light of surrounding circumstances at the contract’s formation:
1) Is the required performance so fundamentally or radically different from what was
contemplated at the time of the K that the parties should be excused?
2) Can it be reasonably assumed that one party accepted the risk of the future event?
o If no party assumed the risk and was fundamentally different = Frustration
• Frustration is not a matter of an implied term of a contract, something that is not contemplated
by either party cannot be implied (David Contractors)
• Frustration does not depend on impossibility or severe difficulty – can always contract the
impossible and will breach if not done (David Contractors)
• Only if neither party assumed the risk, the contract is frustrated (Codelfa)
Remedies for Frustrated Contracts:

Frustrated Contracts Act: Remedies and Restitution for frustrated contracts

85
1. Expenses: Sums paid are to be returned subject to the deductions/expenses incurred before the
frustration
• All deposits must be returned less money spent on the performance
2. If a benefit is conferred to the other party to a contract, person is entitled to the fair market
value of the performance.
• If there are benefits conferred, you must pay the fair market value of the benefit (not the
contract price) – if P pays for 3 machines, D only makes 1 before frustration and need all
three to do task = no benefit conferred, no sum of money for that
3. If the whole performance is severable (or where part has been performed), you treat part
performance as if they were separate contracts.
• If the main contract is divisible into chunks, we may create new contracts based on this
division (i.e. part performance)
Force Majeure Clause: Can contract out of the Frustrated Goods Act. Needs to outline specific
remedies if specific situations occur
• However, according to Lord Reid in Davis Contractors, if the circumstances are written down
they will have been “contemplated” by both parties, so there is already no frustration and the
Act wouldn’t apply anyways!

Taylor v Caldwell [1863] UK – Old test


Facts: D signed lease to rent music hall for four days of concerts. Between time of signing contract
and the concert, hall burns down. P says that they can still do the concert on this land. D says that
wasn’t the deal and pleads mistake to a future event.
Holding: Judgment for D.
Reasons: Because the burning down of the music hall occurred through no fault of either party and
the existence of the hall was an implied term of contract in the original contract, D is not liable to P
for breach of contract. The burning down of the hall excuses both parties from performance of the
contract. – An implied term of the contract means that contract is not intended if something changes
the contract dramatically
Ratio: If the nature of the contract is such that the parties must have known at the time of
contracting that it cannot be fulfilled unless some specified thing continued to exist, it is not a
positive contract. An implied condition exists that the parties will be excused from performance if
that thing ceases to exist without fault of the parties.
Old Test: (1) Whether the performance would have been expected to continue in spite of the event;
was one of the parties expected to be responsible for this sort of risk? If so, there is no frustration. (2)
Was this a situation that was meant to be covered by the contract or is this an event that is so out of
contemplation that performance should be excused?

Davis Contractors v Fareham UDC –[1956] UK HL Principle Case


Facts: P and D have contract to build 78 houses in 8 months, subject to a penalty clause if not done
on time. At this time, skilled labor was hard to find and it ended up taking 22 months to finish the
contract. Plaintiff paid the penalty and lost a lot of money. He claims that contract should be
frustrated, he shouldn’t have to pay penalty and he should get fair value for performance of contract.
Holding: Accepted risk (Knowledge of skilled labor was well known) No frustration of contract.
Judgment for D
Reasons: REID: 1. Frustration is not an implied term (test in Taylor in Caldwell is wrong).
Frustration only occurs outside of anything that parties could have intended or contemplated,
therefore can’t be implied. 2. You don’t ask whether performance is impossible or more difficult.
People can contract to do impossible and will still be breach of K if can’t do it. To determine if K
is frustrated, must consider the contract and construe it as a whole in light of all the circumstances
that existed at the time of K. Ask if what happened is so fundamentally different that you could have
excused performance or if one of the parties agreed to accept the risk. Here, builder agreed to accept

86
the risk. At time of contract, it was clear there was a shortage of skilled labor and was known to both
parties and P took on K in spite of that. Fact that skilled labor became impossible to get shouldn’t
excuse performance.
Ratio: TEST for when a contract is frustrated:
The test is whether, during the construction of the contract and construing the contract in light of
surrounding circumstances at the date of the contract, it is reasonable to assume that one party
accepted the risk of the future event, or whether the required performance is so different from what
was contemplated that the parties should be excused for performing.
In order to have frustration, the performance under the new circumstances has to be fundamentally
or radically different than what was contemplated by the parties at the time of the contract.

Frustrated Contract Act, RSO 1990


Frustrated Contracts Act: Remedies and restitution for frustrated contracts:
1. Sums paid are to be returned subject to the deductions/expenses incurred before the
frustration,
• All deposits must be returned less money spent on the performance
2. If a benefit is conferred to the other party to a contract, person is entitled to the fair value of
performance
• If there are benefits conferred, you must pay the fair market value of the benefit (not the
contract price).
3. If the whole performance is severable (or where part has been performed), you can treat
part performance as if they were separate contracts
If the main contract is divisible into chunks, we may create new contracts based on this division (i.e.
part performance)
• Act states what happens when a contract is frustrated
• Can contract out of whole area of frustration:
Force Majeure Clause: see certain risks here, and in those events we agree that performance will
still occur (Times of war) = parties accepting the risks – must name the frustration events, results of
what happens in that event

Codelfa Construction v State Rail Authority


If you can’t imply a term in the contract – argue that the contract was frustrated
They were contracted to build subway in short amount of time and then were banned to construct
on weekends
Courts looked at Davis case and found that they think that both parties felt that an injunction
would not have been allowed and because they were not able to – he should receive the fair value
of the benefit that they conferred to the city
Codelfa lost on implied terms but won on Frustration

3) Remedy

Damages General Rules:


• Courts will generally grant a money away (damages) for breach of contract
Measure – • The general principle is that damages will put the plaintiff in the same situation as if
ment of the contract had been performed (Victoria Laundry v Newman Industries)
Damages • Generally, this is the difference in value between the contracted performance and
the actual performance.

Things that affect measurement: Specified liquidated damages in K beforehand OR


exclusion or limitation clauses

1. Personal Performance/Unique (Ruxley Electronics Ltd v Forsyth)

87
a. If the breach of contract arises in a personal situation, you should get the
money to complete the performance of K; unless that amount of money is
thought to be unreasonable
b. Consider: whether the remunerated party will actually use their damages to
rectify the breach of K
c. Is it reasonable for the P to do so? (objective)
If not reasonable, then maybe compensated nominal amount for loss
enjoyment (Ruxley)
2. Commercial Performance (Groves v John Wunder Co; Thompson V Robinson)
a. If it is a commercial contract/commonly replaceable good, you can recover the
loss stemming from the breach of contract, which means that you can
recover the difference between the market value of the performance and its
current value given the breach of contract (Groves v John Wunder)
3. Loss of Chance (Howe v Teefy)
a. Damages can also be awarded for loss of chance, even if actual loss is uncertain
b. TEST for Loss of Chance: Did Plaintiff:
Possess something which had a monetary value AND
Was deprived by D’s breach of contract
4. Sale of Goods/Property
a. Sale of Goods Act – s 48. When assessing damages in context of sale of
goods/property look at the difference between the contract price and the
market price (Sale of Goods Act)
b. When buyer refuses to accept a previously ordered good: Damages from lost
profit can be recoverable even if the profit could have been obtained later on
(Thompson v Robinson)
5. Punitive Damages – TEST – Whiten v Pilot Insurance
a. (1) Something particularly reprehensible has happened (conduct that
represents a marked departure from standards of decency) AND
b. (2) More than simply a breach of contract (i.e. not necessarily independent
tort but more than just a breach)
6. Intangible Losses
a. Courts will award damages for intangible losses (such as enjoyment and
mental distress) if the parties could have reasonably contemplated them at
the time the contract was made (Fidler v Sun Life; Jackson v Horizon). These
losses must not be remote pursuant to the remoteness test in Hadley v
Baxendale
7. Contract damages do not have to be certain to arise but there must be a serious
possibility or a real danger of the damage (Victoria Laundry)
Mitigation of Plaintiff must take reasonable steps to mitigate (or reduce) losses. In commercial
Damages contracts, it is reasonable to accept an offer from the party in default (ex: for means of
how else to pay) (Payzu v Saunders)
Remoteness Principle: The court tries to award such an amount of money for damages as would
of Damages put the plaintiff in the same situation monetarily had the contract not been breached.
Test for determining remoteness of damages (Hadley v Baxendale)
1. Arise in the ordinary course: What is fairly and reasonably considered as arising
naturally, according to the ordinary course of things, from the breach of contract?
• Objective test
• Consider: did damages result from ordinary course of events? Could a
reasonable person contemplate them?
2. Arise from Special Circumstances: If not ordinary, special damages must
reasonably have been in contemplation of both parties (communicated) at time
of K
• Subjective/Objective Test – Special Information measured against objective
reasonable person test

88
• Consider: What did the defendant actually know? Do they have special
knowledge? What things are reasonable in contemplation of both parties at the
time the contract is made? (Victoria v Laundry)
Specific Where damages are an ineffective remedy for reasons such as impossibility of
Performance calculating damages, unique/personal property in question, then the court will
or enforce other remedies:
Injunction
1. Specific Performance: court orders performance of contract. Used when damages
is not an appropriate remedy to get people to perform positive covenants
• Exception: Cannot enforce specific performance of personal services –
public policy (Warner Bros v Nelson; Co-operative Insurance)
2. Injunction: court orders person not to breach terms of contract. Used to get
people to adhere to negative covenants if performance under these conditions
does not amount to forcing the individual to perform the positive covenants.
(Warner Bros). This means restraining D from acting in breach of contract
• Application: Negative covenants will generally only be enforced if they do
not render the defendant (1) entirely idle (2) there is a limitation on time
and (3) enforcement will not compel the party to perform the contract
(Warner Bros v Nelson)
• Only valid within the jurisdiction of the court

Fuller:
1) Restitution Interest: If you confer a benefit in a contract and the other party doesn’t perform –
you’re going to get back what you would have if you performed the contract (give deposit, get it
back)
2) Reliance Interest: When spend money under reliance in a contractual performance – such an
amount of money to put in the same position as if the contract has been performed
3) Expectation Interest: the profits that I was going to make from the contract, should be entitled to
recover that as well
• 2 and 3 have an exception: Remoteness of damages (Hadley v Baxendale)

Damages
Interest Protected: Measurement
• How do we measure damages? Plaintiff has to prove the measure of their loss
• Gets tough when this is not a commercial contact – when purchase something with
sentimental value – how do we measure damages?
Commercial:
• Lesser of the cost of the performance and the economic value of performance
Personal:
• Cost of a replacement performance (provided that the amount is reasonable: i.e. will plaintiff
actually buy replacement performance Ruxley)

Ruxley Electronics Ltd v Forsyth [1994] UK HL


Facts: Contract to build a swimming pool. Express term of the contract was for the maximum depth
of pool to be 7 feet, 6 inches. Construction Company failed to provide the depth requirement. Mr.
Forsyth refuses to pay, so company sues him. Forsyth wants the amount of money it would cost to rip
pool out and rebuild so he can dive. Company says that isn’t reasonable bc Forsyth won’t actually use
the money to rebuild the pool. Trial judge holds $2500 for lost enjoyment. Appeal court says he
should get 65K
Issue: How much money should he get to put him in the same position as if contract had been
performed? Builder says 65K is too much bc land is worth the same with or without performance of

89
K
Holding: 2500 for loss of enjoyment
Reasons: HL – should get different between value of the property with the improvement and value
without the improvement. Would be unreasonable under circumstances to require rebuilding and to
give the money equivalent of fixing it because we don’t believe you will fix it. Issue is that there are
contracts that are commercial in nature and value that is suffered is a commercial amount – with
performance, financial value of property is increased. What about where performance is personal
(i.e. I want to be able to dive!) Difference in value is only 2500 for value of land. No defence to say
that you were saved money bc you didn’t get what you wanted – It is difficult in personal
performance to quantify the difference in value between the non-performance and performance
Ratio: If the breach of contract arises from a personal situation, you should get the money to
complete the performance of K; unless that amount of money is thought to be unreasonable (factor
considered: whether the remunerated party will actually use their damages to rectify the breach of
K).
a) Does the plaintiff have a serious intention of doing the work?
b) Is it reasonable for them to do the work?

Howe v Teefy [1927] EN


Facts: Plaintiff [Teefy] was leasing a horse off the Defendant for the purpose of racing it and
generally making money off it (through bets etc.). The Defendant randomly took the horse back. The
Plaintiff sued for damages, including the loss of chance (to win money through races, bets, selling
information on the horse to other gamblers etc.)
Issue: How do you compensate loss of chance (when the plaintiff can’t actually prove their damages)
Holding: Appeal dismissed, plaintiff keeps damages awarded.
Reasons: The agreement was made in place to give the Plaintiff a chance of making money off the
horse. He was deprived of that chance. The presence of contingencies (i.e. whether the horse would
win races, whether the Plaintiff would win bets etc.) do not render damages incapable of assessment.
The value of the chance can still be assessed. Put before the court what you get and ad the court will
try and determine what your damages are – will still accept and try to evaluate loss of chance
Ratio: Damages can also be awarded for the loss of chance, even if actual loss is uncertain. To
determine damages for loss of chance you must look at the circumstances to determine (in this case,
the court could look to see if the horse actually won any races). Compensation in these cases aren’t
very scientific, all the court can do is estimate what the losses are.
Forbes: Hire someone to drill a hole for oil, person doesn’t do it can’t sue for lost chance (don’t
know there is oil). Sue for damages for not completed performance & hire someone else
- If a lease depended on having the hole drilled – that is a loss of chance

Groves v John Wunder Co. [1939] USA – Forbes thinks wrongly decided
Facts: Groves leased some land to Wunder. Groves let Wunder use the land and take sand and gravel
from it. Wunder (D) was supposed to pay Groves $105,000 and leave the property at a “uniform
grade.” D intentionally breached and didn’t leave a uniform grade. It was found it would cost about
$60,000 to fix the grade, however, the property itself if the D had performed would only have been
worth about $16,000; without it, it is worth 4K. At trial, the plaintiff recovered the $16,000 plus
interest. The plaintiff appealed. – Value of land was supposed to be let at was $16,000 but it would
cost $60,000 to do it properly – what are the damages then?
Issue: How much damage should D pay to put P in the same amount as had the K been performed?
Holding: P should get 60K . Prof – this is crazy result bc it should be the difference (16,000 – 4000),
not the cost of the grading
Dissent: This award would put the plaintiff in a far better position than if performance would have
been done. Had the contract been performed, the plaintiff would have $12K, whereas the damages
suggested by the majority would give the plaintiff 60K

90
Ratio: If it is a commercial situation, you can recover the loss stemming from the breach of contract,
which means that you can recover the difference between the market value of the performance and
its current value given the breach of contract. (i.e. what would it have been worth if performance had
been completed – what it is worth w/o complete performance)

Sale of Goods Act, RSO 1990


s. 48 – When assessing damages in contracts of sale of goods/property look at the difference
between the contract price and the market price (K value subtract market price value)

Thompson Ltd. V Robinson Ltd. [1955] UK


Facts: D had contracted to buy a standard model car from P, who were dealers in cars. D, in breach of
K, refused to accept delivery of the car and P returned to their suppliers who took it back. P claimed
they lost profit that they would have made if D had bought the car. D relied on s 48 of the Sale of
Goods Act.
Holding: The court ruled that P could recover the losses resulting from the lost profit of not selling
the car to D. These losses are the difference between the market price of the car and the K price.
Ratio: Damages from lost profit can be recoverable even if the profit could have been obtained later
on.

Mitigation:
• Have to act in a reasonable fashion to limit my damages

Payzu v Saunders [1919] KB CA


Facts: D agreed to sell silk to P from Jan to April. Silk was delivered. Cheque from P to D was delayed.
D said they wouldn’t sell to P anymore unless they pay in cash for each order. D brought claim for
difference between the market price and contract price.
Ratio: Plaintiff must take reasonable steps to mitigate (or reduce) losses, or they will only be able to
recover limited damages. In commercial contracts, it is reasonable to accept an offer from the party
in default. In assessing damages for failure to perform, the court will take into account whatever the
plaintiff has or ought to have done to diminish his losses (they must act reasonably in the
circumstances = mitigate the loss)
Does mitigating damages mean going back to work with person that had breached the contract?
Duty to mitigate doesn’t require you to re-contract if there is a huge lack of trust but otherwise, duty
to mitigate does require to re-contract if it lessens the damages
Analysis: The plaintiffs were in the position to pay cash and instead of accepting the bona fide offer,
they decided to incur a substantial loss which a reasonable person would have avoided.

Remoteness

Hadley v Baxendale [1854] EN


Facts: P owned steam powered mill, send D a broken shaft as a pattern for a new one, delivery of
other shaft was delayed to D and subsequently back to P. P brought an action against D for loss of
profits during the mills idle time.
Issue: Should the defendants remunerate the plaintiffs for lost profits due to the breach of contract
resulting from delay in shipping?
Holding: Alderson: Ruled in favor of D
Reasons: 1) It was not ordinary: the court held that the damage was not foreseeable by D at the time
of the contract – broken shaft was a special circumstance not known to D. 2) Special circumstance
was not communicated by P
RATIO: Two stage test for determining remoteness of damages:

91
1) Arise in the ordinary course: What is fairly and reasonably considered arising naturally from
the breach of the contract itself?
2) Arise from Special Circumstance: If special circumstance exist, both parties must be aware of
circumstances for breach to amount to damage that includes this special circumstance

Victoria Laundry Ltd v Newman Ltd [1949] KB CA


Facts: Laundry Company (P) buys boiler from D (company of engineers), say they need it ASAP.
Delivery delayed for 20 weeks (3rd party delivery injures boiler). P argues they are owed damages for
lost profits caused by the boiler bc they could have taken on more customers and lost a highly
lucrative dying contract with government.
Issues: Is P owed damages for breach of contract?
Holding: ASQUITH LJ: Yes, because D had special knowledge as engineers and that company needed
it ASAP, that they would lost profits. Not reasonably foreseeable that they lost a large deal.
Reasons: Applied Hadley v Baxendale Test: 1) Reasonable person would NOT have foreseen lost
profits 2) Ds being engineers and knowing the time sensitivity, had specialized knowledge of the use
for the boilers and would foresee the lost profits for the boiler not being delivered on time.
RATIO: In breached contracts, the party is only able to recover the part of the loss reasonably
foreseeable at the time of the contract. It is enough if the party is likely to see the loss. Use Hadley v
Baxendale

Intangible Injuries/Punitive Damages


• If contract is for enjoyment, peace of mind – breach of contract may give rise to recovery for
upset, loss of holiday etc. – get an amount of money to compensate for the loss of enjoyment

Fidler v Sun Life [2006] SCC


Facts: Insured was diagnosed with chronic fatigue syndrome and fibromyalgia and was receiving
long term disability benefits. Despite lack of medical evidence that insured could return to work,
insurer terminated benefits after video surveillance showed insured driving and shopping. Insured
brought action for payment of benefits. One week prior to trial, insurer agreed to reinstate benefits
and pay outstanding amounts plus interest. Trial judge awarded insured aggravated damages of
$20,000. Award was upheld on appeal, insurer appealed again.
Issue: Is loss of enjoyment, mental distress actionable in contract?
Holding: Trial judgment restored. 20K for aggravated damages.
Reasons: SCC agreed with trial judge that punitive damages weren’t appropriate in this case. While
the insurer’s conduct in denying for over five years in the absence of medical support was extremely
troubling, it was not sufficiently so as to justify interfering with the trial judge’s conclusion that there
was no bad faith to warrant an award of punitive damages. To attract punitive damages, the insurer’s
conduct had to depart markedly from ordinary standards of decency and be independently
actionable.
Ratio: Damages are subject to the remoteness test set out in Hadley v Baxendale. Damages (1) must
be foreseeably to have arisen naturally in the ordinary course of things from the breach, or (2) are
within the contemplation of the parties at the time of the contract to the probable result of the breach
(“special circumstances” are not contemplated, so need to be communicated to recover damages.
Reasonable time/notice following communication is required). Loss of enjoyment and/or mental
distress is recoverable under these two heads.
- Possible to get mental distress damages in contract but has to fall under Hadley v Baxendale

Jackson v Horizon Holidays [1975] EN CA


Facts: Bad trip to Sri Lanka. Damages for upset and loss of enjoyment. Privity issue regarding other
family members. Wanted damages for the loss of holiday of him and his family – poor quality hotel.

92
DENNING: Greater loss of enjoyment because of LOE of family members. Families look forward to a
holiday, it is difficult to assign monetary value to but it is the task of judges. Dismissed appeal with
damages decided by trial judge. James LJ: possible to say he was acting as agent and collect directly?

Do we award punitive damages for breach of contract? Maybe

Whiten v Pilot Insurance [2002] SCC


Facts: Whiten’s family home burned down in the middle of winter and they lost everything.
Insurance company refused to pay the insurance claim and alleged that the family had committed
arson (which was found to have no air of reality). The denial of the insurance claim was designed to
force the family to settle for less than what they were entitled to. Trial jury awarded family $1 million
punitive damages. ONT CA reduced this to $100K. Family appealed to SCC.
Issue: Should the award be returned to original?
Holding: Although high, award did not exceed limit of reasonable damages. The award of punitive
damages was a rational response by the jurors to the evidence before them. The insurer’s conduct in
maintaining the allegation of arson exceeded the requirements of prudent practice or due diligence.
The award of punitive damages was proportionate to the harm or potential harm directed at the
insured by the insurer, 1 million given back.
Ratio: Punitive damages may be awarded if:
1. Something particularly reprehensible has happened (conduct that represents a marked departure
from standards of decency) and
2. More than just a simple breach of contract (i.e. not necessarily an independent tort but more than
just a breach. This is about deterrence)
This was simply an attempt to make them agree to a lesser settlement amount – reprehensible –
Bhasin can very eventually cross over to Whiten

Specific Performance

Falke v Gray [1859]


Facts: Unique vases: agreed to buy vases, vases were worth ore than that price. D sold to third party
instead (for higher price). P sues for SP.
Reasons: If chattel is unique and damages won’t be an effective remedy SP (unless sold to 3P
damages will have to suffice)
Ratio: AP is an effective remedy, unless item sold to a third party.

Co-operative Insurance Society v Argyll Stores [1998] HL


Facts: Ongoing 30 yr contract. P wants SP to force D to uphold contract.
Holding: No SP. Overall, would be unfair to D to order SP.
Reasons: SP is a waste:
• Can’t force someone to run a business
• Economic loss would occur (also, waste of personal time)
• Hard to frame court order
• Wasteful litigation over compliance – continues court monitoring
• Oppressive: running business under fear of contempt
Against public interest if compensation was a plausible alternative.
Ratio: SP will not be ordered when compensation will suffice. Court will not force someone into
oppressive regime with SP.

Injunction:

93
Warner Bros v Nelson [1937] EN (Bette Davis Case)
Facts: Bette Davis (D) has a contract to give her services as an actress to Warner Bros (positive
covenant) and it prevents her from taking any other employment or acting in any other shows
(negative covenant). Davis felt too restricted by her contract with Warner Bros and her parts were
too limited. Went to England to perform on their stage. Warner Bros sought an action for SP.
Issue: Should D be forced into an injunction or SP to carry out the negative covenants of the
contract?
Holding BRANSON: Injunction should be granted and question remains of the period that the
injunction should operate.
Reasons: No jurisdiction can order SP of positive covenants = PUBLIC POLICY; cannot order personal
services. Injunction is imperative because terms of contract are she is “special, unique” and “loss
cannot be reasonably or adequately compensated in damages”
Ratio: Equitable remedies are used when damages are not appropriate. Positive stipulations feed
specific performance. Negative stipulations feed injunctions. Can’t order an equitable remedy when
damages are an appropriate remedy under the circumstances.

94

You might also like